Re: [obm-l] problema de probabilidade

2022-11-09 Por tôpico Claudio Buffara
Essa também:
https://thedailyviz.com/2016/09/17/how-common-is-your-birthday-dailyviz/


On Wed, Nov 9, 2022 at 12:04 PM Claudio Buffara 
wrote:

> Achei isso aqui interessante: https://www.panix.com/~murphy/bday.html
>
> []s,
> Claudio.
>
> On Tue, Nov 8, 2022 at 9:56 PM Ralph Costa Teixeira 
> wrote:
>
>> Mis ou menos... O que faltou foi a hipótese exata da distribuição de
>> probabilidade dos aniversários.
>>
>> Se a gente supõe que cada mês tem os mesmos 1/12 de chance para cada
>> aluno, e que os meses são independentes entre si, sim,
>> p=12/12^2=1/12~8.3%.
>>
>> Agora, talvez um modelo um pouco mais preciso seria supor que cada DIA do
>> ano tem a mesma probabilidade (e que são independentes entre si). Isto
>> afeta um tiquinho a resposta, porque cada mes têm um número ligeiramente
>> diferente de dias! Ignorando anos bissextos (huh!?!), temos:
>> -- 7 meses com 31 dias;
>> -- 4 meses com 30 dias;
>> -- 1 mes com 28 dias;
>> Portanto, seria um pouco mais "realista" usar:
>> p=(7*31^2+4*30^2+28^2)/(365^2) ~ 8.34003%
>>
>> Eu ponho esse "realista" bem entre aspas; primeiro, porque eu ignorei
>> anos bissextos (fique à vontade para inclui-los e refazer a conta :D :D
>> :D); mas a hipótese de que todos os dias do ano tem a mesma probabilidade
>> não é tão realista quanto parece! Existe uma certa "concentração" de
>> aniversários em determinadas épocas do ano... mas, sem dados exatos sobre
>> como seja a tal concentração, o melhor que podemos fazer seria uma das
>> estimativas acima.
>>
>> Ainda tem um segundo problema sutil: *mesmo que todos os dias tivessem a
>> mesma probabilidade, talvez n*ã*o seja 100% correto supor que os
>> aniversários dos alunos da mesma turma do CMBel sejam independentes*!
>> Por exemplo, existe uma probabilidade maior que zero de ter gêmeos numa
>> mesma turma (comum uma família com gêmeos colocá-los na mesma escola), o
>> que afeta a independência dos dados, e muda um pouquinho aqueles 8.3% (para
>> cima)... sem uma estimativa desta probabilidade de ter gêmeos na mesma
>> turma, não conseguimos calcular a resposta "exata".
>>
>> Isto tudo dito... em quase qualquer problema de probabilidade a gente vai
>> ter que fazer ALGUMA hipótese simplificadora para poder sair do lugar.
>> Assim, eu diria que o problema não está 100% bem posto, mas não acho
>> ridículo fazer uma das hipóteses simplificadoras acima que levam a 8.3%
>> ou 8.34003% (e a diferença me parece tão pequena que eu aceitaria ambas as
>> respostas como corretas, desde que as hipóteses utilizadas em cada caso
>> fossem citadas).
>>
>> Abraço, Ralph.
>>
>> On Tue, Nov 8, 2022 at 3:07 PM Luis Paulo  wrote:
>>
>>> Prezados, o problema abaixo está bem posto?
>>>
>>> Uma turma do CMBel tem 25 alunos. Escolhendo-se aleatoriamente dois
>>> estudantes dessa turma, qual a probabilidade de eles façam aniversário no
>>> mesmo mês?
>>>
>>> A resposta da banca: 1/12.
>>>
>>>
>>>
>>> --
>>> Esta mensagem foi verificada pelo sistema de antivírus e
>>> acredita-se estar livre de perigo.
>>
>>
>> --
>> Esta mensagem foi verificada pelo sistema de antivírus e
>> acredita-se estar livre de perigo.
>
>

-- 
Esta mensagem foi verificada pelo sistema de antiv�rus e
 acredita-se estar livre de perigo.



Re: [obm-l] problema de probabilidade

2022-11-09 Por tôpico Claudio Buffara
Achei isso aqui interessante: https://www.panix.com/~murphy/bday.html

[]s,
Claudio.

On Tue, Nov 8, 2022 at 9:56 PM Ralph Costa Teixeira 
wrote:

> Mis ou menos... O que faltou foi a hipótese exata da distribuição de
> probabilidade dos aniversários.
>
> Se a gente supõe que cada mês tem os mesmos 1/12 de chance para cada
> aluno, e que os meses são independentes entre si, sim,
> p=12/12^2=1/12~8.3%.
>
> Agora, talvez um modelo um pouco mais preciso seria supor que cada DIA do
> ano tem a mesma probabilidade (e que são independentes entre si). Isto
> afeta um tiquinho a resposta, porque cada mes têm um número ligeiramente
> diferente de dias! Ignorando anos bissextos (huh!?!), temos:
> -- 7 meses com 31 dias;
> -- 4 meses com 30 dias;
> -- 1 mes com 28 dias;
> Portanto, seria um pouco mais "realista" usar:
> p=(7*31^2+4*30^2+28^2)/(365^2) ~ 8.34003%
>
> Eu ponho esse "realista" bem entre aspas; primeiro, porque eu ignorei
> anos bissextos (fique à vontade para inclui-los e refazer a conta :D :D
> :D); mas a hipótese de que todos os dias do ano tem a mesma probabilidade
> não é tão realista quanto parece! Existe uma certa "concentração" de
> aniversários em determinadas épocas do ano... mas, sem dados exatos sobre
> como seja a tal concentração, o melhor que podemos fazer seria uma das
> estimativas acima.
>
> Ainda tem um segundo problema sutil: *mesmo que todos os dias tivessem a
> mesma probabilidade, talvez n*ã*o seja 100% correto supor que os
> aniversários dos alunos da mesma turma do CMBel sejam independentes*! Por
> exemplo, existe uma probabilidade maior que zero de ter gêmeos numa mesma
> turma (comum uma família com gêmeos colocá-los na mesma escola), o que
> afeta a independência dos dados, e muda um pouquinho aqueles 8.3% (para
> cima)... sem uma estimativa desta probabilidade de ter gêmeos na mesma
> turma, não conseguimos calcular a resposta "exata".
>
> Isto tudo dito... em quase qualquer problema de probabilidade a gente vai
> ter que fazer ALGUMA hipótese simplificadora para poder sair do lugar.
> Assim, eu diria que o problema não está 100% bem posto, mas não acho
> ridículo fazer uma das hipóteses simplificadoras acima que levam a 8.3%
> ou 8.34003% (e a diferença me parece tão pequena que eu aceitaria ambas as
> respostas como corretas, desde que as hipóteses utilizadas em cada caso
> fossem citadas).
>
> Abraço, Ralph.
>
> On Tue, Nov 8, 2022 at 3:07 PM Luis Paulo  wrote:
>
>> Prezados, o problema abaixo está bem posto?
>>
>> Uma turma do CMBel tem 25 alunos. Escolhendo-se aleatoriamente dois
>> estudantes dessa turma, qual a probabilidade de eles façam aniversário no
>> mesmo mês?
>>
>> A resposta da banca: 1/12.
>>
>>
>>
>> --
>> Esta mensagem foi verificada pelo sistema de antivírus e
>> acredita-se estar livre de perigo.
>
>
> --
> Esta mensagem foi verificada pelo sistema de antivírus e
> acredita-se estar livre de perigo.

-- 
Esta mensagem foi verificada pelo sistema de antiv�rus e
 acredita-se estar livre de perigo.



Re: [obm-l] problema de probabilidade

2022-11-09 Por tôpico Anderson Torres
Em ter, 8 de nov de 2022 21:55, Ralph Costa Teixeira 
escreveu:

> Mis ou menos... O que faltou foi a hipótese exata da distribuição de
> probabilidade dos aniversários.
>
> Se a gente supõe que cada mês tem os mesmos 1/12 de chance para cada
> aluno, e que os meses são independentes entre si, sim,
> p=12/12^2=1/12~8.3%.
>
> Agora, talvez um modelo um pouco mais preciso seria supor que cada DIA do
> ano tem a mesma probabilidade (e que são independentes entre si). Isto
> afeta um tiquinho a resposta, porque cada mes têm um número ligeiramente
> diferente de dias! Ignorando anos bissextos (huh!?!), temos:
> -- 7 meses com 31 dias;
> -- 4 meses com 30 dias;
> -- 1 mes com 28 dias;
> Portanto, seria um pouco mais "realista" usar:
> p=(7*31^2+4*30^2+28^2)/(365^2) ~ 8.34003%
>
> Eu ponho esse "realista" bem entre aspas; primeiro, porque eu ignorei
> anos bissextos (fique à vontade para inclui-los e refazer a conta :D :D
> :D); mas a hipótese de que todos os dias do ano tem a mesma probabilidade
> não é tão realista quanto parece! Existe uma certa "concentração" de
> aniversários em determinadas épocas do ano... mas, sem dados exatos sobre
> como seja a tal concentração, o melhor que podemos fazer seria uma das
> estimativas acima.
>

Em uma turma com tão pouca gente, eu acho que considerações como "a
concentração de pessoas concebidas no Carnaval" podem ser ignoradas para um
problema tão simples. E, pelo que se nota, a conta mais limpa dá uma
diferença minúscula, 0,01%. Desconheço aplicação tão precisa na prática.


> Ainda tem um segundo problema sutil: *mesmo que todos os dias tivessem a
> mesma probabilidade, talvez n*ã*o seja 100% correto supor que os
> aniversários dos alunos da mesma turma do CMBel sejam independentes*! Por
> exemplo, existe uma probabilidade maior que zero de ter gêmeos numa mesma
> turma (comum uma família com gêmeos colocá-los na mesma escola), o que
> afeta a independência dos dados, e muda um pouquinho aqueles 8.3% (para
> cima)... sem uma estimativa desta probabilidade de ter gêmeos na mesma
> turma, não conseguimos calcular a resposta "exata".
>
> Isto tudo dito... em quase qualquer problema de probabilidade a gente vai
> ter que fazer ALGUMA hipótese simplificadora para poder sair do lugar.
> Assim, eu diria que o problema não está 100% bem posto, mas não acho
> ridículo fazer uma das hipóteses simplificadoras acima que levam a 8.3%
> ou 8.34003% (e a diferença me parece tão pequena que eu aceitaria ambas as
> respostas como corretas, desde que as hipóteses utilizadas em cada caso
> fossem citadas).
>
> Abraço, Ralph.
>
> On Tue, Nov 8, 2022 at 3:07 PM Luis Paulo  wrote:
>
>> Prezados, o problema abaixo está bem posto?
>>
>> Uma turma do CMBel tem 25 alunos. Escolhendo-se aleatoriamente dois
>> estudantes dessa turma, qual a probabilidade de eles façam aniversário no
>> mesmo mês?
>>
>> A resposta da banca: 1/12.
>>
>>
>>
>> --
>> Esta mensagem foi verificada pelo sistema de antivírus e
>> acredita-se estar livre de perigo.
>
>
> --
> Esta mensagem foi verificada pelo sistema de antivírus e
> acredita-se estar livre de perigo.

-- 
Esta mensagem foi verificada pelo sistema de antiv�rus e
 acredita-se estar livre de perigo.



Re: [obm-l] problema de probabilidade

2022-11-08 Por tôpico Ralph Costa Teixeira
Mis ou menos... O que faltou foi a hipótese exata da distribuição de
probabilidade dos aniversários.

Se a gente supõe que cada mês tem os mesmos 1/12 de chance para cada aluno,
e que os meses são independentes entre si, sim, p=12/12^2=1/12~8.3%.

Agora, talvez um modelo um pouco mais preciso seria supor que cada DIA do
ano tem a mesma probabilidade (e que são independentes entre si). Isto
afeta um tiquinho a resposta, porque cada mes têm um número ligeiramente
diferente de dias! Ignorando anos bissextos (huh!?!), temos:
-- 7 meses com 31 dias;
-- 4 meses com 30 dias;
-- 1 mes com 28 dias;
Portanto, seria um pouco mais "realista" usar:
p=(7*31^2+4*30^2+28^2)/(365^2) ~ 8.34003%

Eu ponho esse "realista" bem entre aspas; primeiro, porque eu ignorei
anos bissextos (fique à vontade para inclui-los e refazer a conta :D :D
:D); mas a hipótese de que todos os dias do ano tem a mesma probabilidade
não é tão realista quanto parece! Existe uma certa "concentração" de
aniversários em determinadas épocas do ano... mas, sem dados exatos sobre
como seja a tal concentração, o melhor que podemos fazer seria uma das
estimativas acima.

Ainda tem um segundo problema sutil: *mesmo que todos os dias tivessem a
mesma probabilidade, talvez n*ã*o seja 100% correto supor que os
aniversários dos alunos da mesma turma do CMBel sejam independentes*! Por
exemplo, existe uma probabilidade maior que zero de ter gêmeos numa mesma
turma (comum uma família com gêmeos colocá-los na mesma escola), o que
afeta a independência dos dados, e muda um pouquinho aqueles 8.3% (para
cima)... sem uma estimativa desta probabilidade de ter gêmeos na mesma
turma, não conseguimos calcular a resposta "exata".

Isto tudo dito... em quase qualquer problema de probabilidade a gente vai
ter que fazer ALGUMA hipótese simplificadora para poder sair do lugar.
Assim, eu diria que o problema não está 100% bem posto, mas não acho
ridículo fazer uma das hipóteses simplificadoras acima que levam a 8.3%
ou 8.34003% (e a diferença me parece tão pequena que eu aceitaria ambas as
respostas como corretas, desde que as hipóteses utilizadas em cada caso
fossem citadas).

Abraço, Ralph.

On Tue, Nov 8, 2022 at 3:07 PM Luis Paulo  wrote:

> Prezados, o problema abaixo está bem posto?
>
> Uma turma do CMBel tem 25 alunos. Escolhendo-se aleatoriamente dois
> estudantes dessa turma, qual a probabilidade de eles façam aniversário no
> mesmo mês?
>
> A resposta da banca: 1/12.
>
>
>
> --
> Esta mensagem foi verificada pelo sistema de antivírus e
> acredita-se estar livre de perigo.

-- 
Esta mensagem foi verificada pelo sistema de antiv�rus e
 acredita-se estar livre de perigo.



Re: [obm-l] Problema da IMO

2020-09-22 Por tôpico Anderson Torres
Em sáb., 12 de set. de 2020 às 01:18, Pedro José  escreveu:
>
> Boa noite!
> Atrapalhou meu vinho e o filme que estava assistindo mas consegui. Não gostei 
> tanto, agora que consegui, é muito trabalhoso.
>
> 2= [3(y+1)(z+1)-1]/2yz
> yz= 3(yz+2) (i)
> z(y-3)= 3y +2 (ii)
> y(z-3)=3z+2 (iii)
> (i)*(ii) yz(z-3)(y-3)= 9yz+6(y+z)+4 e Voilá: (z-3)(y-3)=11.
>
> Saudações,
> PJMS
>
>
>
>
>
> Em sáb., 12 de set. de 2020 às 00:35, Pedro José  
> escreveu:
>>
>> Boa noite!
>> Fui em uma linha parecida com a primeira solução, embora não visse 
>> necessidade de mudança de variáveis.
>> Mas o b achei sempre por restrição.
>> Esse "it implies" e aparece um número fatorado, não consegui captar, embora 
>> tenha gostado do recurso, já que é bem restritivo.
>>
>> Sudações,
>> PJMS
>>
>>
>> Em sáb., 12 de set. de 2020 às 00:08, Pedro José  
>> escreveu:
>>>
>>> Boa noite!
>>> Grato, Ralph!
>>>
>>> Estou estudando a solução. Pelo menos, não me decepcionei. A resposta 
>>> estava correta,
>>>
>>> Saudações.
>>> PJMS
>>>
>>> Em sex., 11 de set. de 2020 às 22:33, Ralph Costa Teixeira 
>>>  escreveu:

 Essa eh da IMO 1992. Tem uma solucao aqui:
 http://sms.math.nus.edu.sg/Simo/IMO_Problems/92.pdf

 On Fri, Sep 11, 2020 at 10:06 PM Pedro José  wrote:
>
> Bom dia!
>
> Recebi de um filho de um amigo, um problema que já o fizera.
> (a-1)(b-1)(c-1) | abc-1;  1
> Confesso que desta feita gastei mais tempo que da  primeira vez. Curioso, 
> da primeira ,eu pensei, dessa vez, eu tentei lembrar como eu resolvera, 
> aí nem lembrava, nem pensava. Apelei para a internet, mas não encontrei 
> nada. Mas no fim, recordei o que havia feito.
> (1+1/(a-1))(1+1/(b-1))(1+1/(c-1)) = k, onde k é inteiro.
> vê-se que k>1, e para um dado a k é máximo para b e c mínimos logo b=a+1 
> e c=a+2
> [a(a+1)(a+2)]/[(a-1)(a)(a+1)] > [a(a+1)(a+2)-1]/[(a-1)(a)(a+1)]>=2, então 
> (a+2)/(a-1)>2 ==> a <4, a=2 ou a=3.
> O k é máximo para a=2, b=3 e c=4 ==> k <4, logo k=2 ou k=3.
> S.p.g, se a é ímpar (a-1)(b-1)(c-1) é par; então b,c ímpares e k é livre.
> S.p.g se a é par abc-1 é ímpar; então b,c são pares e k ímpar.
> a=2, temos  2b(b+1)/[(b-1)b] >3, não usei a restrição de paridade para c 
> para facilitar a simplificação. b<5 Logo a=2 k é ímpar k=3. Logo c= 8. (2,4,8) é uma solução.
>
> a=3 temos 3b(b+1)/[2(b-1)b] > 2; b<7 e 3 para a=3 e b=5. kmax <= (15*7-1)/(2*4*6) <=2;pois k é inteiro.
> 1 k=2 e c= 15. (3,5,15) é a outra solução.
>
> Só agora me apercebi de que c=ab nas duas soluções. Então tentei uma nova 
> solução.
> (a-1)(b-1)(c-1) | abc-1 e (a-1)(b-1)(c-1) | abc + c(1 - (a+b)) -ab+ (a+b) 
> - 1 logo divide a diferença:
> (a-1)(b-1)(c-1) | (a+b) (c-1) + ab -1 - (c-1) logo c-1 | ab-1, então 
> ab-1= w(c-1), para algum w inteiro e ab=w(c-1) +1 (i)
> Como a=2 ou a=3
> Se a=2. e w>=2
> Temos por (i) 2b>= 2 (c-1) +1 c-1>=b, logo absurdo.
> Se a=3
> Temos por (i) 3b>= w(c-1)+1; w=3 ==>3b< 3 (c-1) +1 pois c>b
> w=2 ==> 3b =2(c-1) +1 ==> c=(3b+1)/2
> 2(b-1)(3b-1)/2 | 3b(3b+1)/2 -1 ==>  2(b-1)(3b-1)/ | 3b(3b+1) -2 ==> 
> 6b^2-8b-2 | 9b^2+3b-1 ==>  6b^2-8b-2 | 3b^2 +11b+ 4
> ==> b <=5. Como b>a=3 ==> b=5 e c= 8, ferindo a paridade.
> Logo ab-1=c-1 ==> ab=c ==> (a-1)(b-1)(c-1) | c^2-1 ==>  (a-1)(b-1) |  c+1 
> (a-1)(b-1) |ab+1==> (a-1)(b-1)!a+b
> a=2 ==> 2b-2= 2+b. b=4 e c=ab=8 (2,4,8)
> a=3 ==> 2(b-1) | 3+b ==> 2(b-1) = 3+. b=5 e c=ab=15. (3,5,15),
> Forcei um pouco a barra para mostrar que c=ab.
> Alguém teria uma outra solução, ou um endereço onde se tem as questões da 
> IMO e suas resoluções?

O bom e velho MathLinks! Ou melhor, AOPS:

https://artofproblemsolving.com/community/c89

A bem da verdade, eu tinha pego minha antiga papelada e convertido uma
imensa parte para LaTeX. Entre estas, estavam várias listas de
exercícios, além de problemas das IMOs. Em breve vou jogar no
Bitbucket.

>
> Grato!
> Saudações,
> PJMS
>
>
>
> --
> Esta mensagem foi verificada pelo sistema de antivírus e
> acredita-se estar livre de perigo.


 --
 Esta mensagem foi verificada pelo sistema de antivírus e
 acredita-se estar livre de perigo.
>
>
> --
> Esta mensagem foi verificada pelo sistema de antivírus e
> acredita-se estar livre de perigo.

-- 
Esta mensagem foi verificada pelo sistema de antiv�rus e
 acredita-se estar livre de perigo.


=
Instru��es para entrar na lista, sair da lista e usar a lista em
http://www.mat.puc-rio.br/~obmlistas/obm-l.html
=


Re: [obm-l] Problema da IMO

2020-09-11 Por tôpico Pedro José
Boa noite!
Atrapalhou meu vinho e o filme que estava assistindo mas consegui. Não
gostei tanto, agora que consegui, é muito trabalhoso.

2= [3(y+1)(z+1)-1]/2yz
yz= 3(yz+2) (i)
z(y-3)= 3y +2 (ii)
y(z-3)=3z+2 (iii)
(i)*(ii) yz(z-3)(y-3)= 9yz+6(y+z)+4 e Voilá: (z-3)(y-3)=11.

Saudações,
PJMS





Em sáb., 12 de set. de 2020 às 00:35, Pedro José 
escreveu:

> Boa noite!
> Fui em uma linha parecida com a primeira solução, embora não visse
> necessidade de mudança de variáveis.
> Mas o b achei sempre por restrição.
> Esse "it implies" e aparece um número fatorado, não consegui captar,
> embora tenha gostado do recurso, já que é bem restritivo.
>
> Sudações,
> PJMS
>
>
> Em sáb., 12 de set. de 2020 às 00:08, Pedro José 
> escreveu:
>
>> Boa noite!
>> Grato, Ralph!
>>
>> Estou estudando a solução. Pelo menos, não me decepcionei. A resposta
>> estava correta,
>>
>> Saudações.
>> PJMS
>>
>> Em sex., 11 de set. de 2020 às 22:33, Ralph Costa Teixeira <
>> ralp...@gmail.com> escreveu:
>>
>>> Essa eh da IMO 1992. Tem uma solucao aqui:
>>> http://sms.math.nus.edu.sg/Simo/IMO_Problems/92.pdf
>>>
>>> On Fri, Sep 11, 2020 at 10:06 PM Pedro José  wrote:
>>>
 Bom dia!

 Recebi de um filho de um amigo, um problema que já o fizera.
 (a-1)(b-1)(c-1) | abc-1;  1>>>
 Confesso que desta feita gastei mais tempo que da  primeira vez.
 Curioso, da primeira ,eu pensei, dessa vez, eu tentei lembrar como eu
 resolvera, aí nem lembrava, nem pensava. Apelei para a internet, mas não
 encontrei nada. Mas no fim, recordei o que havia feito.
 (1+1/(a-1))(1+1/(b-1))(1+1/(c-1)) = k, onde k é inteiro.
 vê-se que k>1, e para um dado a k é máximo para b e c mínimos logo
 b=a+1 e c=a+2
 [a(a+1)(a+2)]/[(a-1)(a)(a+1)] > [a(a+1)(a+2)-1]/[(a-1)(a)(a+1)]>=2,
 então (a+2)/(a-1)>2 ==> a <4, a=2 ou a=3.
 O k é máximo para a=2, b=3 e c=4 ==> k <4, logo k=2 ou k=3.
 S.p.g, se a é ímpar (a-1)(b-1)(c-1) é par; então b,c ímpares e k é
 livre.
 S.p.g se a é par abc-1 é ímpar; então b,c são pares e k ímpar.
 a=2, temos  2b(b+1)/[(b-1)b] >3, não usei a restrição de paridade para
 c para facilitar a simplificação. b<5 Logo a=2>>> k é ímpar k=3. Logo c= 8. (2,4,8) é uma solução.

 a=3 temos 3b(b+1)/[2(b-1)b] > 2; b<7 e 3>>> para a=3 e b=5. kmax <= (15*7-1)/(2*4*6) <=2;pois k é inteiro.
 1 k=2 e c= 15. (3,5,15) é a outra solução.

 Só agora me apercebi de que c=ab nas duas soluções. Então tentei uma
 nova solução.
 (a-1)(b-1)(c-1) | abc-1 e (a-1)(b-1)(c-1) | abc + c(1 - (a+b)) -ab+
 (a+b) - 1 logo divide a diferença:
 (a-1)(b-1)(c-1) | (a+b) (c-1) + ab -1 - (c-1) logo c-1 | ab-1, então
 ab-1= w(c-1), para algum w inteiro e ab=w(c-1) +1 (i)
 Como a=2 ou a=3
 Se a=2. e w>=2
 Temos por (i) 2b>= 2 (c-1) +1 c-1>=b, logo absurdo.
 Se a=3
 Temos por (i) 3b>= w(c-1)+1; w=3 ==>3b< 3 (c-1) +1 pois c>b
 w=2 ==> 3b =2(c-1) +1 ==> c=(3b+1)/2
 2(b-1)(3b-1)/2 | 3b(3b+1)/2 -1 ==>  2(b-1)(3b-1)/ | 3b(3b+1) -2 ==>
 6b^2-8b-2 | 9b^2+3b-1 ==>  6b^2-8b-2 | 3b^2 +11b+ 4
 ==> b <=5. Como b>a=3 ==> b=5 e c= 8, ferindo a paridade.
 Logo ab-1=c-1 ==> ab=c ==> (a-1)(b-1)(c-1) | c^2-1 ==>  (a-1)(b-1) |
 c+1 (a-1)(b-1) |ab+1==> (a-1)(b-1)!a+b
 a=2 ==> 2b-2= 2+b. b=4 e c=ab=8 (2,4,8)
 a=3 ==> 2(b-1) | 3+b ==> 2(b-1) = 3+. b=5 e c=ab=15. (3,5,15),
 Forcei um pouco a barra para mostrar que c=ab.
 Alguém teria uma outra solução, ou um endereço onde se tem as questões
 da IMO e suas resoluções?

 Grato!
 Saudações,
 PJMS



 --
 Esta mensagem foi verificada pelo sistema de antivírus e
 acredita-se estar livre de perigo.
>>>
>>>
>>> --
>>> Esta mensagem foi verificada pelo sistema de antivírus e
>>> acredita-se estar livre de perigo.
>>
>>

-- 
Esta mensagem foi verificada pelo sistema de antiv�rus e
 acredita-se estar livre de perigo.



Re: [obm-l] Problema da IMO

2020-09-11 Por tôpico Pedro José
Boa noite!
Fui em uma linha parecida com a primeira solução, embora não visse
necessidade de mudança de variáveis.
Mas o b achei sempre por restrição.
Esse "it implies" e aparece um número fatorado, não consegui captar, embora
tenha gostado do recurso, já que é bem restritivo.

Sudações,
PJMS


Em sáb., 12 de set. de 2020 às 00:08, Pedro José 
escreveu:

> Boa noite!
> Grato, Ralph!
>
> Estou estudando a solução. Pelo menos, não me decepcionei. A resposta
> estava correta,
>
> Saudações.
> PJMS
>
> Em sex., 11 de set. de 2020 às 22:33, Ralph Costa Teixeira <
> ralp...@gmail.com> escreveu:
>
>> Essa eh da IMO 1992. Tem uma solucao aqui:
>> http://sms.math.nus.edu.sg/Simo/IMO_Problems/92.pdf
>>
>> On Fri, Sep 11, 2020 at 10:06 PM Pedro José  wrote:
>>
>>> Bom dia!
>>>
>>> Recebi de um filho de um amigo, um problema que já o fizera.
>>> (a-1)(b-1)(c-1) | abc-1;  1>>
>>> Confesso que desta feita gastei mais tempo que da  primeira vez.
>>> Curioso, da primeira ,eu pensei, dessa vez, eu tentei lembrar como eu
>>> resolvera, aí nem lembrava, nem pensava. Apelei para a internet, mas não
>>> encontrei nada. Mas no fim, recordei o que havia feito.
>>> (1+1/(a-1))(1+1/(b-1))(1+1/(c-1)) = k, onde k é inteiro.
>>> vê-se que k>1, e para um dado a k é máximo para b e c mínimos logo b=a+1
>>> e c=a+2
>>> [a(a+1)(a+2)]/[(a-1)(a)(a+1)] > [a(a+1)(a+2)-1]/[(a-1)(a)(a+1)]>=2,
>>> então (a+2)/(a-1)>2 ==> a <4, a=2 ou a=3.
>>> O k é máximo para a=2, b=3 e c=4 ==> k <4, logo k=2 ou k=3.
>>> S.p.g, se a é ímpar (a-1)(b-1)(c-1) é par; então b,c ímpares e k é livre.
>>> S.p.g se a é par abc-1 é ímpar; então b,c são pares e k ímpar.
>>> a=2, temos  2b(b+1)/[(b-1)b] >3, não usei a restrição de paridade para c
>>> para facilitar a simplificação. b<5 Logo a=2>> é ímpar k=3. Logo c= 8. (2,4,8) é uma solução.
>>>
>>> a=3 temos 3b(b+1)/[2(b-1)b] > 2; b<7 e 3>> para a=3 e b=5. kmax <= (15*7-1)/(2*4*6) <=2;pois k é inteiro.
>>> 1 k=2 e c= 15. (3,5,15) é a outra solução.
>>>
>>> Só agora me apercebi de que c=ab nas duas soluções. Então tentei uma
>>> nova solução.
>>> (a-1)(b-1)(c-1) | abc-1 e (a-1)(b-1)(c-1) | abc + c(1 - (a+b)) -ab+
>>> (a+b) - 1 logo divide a diferença:
>>> (a-1)(b-1)(c-1) | (a+b) (c-1) + ab -1 - (c-1) logo c-1 | ab-1, então
>>> ab-1= w(c-1), para algum w inteiro e ab=w(c-1) +1 (i)
>>> Como a=2 ou a=3
>>> Se a=2. e w>=2
>>> Temos por (i) 2b>= 2 (c-1) +1 c-1>=b, logo absurdo.
>>> Se a=3
>>> Temos por (i) 3b>= w(c-1)+1; w=3 ==>3b< 3 (c-1) +1 pois c>b
>>> w=2 ==> 3b =2(c-1) +1 ==> c=(3b+1)/2
>>> 2(b-1)(3b-1)/2 | 3b(3b+1)/2 -1 ==>  2(b-1)(3b-1)/ | 3b(3b+1) -2 ==>
>>> 6b^2-8b-2 | 9b^2+3b-1 ==>  6b^2-8b-2 | 3b^2 +11b+ 4
>>> ==> b <=5. Como b>a=3 ==> b=5 e c= 8, ferindo a paridade.
>>> Logo ab-1=c-1 ==> ab=c ==> (a-1)(b-1)(c-1) | c^2-1 ==>  (a-1)(b-1) |
>>> c+1 (a-1)(b-1) |ab+1==> (a-1)(b-1)!a+b
>>> a=2 ==> 2b-2= 2+b. b=4 e c=ab=8 (2,4,8)
>>> a=3 ==> 2(b-1) | 3+b ==> 2(b-1) = 3+. b=5 e c=ab=15. (3,5,15),
>>> Forcei um pouco a barra para mostrar que c=ab.
>>> Alguém teria uma outra solução, ou um endereço onde se tem as questões
>>> da IMO e suas resoluções?
>>>
>>> Grato!
>>> Saudações,
>>> PJMS
>>>
>>>
>>>
>>> --
>>> Esta mensagem foi verificada pelo sistema de antivírus e
>>> acredita-se estar livre de perigo.
>>
>>
>> --
>> Esta mensagem foi verificada pelo sistema de antivírus e
>> acredita-se estar livre de perigo.
>
>

-- 
Esta mensagem foi verificada pelo sistema de antiv�rus e
 acredita-se estar livre de perigo.



Re: [obm-l] Problema da IMO

2020-09-11 Por tôpico Pedro José
Boa noite!
Grato, Ralph!

Estou estudando a solução. Pelo menos, não me decepcionei. A resposta
estava correta,

Saudações.
PJMS

Em sex., 11 de set. de 2020 às 22:33, Ralph Costa Teixeira <
ralp...@gmail.com> escreveu:

> Essa eh da IMO 1992. Tem uma solucao aqui:
> http://sms.math.nus.edu.sg/Simo/IMO_Problems/92.pdf
>
> On Fri, Sep 11, 2020 at 10:06 PM Pedro José  wrote:
>
>> Bom dia!
>>
>> Recebi de um filho de um amigo, um problema que já o fizera.
>> (a-1)(b-1)(c-1) | abc-1;  1>
>> Confesso que desta feita gastei mais tempo que da  primeira vez. Curioso,
>> da primeira ,eu pensei, dessa vez, eu tentei lembrar como eu resolvera, aí
>> nem lembrava, nem pensava. Apelei para a internet, mas não encontrei nada.
>> Mas no fim, recordei o que havia feito.
>> (1+1/(a-1))(1+1/(b-1))(1+1/(c-1)) = k, onde k é inteiro.
>> vê-se que k>1, e para um dado a k é máximo para b e c mínimos logo b=a+1
>> e c=a+2
>> [a(a+1)(a+2)]/[(a-1)(a)(a+1)] > [a(a+1)(a+2)-1]/[(a-1)(a)(a+1)]>=2, então
>> (a+2)/(a-1)>2 ==> a <4, a=2 ou a=3.
>> O k é máximo para a=2, b=3 e c=4 ==> k <4, logo k=2 ou k=3.
>> S.p.g, se a é ímpar (a-1)(b-1)(c-1) é par; então b,c ímpares e k é livre.
>> S.p.g se a é par abc-1 é ímpar; então b,c são pares e k ímpar.
>> a=2, temos  2b(b+1)/[(b-1)b] >3, não usei a restrição de paridade para c
>> para facilitar a simplificação. b<5 Logo a=2> é ímpar k=3. Logo c= 8. (2,4,8) é uma solução.
>>
>> a=3 temos 3b(b+1)/[2(b-1)b] > 2; b<7 e 3> para a=3 e b=5. kmax <= (15*7-1)/(2*4*6) <=2;pois k é inteiro.
>> 1 k=2 e c= 15. (3,5,15) é a outra solução.
>>
>> Só agora me apercebi de que c=ab nas duas soluções. Então tentei uma nova
>> solução.
>> (a-1)(b-1)(c-1) | abc-1 e (a-1)(b-1)(c-1) | abc + c(1 - (a+b)) -ab+ (a+b)
>> - 1 logo divide a diferença:
>> (a-1)(b-1)(c-1) | (a+b) (c-1) + ab -1 - (c-1) logo c-1 | ab-1, então
>> ab-1= w(c-1), para algum w inteiro e ab=w(c-1) +1 (i)
>> Como a=2 ou a=3
>> Se a=2. e w>=2
>> Temos por (i) 2b>= 2 (c-1) +1 c-1>=b, logo absurdo.
>> Se a=3
>> Temos por (i) 3b>= w(c-1)+1; w=3 ==>3b< 3 (c-1) +1 pois c>b
>> w=2 ==> 3b =2(c-1) +1 ==> c=(3b+1)/2
>> 2(b-1)(3b-1)/2 | 3b(3b+1)/2 -1 ==>  2(b-1)(3b-1)/ | 3b(3b+1) -2 ==>
>> 6b^2-8b-2 | 9b^2+3b-1 ==>  6b^2-8b-2 | 3b^2 +11b+ 4
>> ==> b <=5. Como b>a=3 ==> b=5 e c= 8, ferindo a paridade.
>> Logo ab-1=c-1 ==> ab=c ==> (a-1)(b-1)(c-1) | c^2-1 ==>  (a-1)(b-1) |  c+1
>> (a-1)(b-1) |ab+1==> (a-1)(b-1)!a+b
>> a=2 ==> 2b-2= 2+b. b=4 e c=ab=8 (2,4,8)
>> a=3 ==> 2(b-1) | 3+b ==> 2(b-1) = 3+. b=5 e c=ab=15. (3,5,15),
>> Forcei um pouco a barra para mostrar que c=ab.
>> Alguém teria uma outra solução, ou um endereço onde se tem as questões da
>> IMO e suas resoluções?
>>
>> Grato!
>> Saudações,
>> PJMS
>>
>>
>>
>> --
>> Esta mensagem foi verificada pelo sistema de antivírus e
>> acredita-se estar livre de perigo.
>
>
> --
> Esta mensagem foi verificada pelo sistema de antivírus e
> acredita-se estar livre de perigo.

-- 
Esta mensagem foi verificada pelo sistema de antiv�rus e
 acredita-se estar livre de perigo.



Re: [obm-l] Problema da IMO

2020-09-11 Por tôpico Ralph Costa Teixeira
Essa eh da IMO 1992. Tem uma solucao aqui:
http://sms.math.nus.edu.sg/Simo/IMO_Problems/92.pdf

On Fri, Sep 11, 2020 at 10:06 PM Pedro José  wrote:

> Bom dia!
>
> Recebi de um filho de um amigo, um problema que já o fizera.
> (a-1)(b-1)(c-1) | abc-1;  1
> Confesso que desta feita gastei mais tempo que da  primeira vez. Curioso,
> da primeira ,eu pensei, dessa vez, eu tentei lembrar como eu resolvera, aí
> nem lembrava, nem pensava. Apelei para a internet, mas não encontrei nada.
> Mas no fim, recordei o que havia feito.
> (1+1/(a-1))(1+1/(b-1))(1+1/(c-1)) = k, onde k é inteiro.
> vê-se que k>1, e para um dado a k é máximo para b e c mínimos logo b=a+1 e
> c=a+2
> [a(a+1)(a+2)]/[(a-1)(a)(a+1)] > [a(a+1)(a+2)-1]/[(a-1)(a)(a+1)]>=2, então
> (a+2)/(a-1)>2 ==> a <4, a=2 ou a=3.
> O k é máximo para a=2, b=3 e c=4 ==> k <4, logo k=2 ou k=3.
> S.p.g, se a é ímpar (a-1)(b-1)(c-1) é par; então b,c ímpares e k é livre.
> S.p.g se a é par abc-1 é ímpar; então b,c são pares e k ímpar.
> a=2, temos  2b(b+1)/[(b-1)b] >3, não usei a restrição de paridade para c
> para facilitar a simplificação. b<5 Logo a=2 é ímpar k=3. Logo c= 8. (2,4,8) é uma solução.
>
> a=3 temos 3b(b+1)/[2(b-1)b] > 2; b<7 e 3 para a=3 e b=5. kmax <= (15*7-1)/(2*4*6) <=2;pois k é inteiro.
> 1 k=2 e c= 15. (3,5,15) é a outra solução.
>
> Só agora me apercebi de que c=ab nas duas soluções. Então tentei uma nova
> solução.
> (a-1)(b-1)(c-1) | abc-1 e (a-1)(b-1)(c-1) | abc + c(1 - (a+b)) -ab+ (a+b)
> - 1 logo divide a diferença:
> (a-1)(b-1)(c-1) | (a+b) (c-1) + ab -1 - (c-1) logo c-1 | ab-1, então ab-1=
> w(c-1), para algum w inteiro e ab=w(c-1) +1 (i)
> Como a=2 ou a=3
> Se a=2. e w>=2
> Temos por (i) 2b>= 2 (c-1) +1 c-1>=b, logo absurdo.
> Se a=3
> Temos por (i) 3b>= w(c-1)+1; w=3 ==>3b< 3 (c-1) +1 pois c>b
> w=2 ==> 3b =2(c-1) +1 ==> c=(3b+1)/2
> 2(b-1)(3b-1)/2 | 3b(3b+1)/2 -1 ==>  2(b-1)(3b-1)/ | 3b(3b+1) -2 ==>
> 6b^2-8b-2 | 9b^2+3b-1 ==>  6b^2-8b-2 | 3b^2 +11b+ 4
> ==> b <=5. Como b>a=3 ==> b=5 e c= 8, ferindo a paridade.
> Logo ab-1=c-1 ==> ab=c ==> (a-1)(b-1)(c-1) | c^2-1 ==>  (a-1)(b-1) |  c+1
> (a-1)(b-1) |ab+1==> (a-1)(b-1)!a+b
> a=2 ==> 2b-2= 2+b. b=4 e c=ab=8 (2,4,8)
> a=3 ==> 2(b-1) | 3+b ==> 2(b-1) = 3+. b=5 e c=ab=15. (3,5,15),
> Forcei um pouco a barra para mostrar que c=ab.
> Alguém teria uma outra solução, ou um endereço onde se tem as questões da
> IMO e suas resoluções?
>
> Grato!
> Saudações,
> PJMS
>
>
>
> --
> Esta mensagem foi verificada pelo sistema de antivírus e
> acredita-se estar livre de perigo.

-- 
Esta mensagem foi verificada pelo sistema de antiv�rus e
 acredita-se estar livre de perigo.



Re: [obm-l] Problema simples gera um complicado?

2020-07-02 Por tôpico Professor Vanderlei Nemitz
Muito obrigado, Matheus!
Vou estudar sobre esse ponto especial!

Em qui., 2 de jul. de 2020 às 19:58, Matheus Bezerra <
matheusbezerr...@gmail.com> escreveu:

> Olá Vanderlei, boa noite. Esse é um fato conhecido, essas retas concorrem
> em um ponto chamado de Ponto de Fermat. Pesquisa por isso que você deve
> encontrar alguma prova. ;)
>
> *Matheus BL*
>
>
> Em qui., 2 de jul. de 2020 às 18:55, Professor Vanderlei Nemitz <
> vanderma...@gmail.com> escreveu:
>
>> Oi, pessoal, tudo bem?
>>
>> Resolvi um problema simples, que me fez pensar em outro, talvez
>> complicado. Bom, pelos menos são encontrei uma solução. Será que é verdade?
>> Se alguém puder ajudar a provar, caso seja, ficarei muito agradecido. Sem
>> querer "exigir" nada, afinal de contas eu não consegui, mas se puder ser
>> sem usar geometria analítica... :)
>>
>> Sobre os lados de um triângulo ABC são construídos externamente os
>> triângulos equiláteros BCD, CAE e ABF. Prove que os segmentos AD, BE e CF
>> são congruentes. (Esse é o problema simples!)
>>
>> Gostaria de provar que:
>>
>> *Sobre os lados de um triângulo ABC são construídos externamente os
>> triângulos equiláteros BCD, CAE e ABF. Prove que os segmentos AD, BE e CF
>> (ou suas retas suporte) concorrem em um mesmo ponto.*
>>
>> Muito obrigado!
>>
>> --
>> Esta mensagem foi verificada pelo sistema de antivírus e
>> acredita-se estar livre de perigo.
>
>
> --
> Esta mensagem foi verificada pelo sistema de antivírus e
> acredita-se estar livre de perigo.

-- 
Esta mensagem foi verificada pelo sistema de antiv�rus e
 acredita-se estar livre de perigo.



Re: [obm-l] Problema simples gera um complicado?

2020-07-02 Por tôpico Matheus Bezerra
Olá Vanderlei, boa noite. Esse é um fato conhecido, essas retas concorrem
em um ponto chamado de Ponto de Fermat. Pesquisa por isso que você deve
encontrar alguma prova. ;)

*Matheus BL*


Em qui., 2 de jul. de 2020 às 18:55, Professor Vanderlei Nemitz <
vanderma...@gmail.com> escreveu:

> Oi, pessoal, tudo bem?
>
> Resolvi um problema simples, que me fez pensar em outro, talvez
> complicado. Bom, pelos menos são encontrei uma solução. Será que é verdade?
> Se alguém puder ajudar a provar, caso seja, ficarei muito agradecido. Sem
> querer "exigir" nada, afinal de contas eu não consegui, mas se puder ser
> sem usar geometria analítica... :)
>
> Sobre os lados de um triângulo ABC são construídos externamente os
> triângulos equiláteros BCD, CAE e ABF. Prove que os segmentos AD, BE e CF
> são congruentes. (Esse é o problema simples!)
>
> Gostaria de provar que:
>
> *Sobre os lados de um triângulo ABC são construídos externamente os
> triângulos equiláteros BCD, CAE e ABF. Prove que os segmentos AD, BE e CF
> (ou suas retas suporte) concorrem em um mesmo ponto.*
>
> Muito obrigado!
>
> --
> Esta mensagem foi verificada pelo sistema de antivírus e
> acredita-se estar livre de perigo.

-- 
Esta mensagem foi verificada pelo sistema de antiv�rus e
 acredita-se estar livre de perigo.



Re: [obm-l] Problema de Geometria plana

2020-05-11 Por tôpico Projeto Iteano
De fato, se vc desenhar com régua e compasso dá pra ver q n é verdade

Em seg, 11 de mai de 2020 20:35, Vanderlei Nemitz 
escreveu:

> Boa noite!
> Vi esse problema em uma lista, mas talvez tenha alguma falha no enunciado.
> Ou será no leitor?
> Muito obrigado!
>
> *Seja ABC um triângulo e D um ponto sobre o lado AC tal que AB = CD. Sejam
> E e F os pontos médios de AD e BC, respectivamente. Se a reta BA intersecta
> a reta FE em M, prove que AM = ME*.
>
>
> --
> Esta mensagem foi verificada pelo sistema de antivírus e
> acredita-se estar livre de perigo.

-- 
Esta mensagem foi verificada pelo sistema de antiv�rus e
 acredita-se estar livre de perigo.



Re: [obm-l] Problema

2020-03-17 Por tôpico Pedro José
Boa noite!
Você já formulou esse problema em set/2019 e Daniel Jelin apresentou uma
bela solução.
Saudações,
PJMS

Em ter, 17 de mar de 2020 19:26,  escreveu:

> Problema
> Um mágico e seu assistente realizam um truque da maneira seguinte. Existem
> 12 caixas vazias e fechadas, colocadas em fila. O mágico sai da sala e uma
> pessoa do público escolhe duas caixas e esconde em cada uma delas uma
> moeda, deixando a fila de caixas da mesma forma como era, mas o assistente
> sabe quais são as duas caixas que têm as moedas. O mágico retorna para a
> sala e o assistente escolhe uma caixa que ele sabe que está vazia. Das
> restantes, o mágico então escolhe quatro caixas que são abertas
> simultaneamente. O objetivo do mágico é que, entre essas quatro caixas,
> duas contenham as moedas.
> Desenvolva um método que permita que o mágico e seu assistente realizem a
> mágica com sucesso
>
>
> --
> Esta mensagem foi verificada pelo sistema de antivírus e
> acredita-se estar livre de perigo.Booa
>

-- 
Esta mensagem foi verificada pelo sistema de antiv�rus e
 acredita-se estar livre de perigo.



Re: [obm-l] Problema 5 OBMU 2018

2019-10-31 Por tôpico Claudio Buffara
Mudando um pouco a notação...
Ponha: Df(x) = f(x+1) - f(x).

Para todo x em R+, e todo inteiro positivo k, existe (pelo TVM) y_k entre x
e x+1 tal que (Df)^(k)(x) = f^(k)(x+1) - f^(k)(x) = f'^(k+1)(y_k) > 0.
Logo, Df satisfaz a primeira condição do enunciado.
Além disso, como f' é positiva para todo x em R+, para todo m em R+, existe
(pelo TVM) a_m entre m e m+1 tal que Df(m) = f(m+1) - f(m) = f'(a_m) > 0
==> Df(m) é inteiro positivo.
Logo, Df satisfaz a condição 2 do enunciado.

Suponha que o resultado valha para Df (ou seja, que Df(n) >= 2^(n-1) para
todo inteiro positivo n).
Pela condição 2 do enunciado, f(1) >= 1.
Além disso, para todo n > 1, f(n) = f(1) + Df(1) + Df(2) + ... + Df(n-1) >=
1 + 2^0 + 2^1 + ... + 2^(n-2) = 1 + (2^(n-1) - 1) = 2^(n-1)
Portanto, se o resultado valer para Df, então valerá para f.

Pelo mesmo argumento, as funções D^2f, D^3f, ..., D^(n-1)f, dadas por:
D^2f(x) = Df(x+1) - Df(x),
D^3f(x) = D^2f(x+1) - D^2f(x)
...
D^(n-1)f(x) = D^(n-2)f(x+1) - D^(n-2)f(x)
também satisfazem as duas condições do enunciado.

Logo, se o resultado valer para D^2f, então valerá para Df (e, portanto,
pelo que foi feito acima, para f).
E assim por diante...
se valer para D^3f, então valerá para D^2f,
...
se valer para D^(n-1)f, então valerá para D^(n-2)f.

Agora, D^(n-1)f(1) = 1 (já que D^(n-1)f satisfaz a condição 2 do enunciado).
E assim...
D^(n-2)f(2) = D^(n-2)f(1) + D^(n-1)f(1)  >= 1 + 1 = 2
D^(n-3)f(3) = D^(n-3)f(1) + D^(n-2)f(1) + D^(n-2)f(2) >= 1 + 1 + 2 = 1 + 3
= 4
...
D^2f(n-2) = D^2f(1) + D^3f(1) + ... + D^3f(n-3) >= 1 + 1 + ... + 2^(n-4) =
1 + (2^(n-3) - 1) = 2^(n-3)
Df(n-1) = Df(1) + D^2f(1) + D^2f(2) + ... D^2f(n-2) >= 1 + 1 + 2 + ... +
2^(n-3) = 1 + (2^(n-2) - 1) = 2^(n-2)

Finalmente...
f(n) = f(1) + Df(1) + Df(2) + ... + Df(n-1) >= 1 + 1 + 2^1 + ... + 2^(n-2)
= 1 + (2^(n-1) - 1) = 2^(n-1)

E acabou!

[]s,
Claudio.



On Wed, Oct 30, 2019 at 5:38 PM Ernesto Rodrigues 
wrote:

> Pense um pouco sobre g(x)=f(x+1)-f(x), essa questão é bem tricky, o
> segredo é que a g satisfaz as condições da questão, logo, por indução, vale
> que g(n) é maior ou igual a dois elevado a n menos um, mas isto implica que
> o mesmo vale para f(n+1), completando a indução (tem que pensar bastante
> para sacar a ideia).
>
> Em qua, 30 de out de 2019 13:47, Lucas Dantas 
> escreveu:
>
>> Meu grupo da faculdade estamos com dificuldade de resolver o problema 5
>> da segunda fase da OBM-U 2018.
>>
>> Enunciado: Sejam R+ o conjunto dos números reais positivos e f:R+->R+ uma
>> função infinitamente diferenciável tal que:
>>
>> 1) Para todo k inteiro positivo e para todo real positivo x, f^(k)(x)>0 .
>> (Onde f^(k) representa a k-esima derivada).
>>
>> 2) para todo m inteiro positivo, f(m) é inteiro positivo.
>>
>> Prove que para todo inteiro positivo n, f(n)>= 2^(n-1)
>>
>>
>>
>> Estávamos pensando que qualquer função com todas as derivadas positivas
>> cresceria mais rápido que uma exponencial do tipo b^x com uma base maior
>> que um. Mas daí conseguimos o contraexemplo vish(sqrt(x)). Agora estamos
>> pensando em pensar em algo do tipo que f(x+1)>=2f(x). Mas mesmo assim não
>> sei como continuar. Se alguém tiver alguma sugestão de como resolver ou
>> alguma ideia pra nos ajudar.
>>
>> --
>> Esta mensagem foi verificada pelo sistema de antivírus e
>> acredita-se estar livre de perigo.
>
>
> --
> Esta mensagem foi verificada pelo sistema de antivírus e
> acredita-se estar livre de perigo.

-- 
Esta mensagem foi verificada pelo sistema de antiv�rus e
 acredita-se estar livre de perigo.



Re: [obm-l] Problema 5 OBMU 2018

2019-10-30 Por tôpico Ernesto Rodrigues
Pense um pouco sobre g(x)=f(x+1)-f(x), essa questão é bem tricky, o segredo
é que a g satisfaz as condições da questão, logo, por indução, vale que
g(n) é maior ou igual a dois elevado a n menos um, mas isto implica que o
mesmo vale para f(n+1), completando a indução (tem que pensar bastante para
sacar a ideia).

Em qua, 30 de out de 2019 13:47, Lucas Dantas 
escreveu:

> Meu grupo da faculdade estamos com dificuldade de resolver o problema 5 da
> segunda fase da OBM-U 2018.
>
> Enunciado: Sejam R+ o conjunto dos números reais positivos e f:R+->R+ uma
> função infinitamente diferenciável tal que:
>
> 1) Para todo k inteiro positivo e para todo real positivo x, f^(k)(x)>0 .
> (Onde f^(k) representa a k-esima derivada).
>
> 2) para todo m inteiro positivo, f(m) é inteiro positivo.
>
> Prove que para todo inteiro positivo n, f(n)>= 2^(n-1)
>
>
>
> Estávamos pensando que qualquer função com todas as derivadas positivas
> cresceria mais rápido que uma exponencial do tipo b^x com uma base maior
> que um. Mas daí conseguimos o contraexemplo vish(sqrt(x)). Agora estamos
> pensando em pensar em algo do tipo que f(x+1)>=2f(x). Mas mesmo assim não
> sei como continuar. Se alguém tiver alguma sugestão de como resolver ou
> alguma ideia pra nos ajudar.
>
> --
> Esta mensagem foi verificada pelo sistema de antivírus e
> acredita-se estar livre de perigo.

-- 
Esta mensagem foi verificada pelo sistema de antiv�rus e
 acredita-se estar livre de perigo.



Re: [obm-l] PROBLEMA

2019-09-04 Por tôpico Daniel Jelin
queremos fazer com que cada umas das 12 caixas indique um conjunto único de
outras 4 caixas (aquelas que o mágico irá abrir) de tal modo que o par de
caixas que contenham as moedas seja uma das 6 combinações dos 4 elementos,
2 a 2, desse conjunto. vamos imaginar as caixas numeradas de 1 a 12.

são 12 caixas. então serão 12 conjuntos, cada qual com 6 combinações
possíveis. então serão 72 combinações. como há 66 maneiras de escolher um
par de caixas entre 12 delas, é preciso que nossas 72 combinações tenham no
máximo 6 repetições - ou então não cobriremos todas os pares possíveis.

conseguiremos todas as combinações necessárias se cada conjunto de 4
números for montado de tal forma que as diferenças entre seus elementos
componham um sistema de restos módulo 12, o que pode ser feito, por
exemplo, tomando as caixas x + 1, x + 2, x + 4 e x + 8 (mod 12). as
diferenças entre esses elementos, importando a ordem, vão desde o 1 até o
11 (mod 12). conforme variamos o x, garantimos a variedade de combinações
(e as repetições antes mencionadas serão a dos pares de elementos mod 6).

ao assistente do mágico, caberá escolher a caixa x. ao mágico, caberá abrir
as caixas x + 1, x + 2, x + 4 e x + 8. assim, por exemplo, se o assistente
escolhe a caixa x = 3, essa é a senha para o mágico abrir as caixas 4, 5, 7
e 11. falta combinar o que é x.

sejam a e b os números das caixas que contêm as moedas. e seja b > a. o que
o assistente tem a fazer é verificar a diferença (b - a). seguindo nosso
sistema de restos, se (b-a) for 1, 3 ou 7, ele deve apontar a caixa x = a -
1 (mod 12) - lembrando que 1, 3 e 7 são justamente as diferenças (mod 12)
de [(x+2)-(x+1)], [(x+4)-(x+1)] e [(x+8)-(x+1)]. se (b - a) for 2, 6 ou 11,
o assistente deve apontar a caixa x = a - 2 (mod 12) - lembrando que 2, 6 e
11 são as diferenças (mod 12) de [(x+4)-(x+2)], [(x+8)-(x+2)] e
[(x+1)-(x+2)]). se (b - a) for 4, 9 ou 10, deve-se apontar a caixa x = a -
4 (mod 12) - lembrando que 4, 9 e 10 são justamente as diferenças
[(x+8)-(x+4)], [(x+1)-(x+4)] e [(x+2)-(x+4)]). e, finalmente, se (b - a)
for 5, 6 ou 8, aponta-se a caixa x = a - 8 (mod 12). - lembrando que 5, 6 e
8 são as diferenças [(x+1)-(x+8)], [(x+2)-(x+8)] e [(x+4)-(x+8)]). o mágico
completa o número abrindo x+1, x+2, x+4 e x+8 (mod 12).

abs

On Tue, Sep 3, 2019 at 7:06 PM  wrote:

> Problema
>
> Um mágico e seu assistente realizam uma mágica da maneira seguinte. Há 12
> caixas vazias e fechadas, colocadas em fila. O mágico sai da sala e uma
> pessoa do público escolhe duas caixas e esconde em cada uma delas uma
> moeda, deixando a fila de caixas da mesma forma como era, mas o assistente
> sabe quais são as duas caixas que têm moedas. O mágico retorna para a sala
> e o assistente escolhe uma caixa que ele sabe que está vazia. Das
> restantes, o mágico então escolhe quatro caixas que são abertas
> simultaneamente. O objetivo do mágico é que, entre essas quatro caixas,
> duas contenham as moedas.
>
> Desenvolva um método que permita que o mágico e seu assistente realizem a
> mágica com sucesso.
>
> --
> Esta mensagem foi verificada pelo sistema de antivírus e
> acredita-se estar livre de perigo.
>

-- 
Esta mensagem foi verificada pelo sistema de antiv�rus e
 acredita-se estar livre de perigo.



Re: [obm-l] Problema sobre Derivadas

2019-08-30 Por tôpico Luiz Antonio Rodrigues
Olá, Claudio!
Olá, Gabriel!
Muito obrigado pela ajuda!
Tudo ficou claro agora!
Abraços
Luiz

On Fri, Aug 30, 2019, 3:15 PM Claudio Buffara 
wrote:

> h'(x) = g'(f(x))*f'(x) ==> h'(3) = g'(f(3))*f'(3) = g'(5)*3 = 4*3 = 12.
>
> Imagino que a sua dificuldade esteja em como aplicar a regra da cadeia,
> que nos livros de cálculo é normalmente enunciada como:
> dy/dx = dy/du * du/dx (*)
> sem especificar quem são os argumentos (variáveis independentes) das
> funções e nem em que pontos as derivadas são calculadas.
>
> A fórmula que escrevi acima explicita isso: h(x) = (g o f)(x) = g(f(x))
> ==> h'(x) = g'(f(x))*f'(x).
> Assim, a derivada de h no ponto x = a, é dada por h'(a) = g'(f(a))*f'(a)
>
> Na fórmula (*) está implícito (mas deveria ser explicitado pelo autor do
> livro) que y é uma função (digamos, g) do argumento u, e u, por sua vez, é
> uma função (digamos, f) do argumento x.
> Logo, y = g(u)  e  u = f(x), o que implica que y = g(f(x)).
> Se queremos calcular dy/dx (a derivada de y em relação a x) no ponto x =
> a, de fato, queremos a derivada da função composta gof no ponto x = a.
>
> Ou seja, dy/dx(x=a) = (gof)'(a)
>
> Em (*), dy/du é a derivada de g em relação a u, calculada no ponto u =
> f(a) ==> dy/du(u=f(a)) = g'(f(a)).
> E du/dx é a derivada de f em relação a x, calculada no ponto x = a ==>
> du/dx(x=a) = f'(a).
>
> Assim, os livros de cálculo que apresentassem a fórmula (*) deveriam, no
> mínimo, escrevê-la como:
> dy/dx(x=a)  =  dy/du(u=u(a)) *  du/dx(x=a)
> Ou seja, explicitando em que ponto cada derivada é calculada.
>
> Espero que isso tenha ajudado.
>
> []s,
> Claudio.
>
>
> On Fri, Aug 30, 2019 at 2:16 PM Luiz Antonio Rodrigues <
> rodrigue...@gmail.com> wrote:
>
>> Olá, pessoal!
>> Boa tarde!
>> Tudo bem?
>> Estou confuso com o problema abaixo.
>> Alguém pode me ajudar?
>> Reconheço que tenho falhas graves em Cálculo e aproveito para pedir uma
>> indicação de material para estudar.
>> Muito obrigado!
>>
>> Temos duas funções f e g e sabemos que:
>>
>> f(3)=5
>> f'(3)=3
>>
>> f(4)=2
>> f'(4)= -3
>>
>> f(5)=1
>> f'(5)=7
>>
>> g(3)=4
>> g'(3)=5
>>
>> g(5)=3
>> g'(5)=4
>>
>> Se h(x)=g(f(x)), quanto vale h'(3)?
>>
>>
>> --
>> Esta mensagem foi verificada pelo sistema de antivírus e
>> acredita-se estar livre de perigo.
>
>
> --
> Esta mensagem foi verificada pelo sistema de antivírus e
> acredita-se estar livre de perigo.

-- 
Esta mensagem foi verificada pelo sistema de antiv�rus e
 acredita-se estar livre de perigo.



Re: [obm-l] Problema sobre Derivadas

2019-08-30 Por tôpico Claudio Buffara
h'(x) = g'(f(x))*f'(x) ==> h'(3) = g'(f(3))*f'(3) = g'(5)*3 = 4*3 = 12.

Imagino que a sua dificuldade esteja em como aplicar a regra da cadeia, que
nos livros de cálculo é normalmente enunciada como:
dy/dx = dy/du * du/dx (*)
sem especificar quem são os argumentos (variáveis independentes) das
funções e nem em que pontos as derivadas são calculadas.

A fórmula que escrevi acima explicita isso: h(x) = (g o f)(x) = g(f(x)) ==>
h'(x) = g'(f(x))*f'(x).
Assim, a derivada de h no ponto x = a, é dada por h'(a) = g'(f(a))*f'(a)

Na fórmula (*) está implícito (mas deveria ser explicitado pelo autor do
livro) que y é uma função (digamos, g) do argumento u, e u, por sua vez, é
uma função (digamos, f) do argumento x.
Logo, y = g(u)  e  u = f(x), o que implica que y = g(f(x)).
Se queremos calcular dy/dx (a derivada de y em relação a x) no ponto x = a,
de fato, queremos a derivada da função composta gof no ponto x = a.

Ou seja, dy/dx(x=a) = (gof)'(a)

Em (*), dy/du é a derivada de g em relação a u, calculada no ponto u = f(a)
==> dy/du(u=f(a)) = g'(f(a)).
E du/dx é a derivada de f em relação a x, calculada no ponto x = a ==>
du/dx(x=a) = f'(a).

Assim, os livros de cálculo que apresentassem a fórmula (*) deveriam, no
mínimo, escrevê-la como:
dy/dx(x=a)  =  dy/du(u=u(a)) *  du/dx(x=a)
Ou seja, explicitando em que ponto cada derivada é calculada.

Espero que isso tenha ajudado.

[]s,
Claudio.


On Fri, Aug 30, 2019 at 2:16 PM Luiz Antonio Rodrigues <
rodrigue...@gmail.com> wrote:

> Olá, pessoal!
> Boa tarde!
> Tudo bem?
> Estou confuso com o problema abaixo.
> Alguém pode me ajudar?
> Reconheço que tenho falhas graves em Cálculo e aproveito para pedir uma
> indicação de material para estudar.
> Muito obrigado!
>
> Temos duas funções f e g e sabemos que:
>
> f(3)=5
> f'(3)=3
>
> f(4)=2
> f'(4)= -3
>
> f(5)=1
> f'(5)=7
>
> g(3)=4
> g'(3)=5
>
> g(5)=3
> g'(5)=4
>
> Se h(x)=g(f(x)), quanto vale h'(3)?
>
>
> --
> Esta mensagem foi verificada pelo sistema de antivírus e
> acredita-se estar livre de perigo.

-- 
Esta mensagem foi verificada pelo sistema de antiv�rus e
 acredita-se estar livre de perigo.



Re: [obm-l] Problema sobre Derivadas

2019-08-30 Por tôpico Gabriel Lopes
Ola, boa tarde. Isso é uma simples aplicação da regra da cadeia.

H'(x) = g'(f (x))*f'(x)

H'(3) = g'(f (3))*f'(3) = g (5) * 3 = 9


Em Sex, 30 de ago de 2019 14:16, Luiz Antonio Rodrigues <
rodrigue...@gmail.com> escreveu:

> Olá, pessoal!
> Boa tarde!
> Tudo bem?
> Estou confuso com o problema abaixo.
> Alguém pode me ajudar?
> Reconheço que tenho falhas graves em Cálculo e aproveito para pedir uma
> indicação de material para estudar.
> Muito obrigado!
>
> Temos duas funções f e g e sabemos que:
>
> f(3)=5
> f'(3)=3
>
> f(4)=2
> f'(4)= -3
>
> f(5)=1
> f'(5)=7
>
> g(3)=4
> g'(3)=5
>
> g(5)=3
> g'(5)=4
>
> Se h(x)=g(f(x)), quanto vale h'(3)?
>
>
> --
> Esta mensagem foi verificada pelo sistema de antivírus e
> acredita-se estar livre de perigo.

-- 
Esta mensagem foi verificada pelo sistema de antiv�rus e
 acredita-se estar livre de perigo.



[obm-l] Re: [obm-l] Problema da Olimpíada Brasileira de Matemática para Universitários

2019-07-12 Por tôpico Pedro Angelo
Pensando rápido aqui. Dados discos D1 e D2, queremos pontos P1 e P2
tais que toda parábola que passa por P1 e P2 toca pelo menos um dos
discos. (Estou assumindo que P1 e P2 estão proibidos de pertencerem
aos discos, pois caso contrário bastaria escolher Pj em Dj.)
Obviamente, P1 e P2 devem estar próximos dos discos (cada um próximo
de um disco). Pensando rápido acho que as seguintes duas condições são
necessárias:

(I) A reta 'r' que liga P1 a P2 deve cruzar os discos
(II) A reta perpendicular a 'r' por Pj deve cruzar Dj

Acho que para tornar essas condições suficientes além de necessárias,
basta adicionar as seguintes condições extras:

(III) Num sistema de coordenadas em que o centro de D1 está na origem
e o centro de D2 está no eixo x positivo, P1 está no terceiro
quadrante (por exemplo, com um argumento de 5pi/4)
(IV) Idem, trocando D1 por D2 e P1 por P2

Posso estar enganado, mas o meu chute é esse. Dada a condição (II), a
condição (III) garante que todas as parábolas com concavidade para um
dos lados de 'r' cruza D1, enquanto a condição (IV) garante que as
parábolas para o outro lado de 'r' cruzam D2.

Le ven. 12 juil. 2019 à 00:15, João Maldonado
 a écrit :
>
> Galera, esse é uma problema da OBM mas não me lembro de qual ano. Eu tentei 
> uma solução e acabei de descobrir que tinha uma falha, não é possível 
> escolher d tal que y' imagem)
>
> O problema é o seguinte:
> Dados dois discos em um plano, prove que sempre é possível escolher dois 
> pontos tais que qualquer parábola que passe por ambos esses pontos sempre 
> passará por pelo menos um dos discos.
>
> Eu tentei uma solução, mas como eu disse está errada. Alguém consegue propor 
> alguma solução? É possível ainda aproveitar o minha linha de raciocínio ou 
> está completamente errado? A solução está em inglês pq postei em um fórum 
> americano rs.
>
>
> --
> Esta mensagem foi verificada pelo sistema de antivírus e
> acredita-se estar livre de perigo.

-- 
Esta mensagem foi verificada pelo sistema de antiv�rus e
 acredita-se estar livre de perigo.


=
Instru��es para entrar na lista, sair da lista e usar a lista em
http://www.mat.puc-rio.br/~obmlistas/obm-l.html
=


[obm-l] Re: [obm-l] Problema de Matemática Financeira

2019-04-01 Por tôpico Luiz Antonio Rodrigues
Olá, Pedro!
Seguirei seu conselho: vou conversar com alguém que entenda bastante do
assunto.
Muito obrigado e um abraço!
Luiz

On Sun, Mar 31, 2019, 7:23 PM Pedro José  wrote:

> Boa noite!
>
> Mas tem de verificar se é praxe fazer assim ou não. Nos juros compostos,
> você pode trabalhar com qualquer referência no tempo e depois levar para
> uma mesma que dá a mesma coisa.
>
> Juro simples não. Ma ninguém trabalha com juro simples. Tem que ver uma
> pessoa que entenda de financeira. Pois, fica difícil adivinhar qual o
> método a utilizar.
>
> Em dom, 31 de mar de 2019 às 18:29, Luiz Antonio Rodrigues <
> rodrigue...@gmail.com> escreveu:
>
>> Olá, Pedro!
>> Tudo bem?
>> Concordo com suas observações.
>> Eu havia chegado no valor calculado no item (1).
>> Mas eu entendi os cálculos dos itens (2) e (3).
>> Agora sim eu percebi qual deve ser o raciocínio para resolver o problema!
>> Muito obrigado pela ajuda!
>> Um abraço!
>> Luiz
>>
>> On Sun, Mar 31, 2019, 1:04 PM Pedro José  wrote:
>>
>>> Bom dia!
>>>
>>> Primeiramente, nenhuma instituição empresta a juros simples. Segundo,
>>> nenhuma instituição permite que o pagamento fique a vontade do cliente. Há
>>> mora para esse caso.
>>> Não consigo entender a natureza desses problemas.
>>> Não entendo muito de matemática financeira. Mas o cálculo  à taxa de
>>> juros simples não se conserva no tempo de referência.
>>> Vamos fazer três fluxos de Caixa:
>>>
>>> 1) Se tomei 50.000 de empréstimo e paguei 5.000, só peguei 45.000 de
>>> empréstimo. O resto é só para testar se o solicitante do empréstimo tem
>>> algum "oxigênio"
>>> dívida inicial de 45.000
>>> 9 meses depois a dívida seria de: 45.000 * 1,27 = 57.150,00
>>> Pago 9.000 minha dívida passou a ser de 48.150,00
>>> Daí a 6meses (15 do pedido de empréstimo) minha dívida seria de
>>> 48.150,00 * 1,18 = 56.817,00
>>> Pago 15.000, passo a dever 41,817,00
>>> Daí a 9 meses passaria a dever 41.817*1,27= 53.107,57
>>>
>>> 2) Vamos levar tudo para 2 anos agora.
>>>
>>> 45.000,00*1,72=77.4000,00 (dívida)
>>> 9.000,00 * 1,45 = 13.050 (amortização)
>>> 15.000,00*1,27 = 19.050 (amortização)
>>> Dívida restante = 45.300,00
>>>
>>> 2) Vamos trazer tudo para o valor presente:
>>>
>>> 45.000,00 fica
>>> 9.000,00/1,27  = 7.086,614
>>> 15.000,00/1,45 = 10.344,883
>>> Divída no dia do empréstimo: 27.568,56
>>> Dívida após dois anos: 27.568.56*1,72= 47.417,92
>>>
>>> Não sei se há alguma convenção para fazer o fluxo de caixa nesse sistema
>>> de juros, que não existe.
>>>
>>> Talvez alguém que entenda de financeira possa ajudá-lo.
>>>
>>> Saudações,
>>> PJMS
>>>
>>>
>>> Em sex, 29 de mar de 2019 às 22:19, Luiz Antonio Rodrigues <
>>> rodrigue...@gmail.com> escreveu:
>>>
 Olá, pessoal!
 Tudo bem?
 Resolvi o problema abaixo e não consigo chegar na resposta.
 Alguém pode me ajudar?
 Muito obrigado e um abraço!
 Luiz

 A empresa SoDevo S.A comprou um equipamento cujo valor a vista era R$
 50.000,00. A empresa pagou 10% de entrada e concordou em financiar o
 restante a uma taxa de juros simples de 3% a.m. Se a empresa pagar ao banco
 R$ 9.000,00 nove meses após a compra e R$ 15.000,00 quinze meses após a
 compra, quanto precisará pagar para liquidar o financiamento dois anos
 depois da compra? Resposta: R$ 47.417,92

 --
 Esta mensagem foi verificada pelo sistema de antivírus e
 acredita-se estar livre de perigo.
>>>
>>>
>>> --
>>> Esta mensagem foi verificada pelo sistema de antivírus e
>>> acredita-se estar livre de perigo.
>>
>>
>> --
>> Esta mensagem foi verificada pelo sistema de antivírus e
>> acredita-se estar livre de perigo.
>
>
> --
> Esta mensagem foi verificada pelo sistema de antivírus e
> acredita-se estar livre de perigo.

-- 
Esta mensagem foi verificada pelo sistema de antiv�rus e
 acredita-se estar livre de perigo.



[obm-l] Re: [obm-l] Re: [obm-l] Re: [obm-l] Problema de Matemática Financeira

2019-03-31 Por tôpico Pedro José
Boa noite!

Mas tem de verificar se é praxe fazer assim ou não. Nos juros compostos,
você pode trabalhar com qualquer referência no tempo e depois levar para
uma mesma que dá a mesma coisa.

Juro simples não. Ma ninguém trabalha com juro simples. Tem que ver uma
pessoa que entenda de financeira. Pois, fica difícil adivinhar qual o
método a utilizar.

Em dom, 31 de mar de 2019 às 18:29, Luiz Antonio Rodrigues <
rodrigue...@gmail.com> escreveu:

> Olá, Pedro!
> Tudo bem?
> Concordo com suas observações.
> Eu havia chegado no valor calculado no item (1).
> Mas eu entendi os cálculos dos itens (2) e (3).
> Agora sim eu percebi qual deve ser o raciocínio para resolver o problema!
> Muito obrigado pela ajuda!
> Um abraço!
> Luiz
>
> On Sun, Mar 31, 2019, 1:04 PM Pedro José  wrote:
>
>> Bom dia!
>>
>> Primeiramente, nenhuma instituição empresta a juros simples. Segundo,
>> nenhuma instituição permite que o pagamento fique a vontade do cliente. Há
>> mora para esse caso.
>> Não consigo entender a natureza desses problemas.
>> Não entendo muito de matemática financeira. Mas o cálculo  à taxa de
>> juros simples não se conserva no tempo de referência.
>> Vamos fazer três fluxos de Caixa:
>>
>> 1) Se tomei 50.000 de empréstimo e paguei 5.000, só peguei 45.000 de
>> empréstimo. O resto é só para testar se o solicitante do empréstimo tem
>> algum "oxigênio"
>> dívida inicial de 45.000
>> 9 meses depois a dívida seria de: 45.000 * 1,27 = 57.150,00
>> Pago 9.000 minha dívida passou a ser de 48.150,00
>> Daí a 6meses (15 do pedido de empréstimo) minha dívida seria de 48.150,00
>> * 1,18 = 56.817,00
>> Pago 15.000, passo a dever 41,817,00
>> Daí a 9 meses passaria a dever 41.817*1,27= 53.107,57
>>
>> 2) Vamos levar tudo para 2 anos agora.
>>
>> 45.000,00*1,72=77.4000,00 (dívida)
>> 9.000,00 * 1,45 = 13.050 (amortização)
>> 15.000,00*1,27 = 19.050 (amortização)
>> Dívida restante = 45.300,00
>>
>> 2) Vamos trazer tudo para o valor presente:
>>
>> 45.000,00 fica
>> 9.000,00/1,27  = 7.086,614
>> 15.000,00/1,45 = 10.344,883
>> Divída no dia do empréstimo: 27.568,56
>> Dívida após dois anos: 27.568.56*1,72= 47.417,92
>>
>> Não sei se há alguma convenção para fazer o fluxo de caixa nesse sistema
>> de juros, que não existe.
>>
>> Talvez alguém que entenda de financeira possa ajudá-lo.
>>
>> Saudações,
>> PJMS
>>
>>
>> Em sex, 29 de mar de 2019 às 22:19, Luiz Antonio Rodrigues <
>> rodrigue...@gmail.com> escreveu:
>>
>>> Olá, pessoal!
>>> Tudo bem?
>>> Resolvi o problema abaixo e não consigo chegar na resposta.
>>> Alguém pode me ajudar?
>>> Muito obrigado e um abraço!
>>> Luiz
>>>
>>> A empresa SoDevo S.A comprou um equipamento cujo valor a vista era R$
>>> 50.000,00. A empresa pagou 10% de entrada e concordou em financiar o
>>> restante a uma taxa de juros simples de 3% a.m. Se a empresa pagar ao banco
>>> R$ 9.000,00 nove meses após a compra e R$ 15.000,00 quinze meses após a
>>> compra, quanto precisará pagar para liquidar o financiamento dois anos
>>> depois da compra? Resposta: R$ 47.417,92
>>>
>>> --
>>> Esta mensagem foi verificada pelo sistema de antivírus e
>>> acredita-se estar livre de perigo.
>>
>>
>> --
>> Esta mensagem foi verificada pelo sistema de antivírus e
>> acredita-se estar livre de perigo.
>
>
> --
> Esta mensagem foi verificada pelo sistema de antivírus e
> acredita-se estar livre de perigo.

-- 
Esta mensagem foi verificada pelo sistema de antiv�rus e
 acredita-se estar livre de perigo.



[obm-l] Re: [obm-l] Re: [obm-l] Problema de Matemática Financeira

2019-03-31 Por tôpico Luiz Antonio Rodrigues
Olá, Pedro!
Tudo bem?
Concordo com suas observações.
Eu havia chegado no valor calculado no item (1).
Mas eu entendi os cálculos dos itens (2) e (3).
Agora sim eu percebi qual deve ser o raciocínio para resolver o problema!
Muito obrigado pela ajuda!
Um abraço!
Luiz

On Sun, Mar 31, 2019, 1:04 PM Pedro José  wrote:

> Bom dia!
>
> Primeiramente, nenhuma instituição empresta a juros simples. Segundo,
> nenhuma instituição permite que o pagamento fique a vontade do cliente. Há
> mora para esse caso.
> Não consigo entender a natureza desses problemas.
> Não entendo muito de matemática financeira. Mas o cálculo  à taxa de juros
> simples não se conserva no tempo de referência.
> Vamos fazer três fluxos de Caixa:
>
> 1) Se tomei 50.000 de empréstimo e paguei 5.000, só peguei 45.000 de
> empréstimo. O resto é só para testar se o solicitante do empréstimo tem
> algum "oxigênio"
> dívida inicial de 45.000
> 9 meses depois a dívida seria de: 45.000 * 1,27 = 57.150,00
> Pago 9.000 minha dívida passou a ser de 48.150,00
> Daí a 6meses (15 do pedido de empréstimo) minha dívida seria de 48.150,00
> * 1,18 = 56.817,00
> Pago 15.000, passo a dever 41,817,00
> Daí a 9 meses passaria a dever 41.817*1,27= 53.107,57
>
> 2) Vamos levar tudo para 2 anos agora.
>
> 45.000,00*1,72=77.4000,00 (dívida)
> 9.000,00 * 1,45 = 13.050 (amortização)
> 15.000,00*1,27 = 19.050 (amortização)
> Dívida restante = 45.300,00
>
> 2) Vamos trazer tudo para o valor presente:
>
> 45.000,00 fica
> 9.000,00/1,27  = 7.086,614
> 15.000,00/1,45 = 10.344,883
> Divída no dia do empréstimo: 27.568,56
> Dívida após dois anos: 27.568.56*1,72= 47.417,92
>
> Não sei se há alguma convenção para fazer o fluxo de caixa nesse sistema
> de juros, que não existe.
>
> Talvez alguém que entenda de financeira possa ajudá-lo.
>
> Saudações,
> PJMS
>
>
> Em sex, 29 de mar de 2019 às 22:19, Luiz Antonio Rodrigues <
> rodrigue...@gmail.com> escreveu:
>
>> Olá, pessoal!
>> Tudo bem?
>> Resolvi o problema abaixo e não consigo chegar na resposta.
>> Alguém pode me ajudar?
>> Muito obrigado e um abraço!
>> Luiz
>>
>> A empresa SoDevo S.A comprou um equipamento cujo valor a vista era R$
>> 50.000,00. A empresa pagou 10% de entrada e concordou em financiar o
>> restante a uma taxa de juros simples de 3% a.m. Se a empresa pagar ao banco
>> R$ 9.000,00 nove meses após a compra e R$ 15.000,00 quinze meses após a
>> compra, quanto precisará pagar para liquidar o financiamento dois anos
>> depois da compra? Resposta: R$ 47.417,92
>>
>> --
>> Esta mensagem foi verificada pelo sistema de antivírus e
>> acredita-se estar livre de perigo.
>
>
> --
> Esta mensagem foi verificada pelo sistema de antivírus e
> acredita-se estar livre de perigo.

-- 
Esta mensagem foi verificada pelo sistema de antiv�rus e
 acredita-se estar livre de perigo.



[obm-l] Re: [obm-l] Problema de Matemática Financeira

2019-03-31 Por tôpico Pedro José
Bom dia!

Primeiramente, nenhuma instituição empresta a juros simples. Segundo,
nenhuma instituição permite que o pagamento fique a vontade do cliente. Há
mora para esse caso.
Não consigo entender a natureza desses problemas.
Não entendo muito de matemática financeira. Mas o cálculo  à taxa de juros
simples não se conserva no tempo de referência.
Vamos fazer três fluxos de Caixa:

1) Se tomei 50.000 de empréstimo e paguei 5.000, só peguei 45.000 de
empréstimo. O resto é só para testar se o solicitante do empréstimo tem
algum "oxigênio"
dívida inicial de 45.000
9 meses depois a dívida seria de: 45.000 * 1,27 = 57.150,00
Pago 9.000 minha dívida passou a ser de 48.150,00
Daí a 6meses (15 do pedido de empréstimo) minha dívida seria de 48.150,00 *
1,18 = 56.817,00
Pago 15.000, passo a dever 41,817,00
Daí a 9 meses passaria a dever 41.817*1,27= 53.107,57

2) Vamos levar tudo para 2 anos agora.

45.000,00*1,72=77.4000,00 (dívida)
9.000,00 * 1,45 = 13.050 (amortização)
15.000,00*1,27 = 19.050 (amortização)
Dívida restante = 45.300,00

2) Vamos trazer tudo para o valor presente:

45.000,00 fica
9.000,00/1,27  = 7.086,614
15.000,00/1,45 = 10.344,883
Divída no dia do empréstimo: 27.568,56
Dívida após dois anos: 27.568.56*1,72= 47.417,92

Não sei se há alguma convenção para fazer o fluxo de caixa nesse sistema de
juros, que não existe.

Talvez alguém que entenda de financeira possa ajudá-lo.

Saudações,
PJMS


Em sex, 29 de mar de 2019 às 22:19, Luiz Antonio Rodrigues <
rodrigue...@gmail.com> escreveu:

> Olá, pessoal!
> Tudo bem?
> Resolvi o problema abaixo e não consigo chegar na resposta.
> Alguém pode me ajudar?
> Muito obrigado e um abraço!
> Luiz
>
> A empresa SoDevo S.A comprou um equipamento cujo valor a vista era R$
> 50.000,00. A empresa pagou 10% de entrada e concordou em financiar o
> restante a uma taxa de juros simples de 3% a.m. Se a empresa pagar ao banco
> R$ 9.000,00 nove meses após a compra e R$ 15.000,00 quinze meses após a
> compra, quanto precisará pagar para liquidar o financiamento dois anos
> depois da compra? Resposta: R$ 47.417,92
>
> --
> Esta mensagem foi verificada pelo sistema de antivírus e
> acredita-se estar livre de perigo.

-- 
Esta mensagem foi verificada pelo sistema de antiv�rus e
 acredita-se estar livre de perigo.



[obm-l] Re: [obm-l] Re: [obm-l] [Problema da Balança]

2019-03-26 Por tôpico gilberto azevedo
Tentei muito assim, não saiu.
Gabarito consta n - 1 mesmo.

Em ter, 26 de mar de 2019 22:47, Gabriel Lopes 
escreveu:

> Para mim o numero de pesagem mínimal é  n-1, para n maior ou igual a 3,
> para se obter tanto o maximo quanto o minimo,( faça indução) .Para obter o
> maximo e  depois o mínimo separe o o menor na primeira pesagem e prossiga
> para obter o maximo n-1 mais n-2 pesagens, acho q é isso
>
> Em Ter, 26 de mar de 2019 21:14, gilberto azevedo 
> escreveu:
>
>> DAdos n ( n maior ou igual do que 2 ) objetos de pesos distintos, prove
>> que é possivel determinar qual o mais pesado fazendo 2n - 3 pesagens em uma
>> balança de pratos. É esse número mínimo de pesagens que permitem determinar
>> o mais leve e o mais pesado ?
>>
>> --
>> Esta mensagem foi verificada pelo sistema de antivírus e
>> acredita-se estar livre de perigo.
>
>
> --
> Esta mensagem foi verificada pelo sistema de antivírus e
> acredita-se estar livre de perigo.

-- 
Esta mensagem foi verificada pelo sistema de antiv�rus e
 acredita-se estar livre de perigo.



[obm-l] Re: [obm-l] [Problema da Balança]

2019-03-26 Por tôpico Gabriel Lopes
Para mim o numero de pesagem mínimal é  n-1, para n maior ou igual a 3,
para se obter tanto o maximo quanto o minimo,( faça indução) .Para obter o
maximo e  depois o mínimo separe o o menor na primeira pesagem e prossiga
para obter o maximo n-1 mais n-2 pesagens, acho q é isso

Em Ter, 26 de mar de 2019 21:14, gilberto azevedo 
escreveu:

> DAdos n ( n maior ou igual do que 2 ) objetos de pesos distintos, prove
> que é possivel determinar qual o mais pesado fazendo 2n - 3 pesagens em uma
> balança de pratos. É esse número mínimo de pesagens que permitem determinar
> o mais leve e o mais pesado ?
>
> --
> Esta mensagem foi verificada pelo sistema de antivírus e
> acredita-se estar livre de perigo.

-- 
Esta mensagem foi verificada pelo sistema de antiv�rus e
 acredita-se estar livre de perigo.



[obm-l] Re: [obm-l] Re: [obm-l] Re: [obm-l] Problema olimpíada de maio

2019-01-23 Por tôpico Ralph Teixeira
Sim, nao vi porque que algum resto apareceria mais do que os outros...
Achei que eu conseguiria uma funcao que levasse cada classe de restos numa
outra, mas soh consegui pareamentos. Com os dois paremntos, deu.

On Wed, Jan 23, 2019 at 10:27 AM Mauricio de Araujo <
mauricio.de.ara...@gmail.com> wrote:

> Bela solução!! mas qual foi o teu insight? Desconfiança de que havia uma
> distribuição uniforme dos restos possíveis?
> Att.
>
> Em qua, 23 de jan de 2019 às 00:47, Ralph Teixeira 
> escreveu:
>
>> Hm, tive uma ideia, confiram se funciona.
>>
>> Seja S o conjunto dos numeros obtidos pela permutacao dos digitos de 1 a
>> 7, e seja x_i a quantidade de elementos de S que deixam resto i na divisao
>> por 7 (i=0,1,2,3,4,5,6).
>>
>> Agora vamos fazer dois pareamentos. (Ou seja, vamos criar funcoes
>> f,g:S->S tal que f(f(N))=N e g(g(N))=N para todo N em S).
>>
>> PRIMEIRO: No primeiro pareamento, troque cada digito x de N=abcdefg pelo
>> digito 8-x obtendo o numero f(N) (por exemplo, o numero N=1574326 eh
>> pareado com f(N)=7314562). Claramente, f(N) esta em S, e f(f(N))=N. Como a
>> soma desses dois numeros eh N+f(N)=888, que deixa resto 1 na divisao
>> por 7, temos automaticamente que:
>> -- Se N mod 7 = 1, entao f(N) mod 7 = 0; em outras palavras para cada
>> numero N que deixa resto 1, temos exatamente um numero f(N) que deixa resto
>> 0, e vice-versa; portanto x_1=x_0.
>> -- Se N mod 7 = 2, entao f(N) mod 7 = 6; portanto x_2=x_6.
>> -- Analogamente, x_3=x_5.
>>
>> SEGUNDO: Agora, g(N) eh obtido a partir de N trocando cada digito x por
>> 7-x, EXCETO O DIGITO 7 que eh mantido. Por exemplo, se N=1754326 entao
>> g(N)=6723451. Claramente g(N) estah em S, e g(g(N))=N. Agora, N+g(N) mod 7
>> = 777 = 0 (pois aquele 7 extra pode ser jogado fora sem alterar o resto
>> modulo 7). Assim, de maneira analoga ao pareamento anterior, concluimos que:
>> -- x_1=x_6; x_2=x_5; x_3=x_4.
>>
>> Encadeando tudo, concluimos que x_0=x_1=x_2=...=x_6. Assim, o numero
>> pedido eh x_0=#(S)/7=6!.
>>
>> Abraco, Ralph.
>>
>> On Tue, Jan 22, 2019 at 9:57 PM Heitor Gama Ribeiro <
>> heitor...@hotmail.com> wrote:
>>
>>> Consideramos todos os números de 7 dígitos que se obtém permutando de
>>> todas as maneiras possíveis os dígitos de 1234567. Quantos deles são
>>> divisíveis por 7?
>>>
>>> --
>>> Esta mensagem foi verificada pelo sistema de antivírus e
>>> acredita-se estar livre de perigo.
>>>
>>
>> --
>> Esta mensagem foi verificada pelo sistema de antivírus e
>> acredita-se estar livre de perigo.
>
>
>
> --
> --
> Abraços,
> Mauricio de Araujo
> [oɾnɐɹɐ ǝp oıɔıɹnɐɯ]
>
> --
> Esta mensagem foi verificada pelo sistema de antivírus e
> acredita-se estar livre de perigo.

-- 
Esta mensagem foi verificada pelo sistema de antiv�rus e
 acredita-se estar livre de perigo.



[obm-l] Re: [obm-l] Re: [obm-l] Problema olimpíada de maio

2019-01-23 Por tôpico Mauricio de Araujo
Bela solução!! mas qual foi o teu insight? Desconfiança de que havia uma
distribuição uniforme dos restos possíveis?
Att.

Em qua, 23 de jan de 2019 às 00:47, Ralph Teixeira 
escreveu:

> Hm, tive uma ideia, confiram se funciona.
>
> Seja S o conjunto dos numeros obtidos pela permutacao dos digitos de 1 a
> 7, e seja x_i a quantidade de elementos de S que deixam resto i na divisao
> por 7 (i=0,1,2,3,4,5,6).
>
> Agora vamos fazer dois pareamentos. (Ou seja, vamos criar funcoes f,g:S->S
> tal que f(f(N))=N e g(g(N))=N para todo N em S).
>
> PRIMEIRO: No primeiro pareamento, troque cada digito x de N=abcdefg pelo
> digito 8-x obtendo o numero f(N) (por exemplo, o numero N=1574326 eh
> pareado com f(N)=7314562). Claramente, f(N) esta em S, e f(f(N))=N. Como a
> soma desses dois numeros eh N+f(N)=888, que deixa resto 1 na divisao
> por 7, temos automaticamente que:
> -- Se N mod 7 = 1, entao f(N) mod 7 = 0; em outras palavras para cada
> numero N que deixa resto 1, temos exatamente um numero f(N) que deixa resto
> 0, e vice-versa; portanto x_1=x_0.
> -- Se N mod 7 = 2, entao f(N) mod 7 = 6; portanto x_2=x_6.
> -- Analogamente, x_3=x_5.
>
> SEGUNDO: Agora, g(N) eh obtido a partir de N trocando cada digito x por
> 7-x, EXCETO O DIGITO 7 que eh mantido. Por exemplo, se N=1754326 entao
> g(N)=6723451. Claramente g(N) estah em S, e g(g(N))=N. Agora, N+g(N) mod 7
> = 777 = 0 (pois aquele 7 extra pode ser jogado fora sem alterar o resto
> modulo 7). Assim, de maneira analoga ao pareamento anterior, concluimos que:
> -- x_1=x_6; x_2=x_5; x_3=x_4.
>
> Encadeando tudo, concluimos que x_0=x_1=x_2=...=x_6. Assim, o numero
> pedido eh x_0=#(S)/7=6!.
>
> Abraco, Ralph.
>
> On Tue, Jan 22, 2019 at 9:57 PM Heitor Gama Ribeiro 
> wrote:
>
>> Consideramos todos os números de 7 dígitos que se obtém permutando de
>> todas as maneiras possíveis os dígitos de 1234567. Quantos deles são
>> divisíveis por 7?
>>
>> --
>> Esta mensagem foi verificada pelo sistema de antivírus e
>> acredita-se estar livre de perigo.
>>
>
> --
> Esta mensagem foi verificada pelo sistema de antivírus e
> acredita-se estar livre de perigo.



-- 
--
Abraços,
Mauricio de Araujo
[oɾnɐɹɐ ǝp oıɔıɹnɐɯ]

-- 
Esta mensagem foi verificada pelo sistema de antiv�rus e
 acredita-se estar livre de perigo.



[obm-l] Re: [obm-l] Problema olimpíada de maio

2019-01-22 Por tôpico Ralph Teixeira
Hm, tive uma ideia, confiram se funciona.

Seja S o conjunto dos numeros obtidos pela permutacao dos digitos de 1 a 7,
e seja x_i a quantidade de elementos de S que deixam resto i na divisao por
7 (i=0,1,2,3,4,5,6).

Agora vamos fazer dois pareamentos. (Ou seja, vamos criar funcoes f,g:S->S
tal que f(f(N))=N e g(g(N))=N para todo N em S).

PRIMEIRO: No primeiro pareamento, troque cada digito x de N=abcdefg pelo
digito 8-x obtendo o numero f(N) (por exemplo, o numero N=1574326 eh
pareado com f(N)=7314562). Claramente, f(N) esta em S, e f(f(N))=N. Como a
soma desses dois numeros eh N+f(N)=888, que deixa resto 1 na divisao
por 7, temos automaticamente que:
-- Se N mod 7 = 1, entao f(N) mod 7 = 0; em outras palavras para cada
numero N que deixa resto 1, temos exatamente um numero f(N) que deixa resto
0, e vice-versa; portanto x_1=x_0.
-- Se N mod 7 = 2, entao f(N) mod 7 = 6; portanto x_2=x_6.
-- Analogamente, x_3=x_5.

SEGUNDO: Agora, g(N) eh obtido a partir de N trocando cada digito x por
7-x, EXCETO O DIGITO 7 que eh mantido. Por exemplo, se N=1754326 entao
g(N)=6723451. Claramente g(N) estah em S, e g(g(N))=N. Agora, N+g(N) mod 7
= 777 = 0 (pois aquele 7 extra pode ser jogado fora sem alterar o resto
modulo 7). Assim, de maneira analoga ao pareamento anterior, concluimos que:
-- x_1=x_6; x_2=x_5; x_3=x_4.

Encadeando tudo, concluimos que x_0=x_1=x_2=...=x_6. Assim, o numero pedido
eh x_0=#(S)/7=6!.

Abraco, Ralph.

On Tue, Jan 22, 2019 at 9:57 PM Heitor Gama Ribeiro 
wrote:

> Consideramos todos os números de 7 dígitos que se obtém permutando de
> todas as maneiras possíveis os dígitos de 1234567. Quantos deles são
> divisíveis por 7?
>
> --
> Esta mensagem foi verificada pelo sistema de antivírus e
> acredita-se estar livre de perigo.
>

-- 
Esta mensagem foi verificada pelo sistema de antiv�rus e
 acredita-se estar livre de perigo.



Re: [obm-l] Problema de Geometria

2019-01-01 Por tôpico Pacini Bores
 

Como disse anteriormente, o enunciado está com problemas. 

Pacini 

Em 31/12/2018 23:19, Pacini Bores escreveu: 

> Oi Marcelo, 
> 
> Está me parecendo que fixando o vértice B e variando o vertice C nas 
> condições do problema , que o ângulo pedido está variando  Pode ser que 
> eu esteja errado, vou verificar!!! 
> 
> Pacini 
> 
> Em 31/12/2018 20:03, Marcelo de Moura Costa escreveu: 
> 
>> Caros colegas, me deparei com um problema que até então não estou enxergando 
>> uma solução, gostaria de uma ajuda. 
>> 
>> Dado um triângulo ABC, tem-se que o ângulo referente ao vértice A mede 48º, 
>> no lado AB tem-se o ponto D de modo que o segmento CD é bissetriz do ângulo 
>> referente ao vértice C. Tem-se o ponto E no lado BC de modo que o segmento 
>> BE é igual ao segmento DE. Determine o valor do ângulo CDE. 
>> 
>> "Matemática é o alfabeto com o qual Deus escreveu o Universo"
>> Galileu Galilei
>> Dúvidas de Matemática? Deixe seu problema em nosso Blog, tentaremos 
>> resolvê-la ou orientá-lo!
>> http://mathhiperbolica.wordpress.com [1] 
>> Marcelo de Moura Costa Currículo Lattes: 
>> http://lattes.cnpq.br/2692706484448480 [2] 
>> 
>> [3]
>> Livre de vírus. www.avast.com [3].
>> 
>> -- 
>> Esta mensagem foi verificada pelo sistema de antivrus e 
>> acredita-se estar livre de perigo.
> 
> -- 
> Esta mensagem foi verificada pelo sistema de antivrus e 
> acredita-se estar livre de perigo.

 

Links:
--
[1] http://mathhiperbolica.wordpress.com
[2] http://lattes.cnpq.br/2692706484448480
[3]
https://www.avast.com/sig-email?utm_medium=emailutm_source=linkutm_campaign=sig-emailutm_content=webmail
-- 
Esta mensagem foi verificada pelo sistema de antiv�rus e
 acredita-se estar livre de perigo.



Re: [obm-l] Problema de Geometria

2019-01-01 Por tôpico Pedro José
Boa noite!
Na verdade, se B>76 não tem resposta. O ponto E ficaria externo ao lado BC.
Teria que mudar o problema para E pertencente a l(B,C). Mas assim mesmo o
ânfulo CDE não seria constante.

Saudações,
PJMS

Em ter, 1 de jan de 2019 14:13, Pedro José  Boa tarde!
> Você tem certeza que o problema é esse.
> Se C=84 e B=48, dá 42.
> Se C=100 e B= 32, dá 66.
> Se B >= 90 não tem resposta.
> Saudações,
> PJMS
>
> Em seg, 31 de dez de 2018 20:12, Marcelo de Moura Costa <
> mat.mo...@gmail.com escreveu:
>
>> Caros colegas, me deparei com um problema que até então não estou
>> enxergando uma solução, gostaria de uma ajuda.
>>
>> Dado um triângulo ABC, tem-se que o ângulo referente ao vértice A mede
>> 48º, no lado AB tem-se o ponto D de modo que o segmento CD é bissetriz do
>> ângulo referente ao vértice C. Tem-se o ponto E no lado BC de modo que o
>> segmento BE é igual ao segmento DE. Determine o valor do ângulo CDE.
>>
>>
>> "Matemática é o alfabeto com o qual Deus escreveu o Universo"
>> Galileu Galilei
>> Dúvidas de Matemática? Deixe seu problema em nosso Blog, tentaremos
>> resolvê-la ou orientá-lo!
>> http://mathhiperbolica.wordpress.com
>> Marcelo de Moura Costa
>> Currículo Lattes: http://lattes.cnpq.br/2692706484448480
>>
>>
>> 
>>  Livre
>> de vírus. www.avast.com
>> .
>>
>> <#m_580051917356088402_m_-7730567473249303737_DAB4FAD8-2DD7-40BB-A1B8-4E2AA1F9FDF2>
>>
>> --
>> Esta mensagem foi verificada pelo sistema de antivírus e
>> acredita-se estar livre de perigo.
>
>
> Em 31 de dez de 2018 20:12, "Marcelo de Moura Costa" 
> escreveu:
>
> Caros colegas, me deparei com um problema que até então não estou
> enxergando uma solução, gostaria de uma ajuda.
>
> Dado um triângulo ABC, tem-se que o ângulo referente ao vértice A mede
> 48º, no lado AB tem-se o ponto D de modo que o segmento CD é bissetriz do
> ângulo referente ao vértice C. Tem-se o ponto E no lado BC de modo que o
> segmento BE é igual ao segmento DE. Determine o valor do ângulo CDE.
>
>
> "Matemática é o alfabeto com o qual Deus escreveu o Universo"
> Galileu Galilei
> Dúvidas de Matemática? Deixe seu problema em nosso Blog, tentaremos
> resolvê-la ou orientá-lo!
> http://mathhiperbolica.wordpress.com
> Marcelo de Moura Costa
> Currículo Lattes: http://lattes.cnpq.br/2692706484448480
>
>
> 
>  Livre
> de vírus. www.avast.com
> .
>
>
> <#m_580051917356088402_m_-7730567473249303737_DAB4FAD8-2DD7-40BB-A1B8-4E2AA1F9FDF2>
>
> --
> Esta mensagem foi verificada pelo sistema de antivírus e
> acredita-se estar livre de perigo.
>
>
>

-- 
Esta mensagem foi verificada pelo sistema de antiv�rus e
 acredita-se estar livre de perigo.



Re: [obm-l] Problema de Geometria

2019-01-01 Por tôpico Pedro José
Boa tarde!
Você tem certeza que o problema é esse.
Se C=84 e B=48, dá 42.
Se C=100 e B= 32, dá 66.
Se B >= 90 não tem resposta.
Saudações,
PJMS

Em seg, 31 de dez de 2018 20:12, Marcelo de Moura Costa  Caros colegas, me deparei com um problema que até então não estou
> enxergando uma solução, gostaria de uma ajuda.
>
> Dado um triângulo ABC, tem-se que o ângulo referente ao vértice A mede
> 48º, no lado AB tem-se o ponto D de modo que o segmento CD é bissetriz do
> ângulo referente ao vértice C. Tem-se o ponto E no lado BC de modo que o
> segmento BE é igual ao segmento DE. Determine o valor do ângulo CDE.
>
>
> "Matemática é o alfabeto com o qual Deus escreveu o Universo"
> Galileu Galilei
> Dúvidas de Matemática? Deixe seu problema em nosso Blog, tentaremos
> resolvê-la ou orientá-lo!
> http://mathhiperbolica.wordpress.com
> Marcelo de Moura Costa
> Currículo Lattes: http://lattes.cnpq.br/2692706484448480
>
>
> 
>  Livre
> de vírus. www.avast.com
> .
> <#m_-7730567473249303737_DAB4FAD8-2DD7-40BB-A1B8-4E2AA1F9FDF2>
>
> --
> Esta mensagem foi verificada pelo sistema de antivírus e
> acredita-se estar livre de perigo.


Em 31 de dez de 2018 20:12, "Marcelo de Moura Costa" 
escreveu:

Caros colegas, me deparei com um problema que até então não estou
enxergando uma solução, gostaria de uma ajuda.

Dado um triângulo ABC, tem-se que o ângulo referente ao vértice A mede 48º,
no lado AB tem-se o ponto D de modo que o segmento CD é bissetriz do ângulo
referente ao vértice C. Tem-se o ponto E no lado BC de modo que o segmento
BE é igual ao segmento DE. Determine o valor do ângulo CDE.


"Matemática é o alfabeto com o qual Deus escreveu o Universo"
Galileu Galilei
Dúvidas de Matemática? Deixe seu problema em nosso Blog, tentaremos
resolvê-la ou orientá-lo!
http://mathhiperbolica.wordpress.com
Marcelo de Moura Costa
Currículo Lattes: http://lattes.cnpq.br/2692706484448480


Livre
de vírus. www.avast.com
.

<#m_-7730567473249303737_DAB4FAD8-2DD7-40BB-A1B8-4E2AA1F9FDF2>

-- 
Esta mensagem foi verificada pelo sistema de antivírus e
acredita-se estar livre de perigo.

-- 
Esta mensagem foi verificada pelo sistema de antiv�rus e
 acredita-se estar livre de perigo.



Re: [obm-l] Problema de Geometria

2018-12-31 Por tôpico Pacini Bores
 

Oi Marcelo, 

Está me parecendo que fixando o vértice B e variando o vertice C nas
condições do problema , que o ângulo pedido está variando  Pode ser
que eu esteja errado, vou verificar!!! 

Pacini 

Em 31/12/2018 20:03, Marcelo de Moura Costa escreveu: 

> Caros colegas, me deparei com um problema que até então não estou enxergando 
> uma solução, gostaria de uma ajuda. 
> 
> Dado um triângulo ABC, tem-se que o ângulo referente ao vértice A mede 48º, 
> no lado AB tem-se o ponto D de modo que o segmento CD é bissetriz do ângulo 
> referente ao vértice C. Tem-se o ponto E no lado BC de modo que o segmento BE 
> é igual ao segmento DE. Determine o valor do ângulo CDE. 
> 
> "Matemática é o alfabeto com o qual Deus escreveu o Universo"
> Galileu Galilei
> Dúvidas de Matemática? Deixe seu problema em nosso Blog, tentaremos 
> resolvê-la ou orientá-lo!
> http://mathhiperbolica.wordpress.com [1] 
> Marcelo de Moura Costa Currículo Lattes: 
> http://lattes.cnpq.br/2692706484448480 [2] 
> 
> [3]
> Livre de vírus. www.avast.com [3].
> 
> -- 
> Esta mensagem foi verificada pelo sistema de antivrus e 
> acredita-se estar livre de perigo.

 

Links:
--
[1] http://mathhiperbolica.wordpress.com
[2] http://lattes.cnpq.br/2692706484448480
[3]
https://www.avast.com/sig-email?utm_medium=emailutm_source=linkutm_campaign=sig-emailutm_content=webmail
-- 
Esta mensagem foi verificada pelo sistema de antiv�rus e
 acredita-se estar livre de perigo.



Re: [obm-l] Problema

2018-11-26 Por tôpico benedito
Bruno, 

Pelo que eu percebi, se as dimensões do tabuleiro quadrado forem pares, não se 
consegue estender para o padrão 2019 x 2019. Por isso, comecei tentando no 
tabuleiro 3 x 3. 
Benedito 


De: "Bruno Visnadi"  
Para: obm-l@mat.puc-rio.br 
Enviadas: Segunda-feira, 26 de novembro de 2018 17:52:58 
Assunto: Re: [obm-l] Problema 

Tentei um tabuleiro 12x12 e consegui uma configuração que não tem nenhuma 
lâmpada ruim. Acho que dá para estender o padrão para um 2017x2017. Mas me 
parece que a paridade importa e talvez o caso 2017x2017 tenha um mínimo de uma 
lâmpada ruim. 
https://i.imgur.com/HhWrZzu.png 

Em seg, 26 de nov de 2018 às 09:27, Claudio Buffara < claudio.buff...@gmail.com 
> escreveu: 



Sem pensar muito no problema, aqui vai uma sugestão: tente com um tabuleiro 
menor, 4x4 ou 5x5, pra ver se acha algum padrão. 
[]s, 
Claudio. 

On Mon, Nov 26, 2018 at 9:52 AM < bened...@ufrnet.br > wrote: 

BQ_BEGIN

Alguém pode me dar uma sugestão para o problema seguinte? 

Problema 
Há uma lâmpada em cada casa de um tabuleiro 2019 x 2019 . Cada lâmpada está 
acesa ou apagada. Uma lâmpada é chamada de ruim se ela tem um número par de 
vizinhas que estão acesas. 
Qual é o menor número possível de lâmpadas ruins no tabuleiro? 
(Duas lâmpadas são vizinhas se elas se encontram em casas do tabuleiro que 
compartilham um lado.) 

NOTA -Tentei raciocinar com o tabuleiro no qual as casas estejam pintadas 
alternadamente de branco e preto. Desse modo, pode-se ver que como as vizinhas 
de uma casa branca são todas pretas, parece que uma casa ruim branca não 
influencia outra branca. Parece que o mesmo deve acontecer com as casas pretas. 
No caso particular do tabuleiro 3x3, encontrei que o número procurado é 1: 
B P B 
P B P 
B P B 

Obrigado. 

Benedito Freire 


-- 
Esta mensagem foi verificada pelo sistema de antivírus e 
acredita-se estar livre de perigo. 




-- 
Esta mensagem foi verificada pelo sistema de antivírus e 
acredita-se estar livre de perigo. 
BQ_END


-- 
Esta mensagem foi verificada pelo sistema de antiv�rus e 
acredita-se estar livre de perigo. 

-- 
Esta mensagem foi verificada pelo sistema de antiv�rus e
 acredita-se estar livre de perigo.



Re: [obm-l] Problema

2018-11-26 Por tôpico Bruno Visnadi
Tentei um tabuleiro 12x12 e consegui uma configuração que não tem nenhuma
lâmpada ruim. Acho que dá para estender o padrão para um 2017x2017. Mas me
parece que a paridade importa e talvez o caso 2017x2017 tenha um mínimo de
uma lâmpada ruim.
https://i.imgur.com/HhWrZzu.png

Em seg, 26 de nov de 2018 às 09:27, Claudio Buffara <
claudio.buff...@gmail.com> escreveu:

> Sem pensar muito no problema, aqui vai uma sugestão: tente com um
> tabuleiro menor, 4x4 ou 5x5, pra ver se acha algum padrão.
>
> []s,
> Claudio.
>
> On Mon, Nov 26, 2018 at 9:52 AM  wrote:
>
>> Alguém pode me dar uma sugestão para o problema seguinte?
>>
>> *Problema*
>> Há uma lâmpada em cada casa de um tabuleiro 2019 x 2019 . Cada lâmpada
>> está acesa ou apagada. Uma lâmpada é chamada de *ruim* se ela tem um
>> número par de vizinhas que estão acesas.
>> Qual é o menor número possível de lâmpadas ruins no tabuleiro?
>> * (Duas lâmpadas são vizinhas se elas se encontram em casas do tabuleiro
>> que compartilham um lado.)*
>>
>> NOTA -Tentei raciocinar com o tabuleiro no qual as casas estejam pintadas
>> alternadamente de branco e preto. Desse modo, pode-se ver que como as
>> vizinhas de uma casa branca são todas pretas, parece que uma casa ruim
>> branca não influencia outra branca. Parece que o mesmo deve acontecer com
>> as casas pretas.
>> No caso particular do tabuleiro 3x3, encontrei que o número procurado é 1:
>> B P B
>> P B P
>> B P B
>>
>> Obrigado.
>>
>> Benedito Freire
>>
>>
>> --
>> Esta mensagem foi verificada pelo sistema de antivírus e
>> acredita-se estar livre de perigo.
>>
>
> --
> Esta mensagem foi verificada pelo sistema de antivírus e
> acredita-se estar livre de perigo.

-- 
Esta mensagem foi verificada pelo sistema de antiv�rus e
 acredita-se estar livre de perigo.



Re: [obm-l] Problema

2018-11-26 Por tôpico Claudio Buffara
Sem pensar muito no problema, aqui vai uma sugestão: tente com um tabuleiro
menor, 4x4 ou 5x5, pra ver se acha algum padrão.

[]s,
Claudio.

On Mon, Nov 26, 2018 at 9:52 AM  wrote:

> Alguém pode me dar uma sugestão para o problema seguinte?
>
> *Problema*
> Há uma lâmpada em cada casa de um tabuleiro 2019 x 2019 . Cada lâmpada
> está acesa ou apagada. Uma lâmpada é chamada de *ruim* se ela tem um
> número par de vizinhas que estão acesas.
> Qual é o menor número possível de lâmpadas ruins no tabuleiro?
> * (Duas lâmpadas são vizinhas se elas se encontram em casas do tabuleiro
> que compartilham um lado.)*
>
> NOTA -Tentei raciocinar com o tabuleiro no qual as casas estejam pintadas
> alternadamente de branco e preto. Desse modo, pode-se ver que como as
> vizinhas de uma casa branca são todas pretas, parece que uma casa ruim
> branca não influencia outra branca. Parece que o mesmo deve acontecer com
> as casas pretas.
> No caso particular do tabuleiro 3x3, encontrei que o número procurado é 1:
> B P B
> P B P
> B P B
>
> Obrigado.
>
> Benedito Freire
>
>
> --
> Esta mensagem foi verificada pelo sistema de antivírus e
> acredita-se estar livre de perigo.
>

-- 
Esta mensagem foi verificada pelo sistema de antiv�rus e
 acredita-se estar livre de perigo.



Re: [obm-l] Problema Simples de Probabilidade

2018-11-23 Por tôpico Luiz Antonio Rodrigues
Olá, Pedro!
Tudo bem?
Muito obrigado pela resposta!
Um abraço!
Luiz

On Thu, Nov 22, 2018, 7:13 PM Pedro José  Boa noite!
>
> Considerando o modelo equiprovável, já que não há menção ao contrário.
> Resolvi de outra maneira e também deu 13/35.
>
> Caminhos possíveis: PBB, BPB e BBQ ==> 2*(4*3*2)/(7*6*5) + (3*2)/(7*6)=
> 13/35
> P preta, B branca Q qualquer
> Menor do que 1/2, o que é esperado, uma vez que há mais bolas pretas do
> que brancas, não há como a probabilidade de se tirar mais brancas, seja
> superior a 50%, a menos se o modelo não for equiprovável.
>
> Saudações,
> PJMS
>
>
> Em qui, 22 de nov de 2018 às 07:35, Luiz Antonio Rodrigues <
> rodrigue...@gmail.com> escreveu:
>
>> Olá, Ralph!
>> Bom dia!
>> Cheguei neste resultado também!
>> Conclusão: gabarito incorreto!
>> Muito obrigado pela ajuda!
>> Um abraço!
>> Luiz
>>
>> On Thu, Nov 22, 2018, 1:58 AM Ralph Teixeira >
>>> Bolas B1,B2,B3,P1,P2,P3,P4.
>>>
>>> Ha C(7,3)=35 maneiras igualmente provaveis de retirar 3 bolas
>>> simultaneamente (ignoro a ordem).
>>>
>>> Destas, tem C(3,2).C(4,1)+C(3,3).C(4,0) = 12+1=13 maneiras de tirar pelo
>>> menos 2 brancas (12 maneiras de tirar 2 brancas e 1 reta, mais uma de tirar
>>> 3 brancas).
>>>
>>> Entao eu acho 13/35... que nao eh 60%. Errei algo? :(
>>>
>>> Abraco, Ralph.
>>>
>>> On Wed, Nov 21, 2018 at 11:12 PM Luiz Antonio Rodrigues <
>>> rodrigue...@gmail.com> wrote:
>>>
 Olá, pessoal!
 Tudo bem?
 Resolvi o seguinte problema, que é simples, de muitas maneiras e não
 chego na resposta do gabarito, que supostamente é 60%.

 Uma urna contém 3 bolas brancas e 4 pretas. São retiradas
 simultaneamente 3 bolas da urna. Qual a probabilidade de que pelo menos 2
 bolas brancas sejam retiradas?

 Alguém pode me ajudar.
 Muito obrigado e um abraço!
 Luiz

 --
 Esta mensagem foi verificada pelo sistema de antivírus e
 acredita-se estar livre de perigo.
>>>
>>>
>>> --
>>> Esta mensagem foi verificada pelo sistema de antivírus e
>>> acredita-se estar livre de perigo.
>>
>>
>> --
>> Esta mensagem foi verificada pelo sistema de antivírus e
>> acredita-se estar livre de perigo.
>
>
> --
> Esta mensagem foi verificada pelo sistema de antivírus e
> acredita-se estar livre de perigo.

-- 
Esta mensagem foi verificada pelo sistema de antiv�rus e
 acredita-se estar livre de perigo.



Re: [obm-l] Problema Simples de Probabilidade

2018-11-22 Por tôpico Pedro José
Boa noite!

Considerando o modelo equiprovável, já que não há menção ao contrário.
Resolvi de outra maneira e também deu 13/35.

Caminhos possíveis: PBB, BPB e BBQ ==> 2*(4*3*2)/(7*6*5) + (3*2)/(7*6)=
13/35
P preta, B branca Q qualquer
Menor do que 1/2, o que é esperado, uma vez que há mais bolas pretas do que
brancas, não há como a probabilidade de se tirar mais brancas, seja
superior a 50%, a menos se o modelo não for equiprovável.

Saudações,
PJMS


Em qui, 22 de nov de 2018 às 07:35, Luiz Antonio Rodrigues <
rodrigue...@gmail.com> escreveu:

> Olá, Ralph!
> Bom dia!
> Cheguei neste resultado também!
> Conclusão: gabarito incorreto!
> Muito obrigado pela ajuda!
> Um abraço!
> Luiz
>
> On Thu, Nov 22, 2018, 1:58 AM Ralph Teixeira 
>> Bolas B1,B2,B3,P1,P2,P3,P4.
>>
>> Ha C(7,3)=35 maneiras igualmente provaveis de retirar 3 bolas
>> simultaneamente (ignoro a ordem).
>>
>> Destas, tem C(3,2).C(4,1)+C(3,3).C(4,0) = 12+1=13 maneiras de tirar pelo
>> menos 2 brancas (12 maneiras de tirar 2 brancas e 1 reta, mais uma de tirar
>> 3 brancas).
>>
>> Entao eu acho 13/35... que nao eh 60%. Errei algo? :(
>>
>> Abraco, Ralph.
>>
>> On Wed, Nov 21, 2018 at 11:12 PM Luiz Antonio Rodrigues <
>> rodrigue...@gmail.com> wrote:
>>
>>> Olá, pessoal!
>>> Tudo bem?
>>> Resolvi o seguinte problema, que é simples, de muitas maneiras e não
>>> chego na resposta do gabarito, que supostamente é 60%.
>>>
>>> Uma urna contém 3 bolas brancas e 4 pretas. São retiradas
>>> simultaneamente 3 bolas da urna. Qual a probabilidade de que pelo menos 2
>>> bolas brancas sejam retiradas?
>>>
>>> Alguém pode me ajudar.
>>> Muito obrigado e um abraço!
>>> Luiz
>>>
>>> --
>>> Esta mensagem foi verificada pelo sistema de antivírus e
>>> acredita-se estar livre de perigo.
>>
>>
>> --
>> Esta mensagem foi verificada pelo sistema de antivírus e
>> acredita-se estar livre de perigo.
>
>
> --
> Esta mensagem foi verificada pelo sistema de antivírus e
> acredita-se estar livre de perigo.

-- 
Esta mensagem foi verificada pelo sistema de antiv�rus e
 acredita-se estar livre de perigo.



Re: [obm-l] Problema Simples de Probabilidade

2018-11-22 Por tôpico Luiz Antonio Rodrigues
Olá, Ralph!
Bom dia!
Cheguei neste resultado também!
Conclusão: gabarito incorreto!
Muito obrigado pela ajuda!
Um abraço!
Luiz

On Thu, Nov 22, 2018, 1:58 AM Ralph Teixeira  Bolas B1,B2,B3,P1,P2,P3,P4.
>
> Ha C(7,3)=35 maneiras igualmente provaveis de retirar 3 bolas
> simultaneamente (ignoro a ordem).
>
> Destas, tem C(3,2).C(4,1)+C(3,3).C(4,0) = 12+1=13 maneiras de tirar pelo
> menos 2 brancas (12 maneiras de tirar 2 brancas e 1 reta, mais uma de tirar
> 3 brancas).
>
> Entao eu acho 13/35... que nao eh 60%. Errei algo? :(
>
> Abraco, Ralph.
>
> On Wed, Nov 21, 2018 at 11:12 PM Luiz Antonio Rodrigues <
> rodrigue...@gmail.com> wrote:
>
>> Olá, pessoal!
>> Tudo bem?
>> Resolvi o seguinte problema, que é simples, de muitas maneiras e não
>> chego na resposta do gabarito, que supostamente é 60%.
>>
>> Uma urna contém 3 bolas brancas e 4 pretas. São retiradas simultaneamente
>> 3 bolas da urna. Qual a probabilidade de que pelo menos 2 bolas brancas
>> sejam retiradas?
>>
>> Alguém pode me ajudar.
>> Muito obrigado e um abraço!
>> Luiz
>>
>> --
>> Esta mensagem foi verificada pelo sistema de antivírus e
>> acredita-se estar livre de perigo.
>
>
> --
> Esta mensagem foi verificada pelo sistema de antivírus e
> acredita-se estar livre de perigo.

-- 
Esta mensagem foi verificada pelo sistema de antiv�rus e
 acredita-se estar livre de perigo.



Re: [obm-l] Problema Simples de Probabilidade

2018-11-21 Por tôpico Ralph Teixeira
Bolas B1,B2,B3,P1,P2,P3,P4.

Ha C(7,3)=35 maneiras igualmente provaveis de retirar 3 bolas
simultaneamente (ignoro a ordem).

Destas, tem C(3,2).C(4,1)+C(3,3).C(4,0) = 12+1=13 maneiras de tirar pelo
menos 2 brancas (12 maneiras de tirar 2 brancas e 1 reta, mais uma de tirar
3 brancas).

Entao eu acho 13/35... que nao eh 60%. Errei algo? :(

Abraco, Ralph.

On Wed, Nov 21, 2018 at 11:12 PM Luiz Antonio Rodrigues <
rodrigue...@gmail.com> wrote:

> Olá, pessoal!
> Tudo bem?
> Resolvi o seguinte problema, que é simples, de muitas maneiras e não chego
> na resposta do gabarito, que supostamente é 60%.
>
> Uma urna contém 3 bolas brancas e 4 pretas. São retiradas simultaneamente
> 3 bolas da urna. Qual a probabilidade de que pelo menos 2 bolas brancas
> sejam retiradas?
>
> Alguém pode me ajudar.
> Muito obrigado e um abraço!
> Luiz
>
> --
> Esta mensagem foi verificada pelo sistema de antivírus e
> acredita-se estar livre de perigo.

-- 
Esta mensagem foi verificada pelo sistema de antiv�rus e
 acredita-se estar livre de perigo.



Re: [obm-l] Problema 6 - IMO 2001

2018-11-15 Por tôpico Pedro José
Boa tarde!
Equivoquei-me quando deduzi a fórmula da diagonal do quadrilátero.
Considerei x o ângulo BAD e y o ângulo ABC mas coloquei senx/seny = AC/BD,
quando era o inverso.
Na verdade onde AC é AB e vice-versa. Até porque BD é que permanece
constante em qualquer ordem e não AC. BD^2=a^2-ac+c^2.
Saudações,
PJMS.


Em Qui, 15 de nov de 2018 13:08, Pedro José  escreveu:

> Boa tarde!
> Em tempo, a ordem usada dos vértices foi A, B, C, D, no sentido
> trigonométrico. Só variou a nomemclatura da medida dos lados.
> Saudações,
> PJMS
>
> Em Qui, 15 de nov de 2018 13:03, Pedro José 
> escreveu:
>
>> Boa tarde!
>> Não tinha atinado que no segundo caso, o fator (ab+cd) está no numerador
>> do valor do quadrado de ambas diagonais.
>> Realmente serve de qualquer jeito.
>> (i) a, b, d, c no sentido trigonométrico.
>> (ad+bc)*(ab+cd) =AC^2*(ac+bd)
>> (ii) a, d, b, c no mesmo sentido.
>> (ab+cd)*(ac+bd)=BD^2*(ad+bc)
>> (ab+cd)*(ad+bc)=AC^2*(ac+bd)
>>
>> Eu havia parado na primeira equação de (ii) pois, perderia o recurso d|x
>> e d>x, absurdo.
>> Mas na segunda de (ii) volto a ter esse recurso.
>> Por isso havia questionado a ordem. Erroneamente julguei que só valesse
>> para a primeira ordem.
>> Aí seria complicado defini-la.
>> A argumentação é a mesma da solução sugerida pelo Cláudio.
>> Se (ab+cd) é primo, então (ac+bd) | (ad+bc); pois, ac+bd >1.
>> Mas ac+bd>ad+bc, absurdo.
>> ab+cd é composto.
>>
>> Saudações,
>> PJMS
>>
>>
>>
>> Em Qui, 15 de nov de 2018 08:57, Anderson Torres <
>> torres.anderson...@gmail.com> escreveu:
>>
>>>
>>>
>>> Em qua, 14 de nov de 2018 16:53, Pedro José >> escreveu:
>>>
 Boa tarde!

 Porém, me ficou uma dúvida! Como definir a ordem dos lados, os de
 medidas a e c devem ser adjacentes, assim como os de medida b e d.
 Mas como definir se os de a e b ou de a e d são adjacentes???

>>>
>>> Bem, tecnicamente qualquer um serviria, afinal a equação é simétrica. Se
>>> trocar a com c ou b com d, obtemos uma solução nova.
>>>
>>> Inda lembro vagamente que o Gugu deu a solução mais ignorante possível:
>>> trata tudo como uma equação de segundo grau em D, verifica quando o delta é
>>> quadrado e substitui loucamente.
>>>

 Grato,
 PJMS

 Em ter, 13 de nov de 2018 às 13:44, Pedro José 
 escreveu:

> Bom dia!
>
> Depois da observação do Anderson Torres é que atinei o quanto é bonita
> a sua solução se você prosseguir.
> Sua preocupação não deve ser em relação ao produto AC*BD, nem com os
> valores AC ou BD; mas sim que tanto BD^2, como AC^2 são inteiros.
> Falta uma beirinha e a solução indicada pelo Cláudio tem a dica final
> Pelo menos para o caminho que vislumbrei.
>
> Saudações,
> PJMS.
>
>
>
>
> Em seg, 12 de nov de 2018 às 16:39, Anderson Torres <
> torres.anderson...@gmail.com> escreveu:
>
>> Você quase resolveu! Posso dizer que esta era basicamente a solução
>> oficial. Tente mais um pouco, que o caminho é esse.
>>
>> Em 8 de nov de 2018 23:27, "Jeferson Almir" 
>> escreveu:
>>
>> Pessoal peço ajuda  no problema :
>>
>> Sejam a, b , c , d inteiros e a > b > c > d > 0 .
>> Suponha que
>> ac + bd = ( b+ d + a - c )( b+ d -a + c )
>>
>> Mostre que ab + cd não é primo .
>>
>>
>> A minha ideia foi:
>>
>> Abrindo a relação de cima temos
>>
>> a^2 -ac + c^2 = b^2 + bd + d^2
>>
>> Então motivado pela ideia de usar geometria que um amigo falou fiz a
>> suposição que temos um quadrilátero de lados a, d,b e c respectivamente e
>>  nessa ultima relação usando lei dos cossenos teríamos A = 60° e C = 120°
>> concluindo então que ABCD é inscritível . Aplicando Ptolomeu temos que 
>> ACxBD=
>> ab + cd e usando desigualdade triangular podemos afirmar que AC e BD não
>> podem ser 1 . Mas ainda tem a possibilidade AC e BD  serem racionais !!
>> Como provar que não podem ser ???
>>
>> --
>> Esta mensagem foi verificada pelo sistema de antivírus e
>> acredita-se estar livre de perigo.
>>
>>
>>
>> --
>> Esta mensagem foi verificada pelo sistema de antivírus e
>> acredita-se estar livre de perigo.
>
>
 --
 Esta mensagem foi verificada pelo sistema de antivírus e
 acredita-se estar livre de perigo.
>>>
>>>
>>> --
>>> Esta mensagem foi verificada pelo sistema de antivírus e
>>> acredita-se estar livre de perigo.
>>
>>

-- 
Esta mensagem foi verificada pelo sistema de antiv�rus e
 acredita-se estar livre de perigo.



Re: [obm-l] Problema 6 - IMO 2001

2018-11-15 Por tôpico Pedro José
Boa tarde!
Em tempo, a ordem usada dos vértices foi A, B, C, D, no sentido
trigonométrico. Só variou a nomemclatura da medida dos lados.
Saudações,
PJMS

Em Qui, 15 de nov de 2018 13:03, Pedro José  escreveu:

> Boa tarde!
> Não tinha atinado que no segundo caso, o fator (ab+cd) está no numerador
> do valor do quadrado de ambas diagonais.
> Realmente serve de qualquer jeito.
> (i) a, b, d, c no sentido trigonométrico.
> (ad+bc)*(ab+cd) =AC^2*(ac+bd)
> (ii) a, d, b, c no mesmo sentido.
> (ab+cd)*(ac+bd)=BD^2*(ad+bc)
> (ab+cd)*(ad+bc)=AC^2*(ac+bd)
>
> Eu havia parado na primeira equação de (ii) pois, perderia o recurso d|x e
> d>x, absurdo.
> Mas na segunda de (ii) volto a ter esse recurso.
> Por isso havia questionado a ordem. Erroneamente julguei que só valesse
> para a primeira ordem.
> Aí seria complicado defini-la.
> A argumentação é a mesma da solução sugerida pelo Cláudio.
> Se (ab+cd) é primo, então (ac+bd) | (ad+bc); pois, ac+bd >1.
> Mas ac+bd>ad+bc, absurdo.
> ab+cd é composto.
>
> Saudações,
> PJMS
>
>
>
> Em Qui, 15 de nov de 2018 08:57, Anderson Torres <
> torres.anderson...@gmail.com> escreveu:
>
>>
>>
>> Em qua, 14 de nov de 2018 16:53, Pedro José > escreveu:
>>
>>> Boa tarde!
>>>
>>> Porém, me ficou uma dúvida! Como definir a ordem dos lados, os de
>>> medidas a e c devem ser adjacentes, assim como os de medida b e d.
>>> Mas como definir se os de a e b ou de a e d são adjacentes???
>>>
>>
>> Bem, tecnicamente qualquer um serviria, afinal a equação é simétrica. Se
>> trocar a com c ou b com d, obtemos uma solução nova.
>>
>> Inda lembro vagamente que o Gugu deu a solução mais ignorante possível:
>> trata tudo como uma equação de segundo grau em D, verifica quando o delta é
>> quadrado e substitui loucamente.
>>
>>>
>>> Grato,
>>> PJMS
>>>
>>> Em ter, 13 de nov de 2018 às 13:44, Pedro José 
>>> escreveu:
>>>
 Bom dia!

 Depois da observação do Anderson Torres é que atinei o quanto é bonita
 a sua solução se você prosseguir.
 Sua preocupação não deve ser em relação ao produto AC*BD, nem com os
 valores AC ou BD; mas sim que tanto BD^2, como AC^2 são inteiros.
 Falta uma beirinha e a solução indicada pelo Cláudio tem a dica final
 Pelo menos para o caminho que vislumbrei.

 Saudações,
 PJMS.




 Em seg, 12 de nov de 2018 às 16:39, Anderson Torres <
 torres.anderson...@gmail.com> escreveu:

> Você quase resolveu! Posso dizer que esta era basicamente a solução
> oficial. Tente mais um pouco, que o caminho é esse.
>
> Em 8 de nov de 2018 23:27, "Jeferson Almir" 
> escreveu:
>
> Pessoal peço ajuda  no problema :
>
> Sejam a, b , c , d inteiros e a > b > c > d > 0 .
> Suponha que
> ac + bd = ( b+ d + a - c )( b+ d -a + c )
>
> Mostre que ab + cd não é primo .
>
>
> A minha ideia foi:
>
> Abrindo a relação de cima temos
>
> a^2 -ac + c^2 = b^2 + bd + d^2
>
> Então motivado pela ideia de usar geometria que um amigo falou fiz a
> suposição que temos um quadrilátero de lados a, d,b e c respectivamente e
>  nessa ultima relação usando lei dos cossenos teríamos A = 60° e C = 120°
> concluindo então que ABCD é inscritível . Aplicando Ptolomeu temos que 
> ACxBD=
> ab + cd e usando desigualdade triangular podemos afirmar que AC e BD não
> podem ser 1 . Mas ainda tem a possibilidade AC e BD  serem racionais !!
> Como provar que não podem ser ???
>
> --
> Esta mensagem foi verificada pelo sistema de antivírus e
> acredita-se estar livre de perigo.
>
>
>
> --
> Esta mensagem foi verificada pelo sistema de antivírus e
> acredita-se estar livre de perigo.


>>> --
>>> Esta mensagem foi verificada pelo sistema de antivírus e
>>> acredita-se estar livre de perigo.
>>
>>
>> --
>> Esta mensagem foi verificada pelo sistema de antivírus e
>> acredita-se estar livre de perigo.
>
>

-- 
Esta mensagem foi verificada pelo sistema de antiv�rus e
 acredita-se estar livre de perigo.



Re: [obm-l] Problema 6 - IMO 2001

2018-11-15 Por tôpico Pedro José
Boa tarde!
Não tinha atinado que no segundo caso, o fator (ab+cd) está no numerador do
valor do quadrado de ambas diagonais.
Realmente serve de qualquer jeito.
(i) a, b, d, c no sentido trigonométrico.
(ad+bc)*(ab+cd) =AC^2*(ac+bd)
(ii) a, d, b, c no mesmo sentido.
(ab+cd)*(ac+bd)=BD^2*(ad+bc)
(ab+cd)*(ad+bc)=AC^2*(ac+bd)

Eu havia parado na primeira equação de (ii) pois, perderia o recurso d|x e
d>x, absurdo.
Mas na segunda de (ii) volto a ter esse recurso.
Por isso havia questionado a ordem. Erroneamente julguei que só valesse
para a primeira ordem.
Aí seria complicado defini-la.
A argumentação é a mesma da solução sugerida pelo Cláudio.
Se (ab+cd) é primo, então (ac+bd) | (ad+bc); pois, ac+bd >1.
Mas ac+bd>ad+bc, absurdo.
ab+cd é composto.

Saudações,
PJMS



Em Qui, 15 de nov de 2018 08:57, Anderson Torres <
torres.anderson...@gmail.com> escreveu:

>
>
> Em qua, 14 de nov de 2018 16:53, Pedro José 
>> Boa tarde!
>>
>> Porém, me ficou uma dúvida! Como definir a ordem dos lados, os de medidas
>> a e c devem ser adjacentes, assim como os de medida b e d.
>> Mas como definir se os de a e b ou de a e d são adjacentes???
>>
>
> Bem, tecnicamente qualquer um serviria, afinal a equação é simétrica. Se
> trocar a com c ou b com d, obtemos uma solução nova.
>
> Inda lembro vagamente que o Gugu deu a solução mais ignorante possível:
> trata tudo como uma equação de segundo grau em D, verifica quando o delta é
> quadrado e substitui loucamente.
>
>>
>> Grato,
>> PJMS
>>
>> Em ter, 13 de nov de 2018 às 13:44, Pedro José 
>> escreveu:
>>
>>> Bom dia!
>>>
>>> Depois da observação do Anderson Torres é que atinei o quanto é bonita a
>>> sua solução se você prosseguir.
>>> Sua preocupação não deve ser em relação ao produto AC*BD, nem com os
>>> valores AC ou BD; mas sim que tanto BD^2, como AC^2 são inteiros.
>>> Falta uma beirinha e a solução indicada pelo Cláudio tem a dica final
>>> Pelo menos para o caminho que vislumbrei.
>>>
>>> Saudações,
>>> PJMS.
>>>
>>>
>>>
>>>
>>> Em seg, 12 de nov de 2018 às 16:39, Anderson Torres <
>>> torres.anderson...@gmail.com> escreveu:
>>>
 Você quase resolveu! Posso dizer que esta era basicamente a solução
 oficial. Tente mais um pouco, que o caminho é esse.

 Em 8 de nov de 2018 23:27, "Jeferson Almir" 
 escreveu:

 Pessoal peço ajuda  no problema :

 Sejam a, b , c , d inteiros e a > b > c > d > 0 .
 Suponha que
 ac + bd = ( b+ d + a - c )( b+ d -a + c )

 Mostre que ab + cd não é primo .


 A minha ideia foi:

 Abrindo a relação de cima temos

 a^2 -ac + c^2 = b^2 + bd + d^2

 Então motivado pela ideia de usar geometria que um amigo falou fiz a
 suposição que temos um quadrilátero de lados a, d,b e c respectivamente e
  nessa ultima relação usando lei dos cossenos teríamos A = 60° e C = 120°
 concluindo então que ABCD é inscritível . Aplicando Ptolomeu temos que 
 ACxBD=
 ab + cd e usando desigualdade triangular podemos afirmar que AC e BD não
 podem ser 1 . Mas ainda tem a possibilidade AC e BD  serem racionais !!
 Como provar que não podem ser ???

 --
 Esta mensagem foi verificada pelo sistema de antivírus e
 acredita-se estar livre de perigo.



 --
 Esta mensagem foi verificada pelo sistema de antivírus e
 acredita-se estar livre de perigo.
>>>
>>>
>> --
>> Esta mensagem foi verificada pelo sistema de antivírus e
>> acredita-se estar livre de perigo.
>
>
> --
> Esta mensagem foi verificada pelo sistema de antivírus e
> acredita-se estar livre de perigo.

-- 
Esta mensagem foi verificada pelo sistema de antiv�rus e
 acredita-se estar livre de perigo.



Re: [obm-l] Problema 6 - IMO 2001

2018-11-15 Por tôpico Anderson Torres
Em qua, 14 de nov de 2018 16:53, Pedro José  Boa tarde!
>
> Porém, me ficou uma dúvida! Como definir a ordem dos lados, os de medidas
> a e c devem ser adjacentes, assim como os de medida b e d.
> Mas como definir se os de a e b ou de a e d são adjacentes???
>

Bem, tecnicamente qualquer um serviria, afinal a equação é simétrica. Se
trocar a com c ou b com d, obtemos uma solução nova.

Inda lembro vagamente que o Gugu deu a solução mais ignorante possível:
trata tudo como uma equação de segundo grau em D, verifica quando o delta é
quadrado e substitui loucamente.

>
> Grato,
> PJMS
>
> Em ter, 13 de nov de 2018 às 13:44, Pedro José 
> escreveu:
>
>> Bom dia!
>>
>> Depois da observação do Anderson Torres é que atinei o quanto é bonita a
>> sua solução se você prosseguir.
>> Sua preocupação não deve ser em relação ao produto AC*BD, nem com os
>> valores AC ou BD; mas sim que tanto BD^2, como AC^2 são inteiros.
>> Falta uma beirinha e a solução indicada pelo Cláudio tem a dica final
>> Pelo menos para o caminho que vislumbrei.
>>
>> Saudações,
>> PJMS.
>>
>>
>>
>>
>> Em seg, 12 de nov de 2018 às 16:39, Anderson Torres <
>> torres.anderson...@gmail.com> escreveu:
>>
>>> Você quase resolveu! Posso dizer que esta era basicamente a solução
>>> oficial. Tente mais um pouco, que o caminho é esse.
>>>
>>> Em 8 de nov de 2018 23:27, "Jeferson Almir" 
>>> escreveu:
>>>
>>> Pessoal peço ajuda  no problema :
>>>
>>> Sejam a, b , c , d inteiros e a > b > c > d > 0 .
>>> Suponha que
>>> ac + bd = ( b+ d + a - c )( b+ d -a + c )
>>>
>>> Mostre que ab + cd não é primo .
>>>
>>>
>>> A minha ideia foi:
>>>
>>> Abrindo a relação de cima temos
>>>
>>> a^2 -ac + c^2 = b^2 + bd + d^2
>>>
>>> Então motivado pela ideia de usar geometria que um amigo falou fiz a
>>> suposição que temos um quadrilátero de lados a, d,b e c respectivamente e
>>>  nessa ultima relação usando lei dos cossenos teríamos A = 60° e C = 120°
>>> concluindo então que ABCD é inscritível . Aplicando Ptolomeu temos que 
>>> ACxBD=
>>> ab + cd e usando desigualdade triangular podemos afirmar que AC e BD não
>>> podem ser 1 . Mas ainda tem a possibilidade AC e BD  serem racionais !!
>>> Como provar que não podem ser ???
>>>
>>> --
>>> Esta mensagem foi verificada pelo sistema de antivírus e
>>> acredita-se estar livre de perigo.
>>>
>>>
>>>
>>> --
>>> Esta mensagem foi verificada pelo sistema de antivírus e
>>> acredita-se estar livre de perigo.
>>
>>
> --
> Esta mensagem foi verificada pelo sistema de antivírus e
> acredita-se estar livre de perigo.

-- 
Esta mensagem foi verificada pelo sistema de antiv�rus e
 acredita-se estar livre de perigo.



Re: [obm-l] Problema 6 - IMO 2001

2018-11-14 Por tôpico Jeferson Almir
A ordem segue a,d,b,c no sentido horário devido a relação a^2 -ac + c^2 =
b^2 + bd + d^2

Em qua, 14 de nov de 2018 às 15:53, Pedro José 
escreveu:

> Boa tarde!
>
> Porém, me ficou uma dúvida! Como definir a ordem dos lados, os de medidas
> a e c devem ser adjacentes, assim como os de medida b e d.
> Mas como definir se os de a e b ou de a e d são adjacentes???
>
> Grato,
> PJMS
>
> Em ter, 13 de nov de 2018 às 13:44, Pedro José 
> escreveu:
>
>> Bom dia!
>>
>> Depois da observação do Anderson Torres é que atinei o quanto é bonita a
>> sua solução se você prosseguir.
>> Sua preocupação não deve ser em relação ao produto AC*BD, nem com os
>> valores AC ou BD; mas sim que tanto BD^2, como AC^2 são inteiros.
>> Falta uma beirinha e a solução indicada pelo Cláudio tem a dica final
>> Pelo menos para o caminho que vislumbrei.
>>
>> Saudações,
>> PJMS.
>>
>>
>>
>>
>> Em seg, 12 de nov de 2018 às 16:39, Anderson Torres <
>> torres.anderson...@gmail.com> escreveu:
>>
>>> Você quase resolveu! Posso dizer que esta era basicamente a solução
>>> oficial. Tente mais um pouco, que o caminho é esse.
>>>
>>> Em 8 de nov de 2018 23:27, "Jeferson Almir" 
>>> escreveu:
>>>
>>> Pessoal peço ajuda  no problema :
>>>
>>> Sejam a, b , c , d inteiros e a > b > c > d > 0 .
>>> Suponha que
>>> ac + bd = ( b+ d + a - c )( b+ d -a + c )
>>>
>>> Mostre que ab + cd não é primo .
>>>
>>>
>>> A minha ideia foi:
>>>
>>> Abrindo a relação de cima temos
>>>
>>> a^2 -ac + c^2 = b^2 + bd + d^2
>>>
>>> Então motivado pela ideia de usar geometria que um amigo falou fiz a
>>> suposição que temos um quadrilátero de lados a, d,b e c respectivamente e
>>>  nessa ultima relação usando lei dos cossenos teríamos A = 60° e C = 120°
>>> concluindo então que ABCD é inscritível . Aplicando Ptolomeu temos que 
>>> ACxBD=
>>> ab + cd e usando desigualdade triangular podemos afirmar que AC e BD não
>>> podem ser 1 . Mas ainda tem a possibilidade AC e BD  serem racionais !!
>>> Como provar que não podem ser ???
>>>
>>> --
>>> Esta mensagem foi verificada pelo sistema de antivírus e
>>> acredita-se estar livre de perigo.
>>>
>>>
>>>
>>> --
>>> Esta mensagem foi verificada pelo sistema de antivírus e
>>> acredita-se estar livre de perigo.
>>
>>
> --
> Esta mensagem foi verificada pelo sistema de antivírus e
> acredita-se estar livre de perigo.

-- 
Esta mensagem foi verificada pelo sistema de antiv�rus e
 acredita-se estar livre de perigo.



Re: [obm-l] Problema 6 - IMO 2001

2018-11-14 Por tôpico Pedro José
Boa tarde!

Porém, me ficou uma dúvida! Como definir a ordem dos lados, os de medidas a
e c devem ser adjacentes, assim como os de medida b e d.
Mas como definir se os de a e b ou de a e d são adjacentes???

Grato,
PJMS

Em ter, 13 de nov de 2018 às 13:44, Pedro José 
escreveu:

> Bom dia!
>
> Depois da observação do Anderson Torres é que atinei o quanto é bonita a
> sua solução se você prosseguir.
> Sua preocupação não deve ser em relação ao produto AC*BD, nem com os
> valores AC ou BD; mas sim que tanto BD^2, como AC^2 são inteiros.
> Falta uma beirinha e a solução indicada pelo Cláudio tem a dica final Pelo
> menos para o caminho que vislumbrei.
>
> Saudações,
> PJMS.
>
>
>
>
> Em seg, 12 de nov de 2018 às 16:39, Anderson Torres <
> torres.anderson...@gmail.com> escreveu:
>
>> Você quase resolveu! Posso dizer que esta era basicamente a solução
>> oficial. Tente mais um pouco, que o caminho é esse.
>>
>> Em 8 de nov de 2018 23:27, "Jeferson Almir" 
>> escreveu:
>>
>> Pessoal peço ajuda  no problema :
>>
>> Sejam a, b , c , d inteiros e a > b > c > d > 0 .
>> Suponha que
>> ac + bd = ( b+ d + a - c )( b+ d -a + c )
>>
>> Mostre que ab + cd não é primo .
>>
>>
>> A minha ideia foi:
>>
>> Abrindo a relação de cima temos
>>
>> a^2 -ac + c^2 = b^2 + bd + d^2
>>
>> Então motivado pela ideia de usar geometria que um amigo falou fiz a
>> suposição que temos um quadrilátero de lados a, d,b e c respectivamente e
>>  nessa ultima relação usando lei dos cossenos teríamos A = 60° e C = 120°
>> concluindo então que ABCD é inscritível . Aplicando Ptolomeu temos que ACxBD=
>> ab + cd e usando desigualdade triangular podemos afirmar que AC e BD não
>> podem ser 1 . Mas ainda tem a possibilidade AC e BD  serem racionais !!
>> Como provar que não podem ser ???
>>
>> --
>> Esta mensagem foi verificada pelo sistema de antivírus e
>> acredita-se estar livre de perigo.
>>
>>
>>
>> --
>> Esta mensagem foi verificada pelo sistema de antivírus e
>> acredita-se estar livre de perigo.
>
>

-- 
Esta mensagem foi verificada pelo sistema de antiv�rus e
 acredita-se estar livre de perigo.



Re: [obm-l] Problema 6 - IMO 2001

2018-11-13 Por tôpico Pedro José
Bom dia!

Depois da observação do Anderson Torres é que atinei o quanto é bonita a
sua solução se você prosseguir.
Sua preocupação não deve ser em relação ao produto AC*BD, nem com os
valores AC ou BD; mas sim que tanto BD^2, como AC^2 são inteiros.
Falta uma beirinha e a solução indicada pelo Cláudio tem a dica final Pelo
menos para o caminho que vislumbrei.

Saudações,
PJMS.




Em seg, 12 de nov de 2018 às 16:39, Anderson Torres <
torres.anderson...@gmail.com> escreveu:

> Você quase resolveu! Posso dizer que esta era basicamente a solução
> oficial. Tente mais um pouco, que o caminho é esse.
>
> Em 8 de nov de 2018 23:27, "Jeferson Almir" 
> escreveu:
>
> Pessoal peço ajuda  no problema :
>
> Sejam a, b , c , d inteiros e a > b > c > d > 0 .
> Suponha que
> ac + bd = ( b+ d + a - c )( b+ d -a + c )
>
> Mostre que ab + cd não é primo .
>
>
> A minha ideia foi:
>
> Abrindo a relação de cima temos
>
> a^2 -ac + c^2 = b^2 + bd + d^2
>
> Então motivado pela ideia de usar geometria que um amigo falou fiz a
> suposição que temos um quadrilátero de lados a, d,b e c respectivamente e
>  nessa ultima relação usando lei dos cossenos teríamos A = 60° e C = 120°
> concluindo então que ABCD é inscritível . Aplicando Ptolomeu temos que ACxBD=
> ab + cd e usando desigualdade triangular podemos afirmar que AC e BD não
> podem ser 1 . Mas ainda tem a possibilidade AC e BD  serem racionais !!
> Como provar que não podem ser ???
>
> --
> Esta mensagem foi verificada pelo sistema de antivírus e
> acredita-se estar livre de perigo.
>
>
>
> --
> Esta mensagem foi verificada pelo sistema de antivírus e
> acredita-se estar livre de perigo.

-- 
Esta mensagem foi verificada pelo sistema de antiv�rus e
 acredita-se estar livre de perigo.



Re: [obm-l] Problema 6 - IMO 2001

2018-11-12 Por tôpico Anderson Torres
Você quase resolveu! Posso dizer que esta era basicamente a solução
oficial. Tente mais um pouco, que o caminho é esse.

Em 8 de nov de 2018 23:27, "Jeferson Almir" 
escreveu:

Pessoal peço ajuda  no problema :

Sejam a, b , c , d inteiros e a > b > c > d > 0 .
Suponha que
ac + bd = ( b+ d + a - c )( b+ d -a + c )

Mostre que ab + cd não é primo .


A minha ideia foi:

Abrindo a relação de cima temos

a^2 -ac + c^2 = b^2 + bd + d^2

Então motivado pela ideia de usar geometria que um amigo falou fiz a
suposição que temos um quadrilátero de lados a, d,b e c respectivamente e
 nessa ultima relação usando lei dos cossenos teríamos A = 60° e C = 120°
concluindo então que ABCD é inscritível . Aplicando Ptolomeu temos que ACxBD=
ab + cd e usando desigualdade triangular podemos afirmar que AC e BD não
podem ser 1 . Mas ainda tem a possibilidade AC e BD  serem racionais !!
Como provar que não podem ser ???

-- 
Esta mensagem foi verificada pelo sistema de antivírus e
acredita-se estar livre de perigo.

-- 
Esta mensagem foi verificada pelo sistema de antiv�rus e
 acredita-se estar livre de perigo.



[obm-l] Re: [obm-l] Re: [obm-l] Re: [obm-l] [Problema] Achar o mínimo do valor absoluto de uma soma complexa

2018-11-12 Por tôpico Bernardo Freitas Paulo da Costa
Oi,

acho que você interpretou o enunciado de forma a "evitar os
complexos".  O problema original fala de "achar um ponto dentro do
círculo", então talvez não sejam apenas os pontos na circunferência
(como parece que a sua solução faz, ao ordenar todos pelos ângulos
centrais), mas qualquer ponto da forma r*cis(theta).  E daí talvez
tenha mais a ver com complexos...

On Mon, Nov 5, 2018 at 4:51 PM Pedro José  wrote:
>
> Boa tarde!
> Se entendi o que você quer, não entendi qual a relação com o mínimo de uma 
> soma complexa?
> Para resolver o problema que você propõe, entendi:
> (i) a excursão como a geração de um setor circular, a partir de um ponto 
> inicial, essa incursão tem dois sentidos, trigonométrico ou horário.
> (ii) Englobar um ponto significa que o ponto pertença ao setor circular, tem 
> que saber se incluem-se os pontos de borda ou não, como não há restrição vou 
> considerar que sim.
> (iii) Estou supondo que seu universo é plano.
> Minha sugestão é defina o conjunto de pontos em coordenadas polares.
> Defina a variável excursão, e dê a ela um sinal para definir o sentido,
> Faça um programa.
> Definir "arrays" dos pontos (caso não estejam em coordenadas polares, tem que 
> fazer uma sub-rotina para transformar as coordenadas em polares) Mod(I) e 
> Teta(I)
> Defina um array de contagens
> Definir uma rotina para contar o número de pontos. N
> Defina uma sub-rotina Achapontos para determinar o índice Imax, cujo 
> Engloba(I) seja máximo.
> ! Comentário: Atentar que podem retornar mais do que um índice. Portanto 
> deve-se definir um array Pontonotável e uma variável de contagem Nmax. E.g., 
> se tiverem três pontos que englobem o número máximo de pontos, deve retornar: 
> Array contagem, com os valores dos índices dos pontos que têm o máximo de 
> Engloba, nas três primeiras posições e o Valor Nmax=3.
> Aplique a sub-rotina de contagem no array de pontos e retorne com N.
> Entre com o valor de excursão
> Faça de I=1 a N
> Tetamax= max (teta(I);teta(i)+excursão)
> Tetamin=min(teta(I);teta(i)+excursão)
> Engloba(I)=0
> Faça de J=1 a N
> Se (teta(J)<=tetamax e teta(J)>=tetamin e mod(J)<=mod(I).
> ! comentário: A relação engloba será reflexiva. Todo ponto engloba si 
> próprio.Caso não se aceite a borda é só tirar os iguais da lógica acima.
> Engloba(I) = Engloba(I)+1
> Fim SE;
> Fim Faça
> Fim faça
> Aplica Sub-rotina acha pontos.
> Salva temos o máximo de pontos englobados para uma excurção de [excursão] 
> para [Nmax] pontos englobando [engloba(Nmax)]
> São eles:
> Faça de I=1 até N
> Pontonotável(I)
> Fim faça.
> !se tiver interesse salva todo array Engloba.
> FIM.
>
> Porém para qualquer setor existir um ponto que seja sempre o que englobe mais 
> pontos, creio que vá depender da nuvem, e.g.
>
> P1= (10,40)
> P2= ( 6,42)
> P3= (9,90)
> P4= (8,100)
> P5= (7,107)
> P6= (7,5; 108)
> Teremos para uma excursão de + 5 graus:
> Engloba (1) = 1;Engloba (2) = 0; Engloba (3) = 0; Engloba (4) = 0 , Engloba 
> (5) = 0 e Engloba (6) = 0
> P1 é o que engloba mais pontos.
> Para uma excursão de +10 graus:
> Engloba (1) = 1;Engloba (2) = 0; Engloba (3) = 1; Engloba (4) = 2 e Engloba 
> (5) = 0 e Engloba (6) = 0
> P3 é o que engloba mais pontos.
>
> Espero ter compreendido o proposto e ajudado.
> Mas o que tem haver com soma de complexos, módulo mínimo???
>
> Saudações,
> PJMS
>
> Em sáb, 3 de nov de 2018 às 22:31, Bruno Visnadi 
>  escreveu:
>>
>> Não entendi a pergunta - o que é uma excursão?
>>
>> Em sáb, 3 de nov de 2018 às 22:18, Jardiel Cunha  
>> escreveu:
>>>
>>> Olá!
>>>
>>>
>>> Estou trabalhando em um projeto e um problema está me tirando o sono há 
>>> algum tempo. Meu trabalho é na área de engenharia de microondas. A solução 
>>> que eu encontrei até agora, acha soluções mas não satisfatórias... Não 
>>> precisam fazer o problema, queria apenas uma luz em que caminho seguir.
>>>
>>>
>>> [Problema] Dados N pontos em um círculo, estou querendo achar um ponto 
>>> dentro do círculo tal que: para qualquer valor de excursão em graus, eu 
>>> garanta que não existe outro ponto que englobe mais pontos no círculo do 
>>> que ele.
>>>
>>>
>>> Por exemplo: se eu der uma excursão de 80 graus... então eu quero um ponto 
>>> tal que englobe o maior número possível desses N pontos estando ele no 
>>> centro de um arco de 80 graus.
>>>
>>>
>>> Mais um exemplo: tenho 10 pontos. Queria um ponto x tal que ele será o 
>>> centro de todos os arcos com o maior número possível de pontos.
>>>
>>>
>>> Primeira pergunta: isso é possível???
>>>
>>> Segunda pergunta: como calcular este ponto?
>>>
>>>
>>> Abs
>>>
>>>
>>> Virus-free. www.avast.com
>>>
>>> --
>>> Esta mensagem foi verificada pelo sistema de antivírus e
>>> acredita-se estar livre de perigo.
>>
>>
>> --
>> Esta mensagem foi verificada pelo sistema de antivírus e
>> acredita-se estar livre de perigo.
>
>
> --
> Esta mensagem foi verificada pelo sistema de antivírus e
> acredita-se estar livre de perigo.



-- 
Bernardo Freitas Paulo da Costa

-- 
Esta 

Re: [obm-l] Problema 6 - IMO 2001

2018-11-09 Por tôpico Claudio Buffara
Ou olhe aqui: https://mks.mff.cuni.cz/kalva/imo/isoln/isoln016.html

On Fri, Nov 9, 2018 at 12:11 AM Bruno Visnadi 
wrote:

> Não entendi. Se a, b, c e d são inteiros, ac e bd certamente são racionais.
>
> Em qui, 8 de nov de 2018 às 22:27, Jeferson Almir <
> jefersonram...@gmail.com> escreveu:
>
>> Pessoal peço ajuda  no problema :
>>
>> Sejam a, b , c , d inteiros e a > b > c > d > 0 .
>> Suponha que
>> ac + bd = ( b+ d + a - c )( b+ d -a + c )
>>
>> Mostre que ab + cd não é primo .
>>
>>
>> A minha ideia foi:
>>
>> Abrindo a relação de cima temos
>>
>> a^2 -ac + c^2 = b^2 + bd + d^2
>>
>> Então motivado pela ideia de usar geometria que um amigo falou fiz a
>> suposição que temos um quadrilátero de lados a, d,b e c respectivamente e
>>  nessa ultima relação usando lei dos cossenos teríamos A = 60° e C = 120°
>> concluindo então que ABCD é inscritível . Aplicando Ptolomeu temos que ACxBD=
>> ab + cd e usando desigualdade triangular podemos afirmar que AC e BD não
>> podem ser 1 . Mas ainda tem a possibilidade AC e BD  serem racionais !!
>> Como provar que não podem ser ???
>>
>> --
>> Esta mensagem foi verificada pelo sistema de antivírus e
>> acredita-se estar livre de perigo.
>
>
> --
> Esta mensagem foi verificada pelo sistema de antivírus e
> acredita-se estar livre de perigo.

-- 
Esta mensagem foi verificada pelo sistema de antiv�rus e
 acredita-se estar livre de perigo.



Re: [obm-l] Problema 6 - IMO 2001

2018-11-08 Por tôpico Bruno Visnadi
Não entendi. Se a, b, c e d são inteiros, ac e bd certamente são racionais.

Em qui, 8 de nov de 2018 às 22:27, Jeferson Almir 
escreveu:

> Pessoal peço ajuda  no problema :
>
> Sejam a, b , c , d inteiros e a > b > c > d > 0 .
> Suponha que
> ac + bd = ( b+ d + a - c )( b+ d -a + c )
>
> Mostre que ab + cd não é primo .
>
>
> A minha ideia foi:
>
> Abrindo a relação de cima temos
>
> a^2 -ac + c^2 = b^2 + bd + d^2
>
> Então motivado pela ideia de usar geometria que um amigo falou fiz a
> suposição que temos um quadrilátero de lados a, d,b e c respectivamente e
>  nessa ultima relação usando lei dos cossenos teríamos A = 60° e C = 120°
> concluindo então que ABCD é inscritível . Aplicando Ptolomeu temos que ACxBD=
> ab + cd e usando desigualdade triangular podemos afirmar que AC e BD não
> podem ser 1 . Mas ainda tem a possibilidade AC e BD  serem racionais !!
> Como provar que não podem ser ???
>
> --
> Esta mensagem foi verificada pelo sistema de antivírus e
> acredita-se estar livre de perigo.

-- 
Esta mensagem foi verificada pelo sistema de antiv�rus e
 acredita-se estar livre de perigo.



Re: [obm-l] Problema de Trigonometria

2018-11-07 Por tôpico Luiz Antonio Rodrigues
Olá, Ralph!
Bom dia!
Muito obrigado pela ajuda!
Agora o problema faz sentido!
Um abraço!
Luiz


On Tue, Nov 6, 2018, 10:45 PM Ralph Teixeira  Eles disseram que a expressão eh uma identidade **em x**. Abrindo a
> expressão da direita e organizando, o que foi dado eh que:
> sinx+2cosx=(Asiny)sinx+(Acosy)cosx vale para todo x real.
>
> Como A e y sao NUMEROS (nao dependem de x), o unico jeito de isso
> acontecer eh se os coeficientes de sinx e cosx de um lado coincidirem com
> os coeficientes do outro lado, respectivamente (se você ainda não acredita
> nisso, explico no "Mais Embaixo"). Então devemos ter:
> Asiny=1
> Acosy=2
> Elevando ao quadrado e somando, vem A^2=5, ou seja, A=raiz(5) (pois eles
> falaram que A>0).
>
> Abraco, Ralph.
>
> M.E.: a afirmacao que eu fiz equivale a dizer que sinx e cosx sao funcoes
> linearmente independentes... em suma, estou dizendo que Bsinx+Ccosx=0 (para
> todo x) se, e somente se, B=C=0. De fato, se B fosse nao nulo, teriamos
> rearrumando que tanx=-C/B PARA TODO x REAL!! Isso eh claramente absurdo,
> tanx nao eh uma funcao constante que nao depende de x! Entao a conclusao eh
> que B=0, e portanto Ccosx=0 para todo x real, e dali C=0 tambem.
>
> Para conectar o problema original com este papo do paragrafo anterior,
> pegue aquela identidade em x em cima e escreva-a assim:
> (Asiny-2) sinx + (Acosy-2) cosx = 0
> Como os caras entre parenteses sao numeros, nao dependem de x, voce pode
> chama-los de B e C... e entao recai no que eu falei sobre linearmente
> independentes ali, ou seja, devemos ter B=C=0, e entao Asiny=1 e Acosy=2,
> como falamos antes.
>
> On Tue, Nov 6, 2018 at 9:59 PM Luiz Antonio Rodrigues <
> rodrigue...@gmail.com> wrote:
>
>> Olá, boa noite!
>> Transcrevi abaixo uma questão da Fuvest (SP). Já vi a resolução em vários
>> sites, mas achei tudo muito estranho...
>> Alguém pode me ajudar?
>> Muito obrigado e um abraço!
>> Luiz
>>
>> Sabe-se que existem números reais A e y, sendo A > 0, tais que:
>>
>> senx + 2cosx=Acos(x-y)
>>
>> para todo x real. Qual o valor de A?
>>
>> --
>> Esta mensagem foi verificada pelo sistema de antivírus e
>> acredita-se estar livre de perigo.
>
>
> --
> Esta mensagem foi verificada pelo sistema de antivírus e
> acredita-se estar livre de perigo.

-- 
Esta mensagem foi verificada pelo sistema de antiv�rus e
 acredita-se estar livre de perigo.



Re: [obm-l] Problema de Trigonometria

2018-11-06 Por tôpico Ralph Teixeira
Eles disseram que a expressão eh uma identidade **em x**. Abrindo a
expressão da direita e organizando, o que foi dado eh que:
sinx+2cosx=(Asiny)sinx+(Acosy)cosx vale para todo x real.

Como A e y sao NUMEROS (nao dependem de x), o unico jeito de isso acontecer
eh se os coeficientes de sinx e cosx de um lado coincidirem com os
coeficientes do outro lado, respectivamente (se você ainda não acredita
nisso, explico no "Mais Embaixo"). Então devemos ter:
Asiny=1
Acosy=2
Elevando ao quadrado e somando, vem A^2=5, ou seja, A=raiz(5) (pois eles
falaram que A>0).

Abraco, Ralph.

M.E.: a afirmacao que eu fiz equivale a dizer que sinx e cosx sao funcoes
linearmente independentes... em suma, estou dizendo que Bsinx+Ccosx=0 (para
todo x) se, e somente se, B=C=0. De fato, se B fosse nao nulo, teriamos
rearrumando que tanx=-C/B PARA TODO x REAL!! Isso eh claramente absurdo,
tanx nao eh uma funcao constante que nao depende de x! Entao a conclusao eh
que B=0, e portanto Ccosx=0 para todo x real, e dali C=0 tambem.

Para conectar o problema original com este papo do paragrafo anterior,
pegue aquela identidade em x em cima e escreva-a assim:
(Asiny-2) sinx + (Acosy-2) cosx = 0
Como os caras entre parenteses sao numeros, nao dependem de x, voce pode
chama-los de B e C... e entao recai no que eu falei sobre linearmente
independentes ali, ou seja, devemos ter B=C=0, e entao Asiny=1 e Acosy=2,
como falamos antes.

On Tue, Nov 6, 2018 at 9:59 PM Luiz Antonio Rodrigues 
wrote:

> Olá, boa noite!
> Transcrevi abaixo uma questão da Fuvest (SP). Já vi a resolução em vários
> sites, mas achei tudo muito estranho...
> Alguém pode me ajudar?
> Muito obrigado e um abraço!
> Luiz
>
> Sabe-se que existem números reais A e y, sendo A > 0, tais que:
>
> senx + 2cosx=Acos(x-y)
>
> para todo x real. Qual o valor de A?
>
> --
> Esta mensagem foi verificada pelo sistema de antivírus e
> acredita-se estar livre de perigo.

-- 
Esta mensagem foi verificada pelo sistema de antiv�rus e
 acredita-se estar livre de perigo.



[obm-l] Re: [obm-l] Re: [obm-l] [Problema] Achar o mínimo do valor absoluto de uma soma complexa

2018-11-05 Por tôpico Pedro José
Boa tarde!

Engano P4 e não Pe é o que engloba mais pontos.
E temos que somar 1 a ca engloba, pois esqueci de contar o próprio ponto.
Mas não influencia para o que englobe o máximo.

Saudações,
PJMS

Em seg, 5 de nov de 2018 às 16:41, Pedro José 
escreveu:

> Boa tarde!
> Se entendi o que você quer, não entendi qual a relação com o mínimo de uma
> soma complexa?
> Para resolver o problema que você propõe, entendi:
> (i) a excursão como a geração de um setor circular, a partir de um ponto
> inicial, essa incursão tem dois sentidos, trigonométrico ou horário.
> (ii) Englobar um ponto significa que o ponto pertença ao setor circular,
> tem que saber se incluem-se os pontos de borda ou não, como não há
> restrição vou considerar que sim.
> (iii) Estou supondo que seu universo é plano.
> Minha sugestão é defina o conjunto de pontos em coordenadas polares.
> Defina a variável excursão, e dê a ela um sinal para definir o sentido,
> Faça um programa.
> Definir "arrays" dos pontos (caso não estejam em coordenadas polares, tem
> que fazer uma sub-rotina para transformar as coordenadas em polares) Mod(I)
> e Teta(I)
> Defina um array de contagens
> Definir uma rotina para contar o número de pontos. N
> Defina uma sub-rotina Achapontos para determinar o índice Imax, cujo
> Engloba(I) seja máximo.
> ! Comentário: Atentar que podem retornar mais do que um índice. Portanto
> deve-se definir um array Pontonotável e uma variável de contagem Nmax.
> E.g., se tiverem três pontos que englobem o número máximo de pontos, deve
> retornar: Array contagem, com os valores dos índices dos pontos que têm o
> máximo de Engloba, nas três primeiras posições e o Valor Nmax=3.
> Aplique a sub-rotina de contagem no array de pontos e retorne com N.
> Entre com o valor de excursão
> Faça de I=1 a N
> Tetamax= max (teta(I);teta(i)+excursão)
> Tetamin=min(teta(I);teta(i)+excursão)
> Engloba(I)=0
> Faça de J=1 a N
> Se (teta(J)<=tetamax e teta(J)>=tetamin e mod(J)<=mod(I).
> ! comentário: A relação engloba será reflexiva. Todo ponto engloba si
> próprio.Caso não se aceite a borda é só tirar os iguais da lógica acima.
> Engloba(I) = Engloba(I)+1
> Fim SE;
> Fim Faça
> Fim faça
> Aplica Sub-rotina acha pontos.
> Salva temos o máximo de pontos englobados para uma excurção de [excursão]
> para [Nmax] pontos englobando [engloba(Nmax)]
> São eles:
> Faça de I=1 até N
> Pontonotável(I)
> Fim faça.
> !se tiver interesse salva todo array Engloba.
> FIM.
>
> Porém para qualquer setor existir um ponto que seja sempre o que englobe
> mais pontos, creio que vá depender da nuvem, e.g.
>
> P1= (10,40)
> P2= ( 6,42)
> P3= (9,90)
> P4= (8,100)
> P5= (7,107)
> P6= (7,5; 108)
> Teremos para uma excursão de + 5 graus:
> Engloba (1) = 1;Engloba (2) = 0; Engloba (3) = 0; Engloba (4) = 0 ,
> Engloba (5) = 0 e Engloba (6) = 0
> P1 é o que engloba mais pontos.
> Para uma excursão de +10 graus:
> Engloba (1) = 1;Engloba (2) = 0; Engloba (3) = 1; Engloba (4) = 2 e
> Engloba (5) = 0 e Engloba (6) = 0
> P3 é o que engloba mais pontos.
>
> Espero ter compreendido o proposto e ajudado.
> Mas o que tem haver com soma de complexos, módulo mínimo???
>
> Saudações,
> PJMS
>
> Em sáb, 3 de nov de 2018 às 22:31, Bruno Visnadi <
> brunovisnadida...@gmail.com> escreveu:
>
>> Não entendi a pergunta - o que é uma excursão?
>>
>> Em sáb, 3 de nov de 2018 às 22:18, Jardiel Cunha 
>> escreveu:
>>
>>> Olá!
>>>
>>>
>>> Estou trabalhando em um projeto e um problema está me tirando o sono há
>>> algum tempo. Meu trabalho é na área de engenharia de microondas. A solução
>>> que eu encontrei até agora, acha soluções mas não satisfatórias... Não
>>> precisam fazer o problema, queria apenas uma luz em que caminho seguir.
>>>
>>>
>>> [Problema] Dados N pontos em um círculo, estou querendo achar um ponto
>>> dentro do círculo tal que: para qualquer valor de excursão em graus, eu
>>> garanta que não existe outro ponto que englobe mais pontos no círculo do
>>> que ele.
>>>
>>>
>>> Por exemplo: se eu der uma excursão de 80 graus... então eu quero um
>>> ponto tal que englobe o maior número possível desses N pontos estando ele
>>> no centro de um arco de 80 graus.
>>>
>>>
>>> Mais um exemplo: tenho 10 pontos. Queria um ponto x tal que ele será o
>>> centro de todos os arcos com o maior número possível de pontos.
>>>
>>>
>>> Primeira pergunta: isso é possível???
>>>
>>> Segunda pergunta: como calcular este ponto?
>>>
>>>
>>> Abs
>>>
>>>
>>>
>>> 
>>>  Virus-free.
>>> www.avast.com
>>> 
>>> <#m_-6604935247907387060_m_3082452599431825113_m_2503312673449891629_DAB4FAD8-2DD7-40BB-A1B8-4E2AA1F9FDF2>
>>>
>>> --
>>> Esta mensagem foi verificada pelo sistema de antivírus e
>>> acredita-se estar livre de perigo.
>>>
>>
>> --
>> Esta mensagem foi verificada pelo sistema de antivírus e
>> 

[obm-l] Re: [obm-l] Re: [obm-l] [Problema] Achar o mínimo do valor absoluto de uma soma complexa

2018-11-05 Por tôpico Pedro José
Boa tarde!
Se entendi o que você quer, não entendi qual a relação com o mínimo de uma
soma complexa?
Para resolver o problema que você propõe, entendi:
(i) a excursão como a geração de um setor circular, a partir de um ponto
inicial, essa incursão tem dois sentidos, trigonométrico ou horário.
(ii) Englobar um ponto significa que o ponto pertença ao setor circular,
tem que saber se incluem-se os pontos de borda ou não, como não há
restrição vou considerar que sim.
(iii) Estou supondo que seu universo é plano.
Minha sugestão é defina o conjunto de pontos em coordenadas polares.
Defina a variável excursão, e dê a ela um sinal para definir o sentido,
Faça um programa.
Definir "arrays" dos pontos (caso não estejam em coordenadas polares, tem
que fazer uma sub-rotina para transformar as coordenadas em polares) Mod(I)
e Teta(I)
Defina um array de contagens
Definir uma rotina para contar o número de pontos. N
Defina uma sub-rotina Achapontos para determinar o índice Imax, cujo
Engloba(I) seja máximo.
! Comentário: Atentar que podem retornar mais do que um índice. Portanto
deve-se definir um array Pontonotável e uma variável de contagem Nmax.
E.g., se tiverem três pontos que englobem o número máximo de pontos, deve
retornar: Array contagem, com os valores dos índices dos pontos que têm o
máximo de Engloba, nas três primeiras posições e o Valor Nmax=3.
Aplique a sub-rotina de contagem no array de pontos e retorne com N.
Entre com o valor de excursão
Faça de I=1 a N
Tetamax= max (teta(I);teta(i)+excursão)
Tetamin=min(teta(I);teta(i)+excursão)
Engloba(I)=0
Faça de J=1 a N
Se (teta(J)<=tetamax e teta(J)>=tetamin e mod(J)<=mod(I).
! comentário: A relação engloba será reflexiva. Todo ponto engloba si
próprio.Caso não se aceite a borda é só tirar os iguais da lógica acima.
Engloba(I) = Engloba(I)+1
Fim SE;
Fim Faça
Fim faça
Aplica Sub-rotina acha pontos.
Salva temos o máximo de pontos englobados para uma excurção de [excursão]
para [Nmax] pontos englobando [engloba(Nmax)]
São eles:
Faça de I=1 até N
Pontonotável(I)
Fim faça.
!se tiver interesse salva todo array Engloba.
FIM.

Porém para qualquer setor existir um ponto que seja sempre o que englobe
mais pontos, creio que vá depender da nuvem, e.g.

P1= (10,40)
P2= ( 6,42)
P3= (9,90)
P4= (8,100)
P5= (7,107)
P6= (7,5; 108)
Teremos para uma excursão de + 5 graus:
Engloba (1) = 1;Engloba (2) = 0; Engloba (3) = 0; Engloba (4) = 0 , Engloba
(5) = 0 e Engloba (6) = 0
P1 é o que engloba mais pontos.
Para uma excursão de +10 graus:
Engloba (1) = 1;Engloba (2) = 0; Engloba (3) = 1; Engloba (4) = 2 e Engloba
(5) = 0 e Engloba (6) = 0
P3 é o que engloba mais pontos.

Espero ter compreendido o proposto e ajudado.
Mas o que tem haver com soma de complexos, módulo mínimo???

Saudações,
PJMS

Em sáb, 3 de nov de 2018 às 22:31, Bruno Visnadi <
brunovisnadida...@gmail.com> escreveu:

> Não entendi a pergunta - o que é uma excursão?
>
> Em sáb, 3 de nov de 2018 às 22:18, Jardiel Cunha 
> escreveu:
>
>> Olá!
>>
>>
>> Estou trabalhando em um projeto e um problema está me tirando o sono há
>> algum tempo. Meu trabalho é na área de engenharia de microondas. A solução
>> que eu encontrei até agora, acha soluções mas não satisfatórias... Não
>> precisam fazer o problema, queria apenas uma luz em que caminho seguir.
>>
>>
>> [Problema] Dados N pontos em um círculo, estou querendo achar um ponto
>> dentro do círculo tal que: para qualquer valor de excursão em graus, eu
>> garanta que não existe outro ponto que englobe mais pontos no círculo do
>> que ele.
>>
>>
>> Por exemplo: se eu der uma excursão de 80 graus... então eu quero um
>> ponto tal que englobe o maior número possível desses N pontos estando ele
>> no centro de um arco de 80 graus.
>>
>>
>> Mais um exemplo: tenho 10 pontos. Queria um ponto x tal que ele será o
>> centro de todos os arcos com o maior número possível de pontos.
>>
>>
>> Primeira pergunta: isso é possível???
>>
>> Segunda pergunta: como calcular este ponto?
>>
>>
>> Abs
>>
>>
>>
>> 
>>  Virus-free.
>> www.avast.com
>> 
>> <#m_3082452599431825113_m_2503312673449891629_DAB4FAD8-2DD7-40BB-A1B8-4E2AA1F9FDF2>
>>
>> --
>> Esta mensagem foi verificada pelo sistema de antivírus e
>> acredita-se estar livre de perigo.
>>
>
> --
> Esta mensagem foi verificada pelo sistema de antivírus e
> acredita-se estar livre de perigo.

-- 
Esta mensagem foi verificada pelo sistema de antiv�rus e
 acredita-se estar livre de perigo.



[obm-l] Re: [obm-l] [Problema] Achar o mínimo do valor absoluto de uma soma complexa

2018-11-03 Por tôpico Bruno Visnadi
Não entendi a pergunta - o que é uma excursão?

Em sáb, 3 de nov de 2018 às 22:18, Jardiel Cunha 
escreveu:

> Olá!
>
>
> Estou trabalhando em um projeto e um problema está me tirando o sono há
> algum tempo. Meu trabalho é na área de engenharia de microondas. A solução
> que eu encontrei até agora, acha soluções mas não satisfatórias... Não
> precisam fazer o problema, queria apenas uma luz em que caminho seguir.
>
>
> [Problema] Dados N pontos em um círculo, estou querendo achar um ponto
> dentro do círculo tal que: para qualquer valor de excursão em graus, eu
> garanta que não existe outro ponto que englobe mais pontos no círculo do
> que ele.
>
>
> Por exemplo: se eu der uma excursão de 80 graus... então eu quero um ponto
> tal que englobe o maior número possível desses N pontos estando ele no
> centro de um arco de 80 graus.
>
>
> Mais um exemplo: tenho 10 pontos. Queria um ponto x tal que ele será o
> centro de todos os arcos com o maior número possível de pontos.
>
>
> Primeira pergunta: isso é possível???
>
> Segunda pergunta: como calcular este ponto?
>
>
> Abs
>
>
>
> 
>  Virus-free.
> www.avast.com
> 
> <#m_2503312673449891629_DAB4FAD8-2DD7-40BB-A1B8-4E2AA1F9FDF2>
>
> --
> Esta mensagem foi verificada pelo sistema de antivírus e
> acredita-se estar livre de perigo.
>

-- 
Esta mensagem foi verificada pelo sistema de antiv�rus e
 acredita-se estar livre de perigo.



Re: [obm-l] problema de contagem

2018-11-03 Por tôpico Anderson Torres
Em ter, 30 de out de 2018 às 15:41, Luís Lopes  escreveu:
>
> Sauda,c~oes,
>
>
> Construir um triângulo dados três quaisquer dos seguintes elementos:
>
>
> A,B,C - ângulos
>
> a,b,c - lados
>
> h_a,h_b,h_c - alturas
>
> m_a,m_b,m_c - medianas
>
> d_a,d_b,d_c - bissetrizes internas
>
> e_a,e_b,e_c - bissetrizes externas
>
> R - circunraio
> r - inraio
>
> r_a,r_b,r_c - exraios
>
>
> Quantos problemas diferentes podem ser criados ?
>
>
> Problemas do tipo (A,B,C) e (A,a,R) são considerados. Assim como
>
> (A,B,a) e (A,B,c). Mas (A,B,a) e (A,B,b) contam como um só.
>

A ideia é qual? Construção supondo ter um resolvedor perfeito? Por
exemplo, se para obter um triangulo com dados X, Y, Z for necessário
calcular a raiz quadrada de pi, isso conta? Ou só conta se for régua e
compasso?

>
> Abs,
>
> Luís
>
>
>
>
>
>
>
>
>
> --
> Esta mensagem foi verificada pelo sistema de antivírus e
> acredita-se estar livre de perigo.

-- 
Esta mensagem foi verificada pelo sistema de antiv�rus e
 acredita-se estar livre de perigo.


=
Instru��es para entrar na lista, sair da lista e usar a lista em
http://www.mat.puc-rio.br/~obmlistas/obm-l.html
=


[obm-l] Re: [obm-l] Problema Olímpico - Nível 1 - Segunda fase - 2011

2018-10-16 Por tôpico Pedro José
Boa tarde!

Desculpe-me, acabei não prestando atenção no seu questionamento:"*Inclusive,
como está o desenho, são 47 pessoas respondendo "sim", e não 48 como
hipótese inicial. Concordam**?*"

Discordo pois não é uma fila é um círculo e o V76, estará a direita de A1,
então teremos de A1 a A24 respondendo sim e de V1 a V24 respondendo sim.
São 48 e não 47.

A1V1A2V2...V23A24V24V25V26V76

Saudações,
PJMS

Em ter, 16 de out de 2018 às 15:36, Pedro José 
escreveu:

> Boa tarde!
> Não vejo erro na solução do sítio da OBM.
>
> 1) Entendi sua referência a início, como o primeiro entrevistado.
> 2) Realmente, não há nenhuma diferenciação entre se começar com azul ou
> por vermelho. Não há restrição para que as respostas "SIM" sejam
> consecutivas. Portanto, se você pegar esse mesmo "círculo" (seria melhor
> circunferência) e iniciar a pergunta por qualquer um do círculo o número de
> respostas "SIM" permanece inalterada, já que não depende do início da
> entrevista, mas sim da tribo do vizinho.
> 3) Não é afirmado na solução do problema que devam existir exatamente, mas
> sim pelo menos 24 pessoas da tribo azul e 24 da tribo vermelha. Embora, a
> solução poderia apontar que seriam necessárias 24 pessoas de uma tribo e 25
> da outra sentadas alternadamente. E como queremos maximizar a vermelha,
> seriam 25 da tribo vermelha e 24 da azul.
>
> A solução poderia ser mais clara, porém, não há erro na solução.
>
> Saudações,
> PJMS
>
>
> Em ter, 16 de out de 2018 às 12:59,  escreveu:
>
>> *Estudando o problema a seguir:*
>>
>> Duas tribos vivem numa ilha. Os da tribo azul só dizem a verdade e os da
>> vermelha, só mentira. Um dia, 100 pessoas da ilha se reuniram num círculo e
>> um repórter se dirigiu a cada uma delas, com a pergunta: “O seu vizinho à
>> direita é um mentiroso?”. Terminada a pesquisa, verificou-se que 48 pessoas
>> responderam “sim”. No máximo, quantas pessoas da tribo vermelha poderiam
>> estar no círculo?
>>
>> *Consta a seguinte solução no site da OBM:*
>>
>> Observe que se uma pessoa responde “sim”, então esta pessoa e a da
>> direita não são da mesma tribo, mas se responder “não”, então ela e a
>> pessoa à sua direita são da mesma tribo. Assim, se 48 pessoas responderam
>> “sim”, então ao percorrer o círculo no sentido horário, observaremos 48
>> trocas de cor da tribo. Para que haja 48 trocas, devem haver pelo menos 24
>> pessoas da tribo azul e 24 da tribo vermelha dispostas alternadamente. Como
>> queremos o máximo de pessoas da tribo vermelha, então podemos colocar as
>> 100 – 24 – 24 = 52 pessoas restantes juntas num mesmo bloco vermelho, como
>> indicado a seguir:
>>
>> AVAVA ... VAV/VV ...VV.
>>
>> *Presunção da necessidade de retificação:*
>>
>> (...) Para que haja 48 trocas, devem haver 49 pessoas, como o problema
>> pede o máximo da tribo tribo vermelha, 25 serão vermelhos e 24 azuis.
>> Inicia-se e termina-se por um da tribo vermelha, e insere-se 51 também
>> vermelhos empós. Daí sim, 25+51 = 76.
>> Da forma como está proposta a solução no site, a resposta está correta,
>> porém, nem tanto a solução, já que não há uma diferenciação entre se
>> iniciar por azul ou por vermelho. Inclusive, como está o desenho, são 47
>> pessoas respondendo "sim", e não 48 como hipótese inicial. Concordam?
>>
>> --
>> Esta mensagem foi verificada pelo sistema de antivírus e
>> acredita-se estar livre de perigo.
>>
>

-- 
Esta mensagem foi verificada pelo sistema de antiv�rus e
 acredita-se estar livre de perigo.



[obm-l] Re: [obm-l] Problema Olímpico - Nível 1 - Segunda fase - 2011

2018-10-16 Por tôpico Pedro José
Boa tarde!
Não vejo erro na solução do sítio da OBM.

1) Entendi sua referência a início, como o primeiro entrevistado.
2) Realmente, não há nenhuma diferenciação entre se começar com azul ou por
vermelho. Não há restrição para que as respostas "SIM" sejam consecutivas.
Portanto, se você pegar esse mesmo "círculo" (seria melhor circunferência)
e iniciar a pergunta por qualquer um do círculo o número de respostas "SIM"
permanece inalterada, já que não depende do início da entrevista, mas sim
da tribo do vizinho.
3) Não é afirmado na solução do problema que devam existir exatamente, mas
sim pelo menos 24 pessoas da tribo azul e 24 da tribo vermelha. Embora, a
solução poderia apontar que seriam necessárias 24 pessoas de uma tribo e 25
da outra sentadas alternadamente. E como queremos maximizar a vermelha,
seriam 25 da tribo vermelha e 24 da azul.

A solução poderia ser mais clara, porém, não há erro na solução.

Saudações,
PJMS


Em ter, 16 de out de 2018 às 12:59,  escreveu:

> *Estudando o problema a seguir:*
>
> Duas tribos vivem numa ilha. Os da tribo azul só dizem a verdade e os da
> vermelha, só mentira. Um dia, 100 pessoas da ilha se reuniram num círculo e
> um repórter se dirigiu a cada uma delas, com a pergunta: “O seu vizinho à
> direita é um mentiroso?”. Terminada a pesquisa, verificou-se que 48 pessoas
> responderam “sim”. No máximo, quantas pessoas da tribo vermelha poderiam
> estar no círculo?
>
> *Consta a seguinte solução no site da OBM:*
>
> Observe que se uma pessoa responde “sim”, então esta pessoa e a da direita
> não são da mesma tribo, mas se responder “não”, então ela e a pessoa à sua
> direita são da mesma tribo. Assim, se 48 pessoas responderam “sim”, então
> ao percorrer o círculo no sentido horário, observaremos 48 trocas de cor da
> tribo. Para que haja 48 trocas, devem haver pelo menos 24 pessoas da tribo
> azul e 24 da tribo vermelha dispostas alternadamente. Como queremos o
> máximo de pessoas da tribo vermelha, então podemos colocar as 100 – 24 – 24
> = 52 pessoas restantes juntas num mesmo bloco vermelho, como indicado a
> seguir:
>
> AVAVA ... VAV/VV ...VV.
>
> *Presunção da necessidade de retificação:*
>
> (...) Para que haja 48 trocas, devem haver 49 pessoas, como o problema
> pede o máximo da tribo tribo vermelha, 25 serão vermelhos e 24 azuis.
> Inicia-se e termina-se por um da tribo vermelha, e insere-se 51 também
> vermelhos empós. Daí sim, 25+51 = 76.
> Da forma como está proposta a solução no site, a resposta está correta,
> porém, nem tanto a solução, já que não há uma diferenciação entre se
> iniciar por azul ou por vermelho. Inclusive, como está o desenho, são 47
> pessoas respondendo "sim", e não 48 como hipótese inicial. Concordam?
>
> --
> Esta mensagem foi verificada pelo sistema de antivírus e
> acredita-se estar livre de perigo.
>

-- 
Esta mensagem foi verificada pelo sistema de antiv�rus e
 acredita-se estar livre de perigo.



Re: [obm-l] Problema da OBM 2017

2018-05-10 Por tôpico Kevin Felipe Kuhl Oliveira
Boa tarde, Vanderlei

Bom, o que pensei nessa letra é o seguinte:
Temos que encontrar o elemento que ocupa a posição 2017 (no conjunto 
crescentemente ordenado dos números que podemos escrever na terra dos Impas). 
Para isso, podemos pensar qual o número mínimo de algarismos que esse número 
procurado deve ter.
Usando combinatória percebemos que:
*Com 1 algarismo -> 5 números
*Com 2 algarismos -> 25 números
*Com 3 algarismos -> 125 números
*Com 4 algarismos -> 625 números
*Com 5 algarismos -> 3125 números
Logo, o número procurado terá 5 algarismos (concorda?). Assim, basta 
encontrarmos esses algarismos.
O último número que conseguiremos escrever com 5 algarismos é o 9 e o 
primeiro número é o 1. Assim, vamos analisar qual número ocupa a posição 
2017. Bom, com até 4 algarismos, ja conseguimos escrever 780 números (certo?), 
então estamos procurando o número de posição 1237 depois do 1. Se o 
primeiro digito for 1, teremos 625 possibilidades. Vamos então para o segundo 
digito (3), teremos mais 625 possibilidades. Mas atenção (625 + 625 > 1237), 
então concluímos que o nosso número começa com 3. Agora se o segundo digito for 
1, teremos 125 possibilidades, assim como se for 3, 5, 7 e 9. Novamente, 
devemos parar antes que a soma de todas as possibilidades seja maior que 1237. 
Logo, teremos que o segundo digito é 9. Trabalhando de modo análogo para o 
terceiro digito, concluiremos que o mesmo é 7 e o quarto digito é 5. Finalmente 
estaremos com o número 3975_ e falta um número. O número 39751 ocupa a posição 
2016 portanto, o número 39753 ocupa a posição 2017 no conjunto analisado.
Veja se é um raciocínio coerente.

Abraços

Kevin Kühl

On 10 May 2018 16:44 -0300, Vanderlei Nemitz , wrote:
> Pessoal, gostaria de uma ajuda no item c dessa questão. Os dois primeiros 
> itens são tranquilos.
>
> Na Terra dos Impas, somente os algarismos ímpares são utilizados para contar 
> e escrever números. Assim, em vez dos números 1, 2, 3, 4, 5, 6, 7, 8, 9, 10, 
> 11, 12, . . . os Impas tem os números correspondentes 1, 3, 5, 7, 9, 11, 13, 
> 15, 17, 19, 31, 33, . . . (note que os números dos Impas têm somente 
> algarismos ímpares). Por exemplo, se uma criança tem 11 anos, os Impas diriam 
> que ela tem 31 anos.
> a) Como os Impas escrevem o nosso numero 20?
> b) Numa escola desse lugar, a professora escreveu no quadro-negro a continha 
> de multiplicar abaixo. Se você fosse um aluno Impa, o que escreveria como 
> resultado?
> 13 × 5
> c) Escreva, na linguagem dos Impas, o numero que na nossa representação ao 
> decimal é escrito como 2017.
>
> Obrigado!
>
> --
> Esta mensagem foi verificada pelo sistema de antiv�rus e
> acredita-se estar livre de perigo.

-- 
Esta mensagem foi verificada pelo sistema de antiv�rus e
 acredita-se estar livre de perigo.



Re: [obm-l] Re: [obm-l] Re: [obm-l] Re: [obm-l] Problema de minimização

2018-03-12 Por tôpico Claudio Buffara
A propriedade de reflexão na elipse é outra consequência interessante da 
desigualdade triangular e, mais precisamente, da solução do problema de achar o 
caminho mais curto entre os pontos A e B tocando uma reta dada (A e B estando 
num mesmo semiplano determinado pela reta).

No fim, o caminho obedece: ângulo de incidência = ângulo de reflexão.

Isso está bem explicado no livro What is Mathematics? de Courant e Robbins, no 
capítulo que trata de máximos e mínimos.

Assim, na elipse com focos A e B contendo o ponto O, uma reta r por O é 
tangente a elipse se e somente se AO e BO fazem o mesmo ângulo com r se e 
somente se a bissetriz de AOB é perpendicular a r.

O problema do Ralph se reduz a achar a tangente comum às duas elipses. E, pela 
propriedade de reflexão, a bissetriz comum dos ângulos AOB e COD (que são 
opostos pelo vértice) é perpendicular à tangente comum. 

Abs,
Claudio.

Enviado do meu iPhone

Em 12 de mar de 2018, à(s) 21:41, Anderson Torres 
 escreveu:

> Em 11 de março de 2018 22:37, Ralph Teixeira  escreveu:
>> ...e portanto a elipse de focos A e B passando por O tem que ser tangente aa
>> elipse de focos C e D passando por O Fica como exercicio pensar o que
>> uma coisa tem a ver com a outra.
> 
> Heuristicamente, eu chutaria que se tais elipses se cortassem em mais
> de um ponto,
> existiria outro ponto no interior das regiões que teria soma menor.
> Talvez o ponto médio...
> 
>> 
>> (O que podia ser visto de outras formas, diga-se de passagem, se voce sabe
>> que a normal a tal elipse eh a bissetriz de AOB).
>> 
>> Abraco, Ralph.
>> 
>> 
>> 
>> 2018-03-11 20:29 GMT-03:00 Claudio Buffara :
>>> 
>>> É isso aí!
>>> Uma aplicação simples mas elegante da desigualdade triangular.
>>> E o ponto O não parece ser tão difícil de conjecturar. Afinal, o ponto de
>>> intersecção das diagonais talvez seja o “ponto notável”
>>> mais óbvio de um quadrilátero (certamente é o mais fácil de construir -
>>> duas aplicações da régua e nenhuma do compasso).
>>> 
>>> E quando, digamos, o vértice D tende ao vértice C do quadrilátero, o 
>>> ponto
>>> O de intersecção das diagonais não tende ao ponto de Fermat do 
>>> triângulo ABC
>>> pois este é o que tem a menor soma das distâncias aos vértices enquanto 
>>> que
>>> O, no limite, minimiza a soma PA + PB + 2PC.
>>> 
>>> Enviado do meu iPhone
>>> 
>>> Em 11 de mar de 2018, Ã (s) 17:20, Douglas Oliveira de Lima
>>>  escreveu:
>>> 
>>> Seja o quadrilátero ABCD cujas diagonais são AC e BD, e O o ponto de
>>> intersecção das diagonais.Â
>>> Seja também um ponto P em seu interior e as distâncias PA, PB, PC, PD,
>>> temos por desigualdade triângularÂ
>>> que PA+PC>=AC e PB+PD>=BD. Claramente vemos que o ponto P coincide com o
>>> ponto O quando a soma das diagonaisÂ
>>> coincide com a igualdade. Desta forma o ponto procurado é o encontro das
>>> diagonais.
>>> 
>>> 
>>> Forte abraço.
>>> Douglas Oliveira.Â
>>> 
>>> Em 10 de março de 2018 21:07, Claudio Buffara 
>>> escreveu:
 
 Aqui vai um bonitinho que eu nunca tinha visto:
 
 Dado um quadrilátero convexo, determine o ponto cuja soma das
 distâncias aos vértices do quadrilátero é mínima.
 
 Interessante que quando a distância entre dois vértices adjacentes
 dados tende a zero (e o quadrilátero “tende†a um 
 triângulo), o ponto
 de mínimo não parece tender ao ponto de Fermat do triângulo 
 (exceto
 quando o triângulo tem um ângulo >= 120 graus.
 
 Abs,
 Claudio.
 
 Enviado do meu iPhone
 --
 Esta mensagem foi verificada pelo sistema de antivírus e
 Â acredita-se estar livre de perigo.
 
 
 =
 Instru�ões para entrar na lista, sair da lista e usar a lista em
 http://www.mat.puc-rio.br/~obmlistas/obm-l.html
 =
>>> 
>>> 
>>> 
>>> --
>>> Esta mensagem foi verificada pelo sistema de antivírus e
>>> acredita-se estar livre de perigo.
>>> 
>>> 
>>> --
>>> Esta mensagem foi verificada pelo sistema de antivírus e
>>> acredita-se estar livre de perigo.
>> 
>> 
>> 
>> --
>> Esta mensagem foi verificada pelo sistema de antivírus e
>> acredita-se estar livre de perigo.
> 
> -- 
> Esta mensagem foi verificada pelo sistema de antivírus e
> acredita-se estar livre de perigo.
> 
> 
> =
> Instruções para entrar na lista, sair da lista e usar a lista em
> http://www.mat.puc-rio.br/~obmlistas/obm-l.html
> =

-- 
Esta mensagem foi verificada pelo sistema de antiv�rus e
 

[obm-l] Re: [obm-l] Re: [obm-l] Re: [obm-l] Problema de minimização

2018-03-12 Por tôpico Anderson Torres
Em 11 de março de 2018 22:37, Ralph Teixeira  escreveu:
> ...e portanto a elipse de focos A e B passando por O tem que ser tangente aa
> elipse de focos C e D passando por O Fica como exercicio pensar o que
> uma coisa tem a ver com a outra.

Heuristicamente, eu chutaria que se tais elipses se cortassem em mais
de um ponto,
existiria outro ponto no interior das regiões que teria soma menor.
Talvez o ponto médio...

>
> (O que podia ser visto de outras formas, diga-se de passagem, se voce sabe
> que a normal a tal elipse eh a bissetriz de AOB).
>
> Abraco, Ralph.
>
>
>
> 2018-03-11 20:29 GMT-03:00 Claudio Buffara :
>>
>> É isso aí!
>> Uma aplicação simples mas elegante da desigualdade triangular.
>> E o ponto O não parece ser tão difícil de conjecturar. Afinal, o ponto de
>> intersecção das diagonais talvez seja o “ponto notável”
>> mais óbvio de um quadrilátero (certamente é o mais fácil de construir -
>> duas aplicações da régua e nenhuma do compasso).
>>
>> E quando, digamos, o vértice D tende ao vértice C do quadrilátero, o ponto
>> O de intersecção das diagonais não tende ao ponto de Fermat do triângulo ABC
>> pois este é o que tem a menor soma das distâncias aos vértices enquanto que
>> O, no limite, minimiza a soma PA + PB + 2PC.
>>
>> Enviado do meu iPhone
>>
>> Em 11 de mar de 2018, à(s) 17:20, Douglas Oliveira de Lima
>>  escreveu:
>>
>> Seja o quadrilátero ABCD cujas diagonais são AC e BD, e O o ponto de
>> intersecção das diagonais.Â
>> Seja também um ponto P em seu interior e as distâncias PA, PB, PC, PD,
>> temos por desigualdade triângularÂ
>> que PA+PC>=AC e PB+PD>=BD. Claramente vemos que o ponto P coincide com o
>> ponto O quando a soma das diagonaisÂ
>> coincide com a igualdade. Desta forma o ponto procurado é o encontro das
>> diagonais.
>>
>>
>> Forte abraço.
>> Douglas Oliveira.Â
>>
>> Em 10 de março de 2018 21:07, Claudio Buffara 
>> escreveu:
>>>
>>> Aqui vai um bonitinho que eu nunca tinha visto:
>>>
>>> Dado um quadrilátero convexo, determine o ponto cuja soma das
>>> distâncias aos vértices do quadrilátero é mínima.
>>>
>>> Interessante que quando a distância entre dois vértices adjacentes
>>> dados tende a zero (e o quadrilátero “tende†a um triângulo), o ponto
>>> de mínimo não parece tender ao ponto de Fermat do triângulo (exceto
>>> quando o triângulo tem um ângulo >= 120 graus.
>>>
>>> Abs,
>>> Claudio.
>>>
>>> Enviado do meu iPhone
>>> --
>>> Esta mensagem foi verificada pelo sistema de antivírus e
>>> Â acredita-se estar livre de perigo.
>>>
>>>
>>> =
>>> Instru�ões para entrar na lista, sair da lista e usar a lista em
>>> http://www.mat.puc-rio.br/~obmlistas/obm-l.html
>>> =
>>
>>
>>
>> --
>> Esta mensagem foi verificada pelo sistema de antivírus e
>> acredita-se estar livre de perigo.
>>
>>
>> --
>> Esta mensagem foi verificada pelo sistema de antivírus e
>> acredita-se estar livre de perigo.
>
>
>
> --
> Esta mensagem foi verificada pelo sistema de antivírus e
> acredita-se estar livre de perigo.

-- 
Esta mensagem foi verificada pelo sistema de antiv�rus e
 acredita-se estar livre de perigo.


=
Instru��es para entrar na lista, sair da lista e usar a lista em
http://www.mat.puc-rio.br/~obmlistas/obm-l.html
=


[obm-l] Re: [obm-l] Re: [obm-l] Problema de minimização

2018-03-11 Por tôpico Ralph Teixeira
...e portanto a elipse de focos A e B passando por O tem que ser tangente
aa elipse de focos C e D passando por O Fica como exercicio pensar o
que uma coisa tem a ver com a outra.

(O que podia ser visto de outras formas, diga-se de passagem, se voce sabe
que a normal a tal elipse eh a bissetriz de AOB).

Abraco, Ralph.



2018-03-11 20:29 GMT-03:00 Claudio Buffara :

> É isso aí!
> Uma aplicação simples mas elegante da desigualdade triangular.
> E o ponto O não parece ser tão difícil de conjecturar. Afinal, o ponto de
> intersecção das diagonais talvez seja o “ponto notável”
> mais óbvio de um quadrilátero (certamente é o mais fácil de construir -
> duas aplicações da régua e nenhuma do compasso).
>
> E quando, digamos, o vértice D tende ao vértice C do quadrilátero, o ponto
> O de intersecção das diagonais não tende ao ponto de Fermat do triângulo
> ABC pois este é o que tem a menor soma das distâncias aos vértices enquanto
> que O, no limite, minimiza a soma PA + PB + 2PC.
>
> Enviado do meu iPhone
>
> Em 11 de mar de 2018, à(s) 17:20, Douglas Oliveira de Lima <
> profdouglaso.del...@gmail.com> escreveu:
>
> Seja o quadrilátero ABCD cujas diagonais são AC e BD, e O o ponto de
> intersecção das diagonais.Â
> Seja também um ponto P em seu interior e as distâncias PA, PB, PC, PD,
> temos por desigualdade triângularÂ
> que PA+PC>=AC e PB+PD>=BD. Claramente vemos que o ponto P coincide com o
> ponto O quando a soma das diagonaisÂ
> coincide com a igualdade. Desta forma o ponto procurado é o encontro das
> diagonais.
>
>
> Forte abraço.
> Douglas Oliveira.Â
>
> Em 10 de março de 2018 21:07, Claudio Buffara 
> escreveu:
>
>> Aqui vai um bonitinho que eu nunca tinha visto:
>>
>> Dado um quadrilátero convexo, determine o ponto cuja soma das
>> distâncias aos vértices do quadrilátero é mínima.
>>
>> Interessante que quando a distância entre dois vértices adjacentes
>> dados tende a zero (e o quadrilátero “tende†a um triângulo), o ponto
>> de mínimo não parece tender ao ponto de Fermat do triângulo (exceto
>> quando o triângulo tem um ângulo >= 120 graus.
>>
>> Abs,
>> Claudio.
>>
>> Enviado do meu iPhone
>> --
>> Esta mensagem foi verificada pelo sistema de antivírus e
>> Â acredita-se estar livre de perigo.
>>
>>
>> =
>> Instru�ões para entrar na lista, sair da lista e usar a lista em
>> http://www.mat.puc-rio.br/~obmlistas/obm-l.html
>> =
>>
>
>
> --
> Esta mensagem foi verificada pelo sistema de antivírus e
> acredita-se estar livre de perigo.
>
>
> --
> Esta mensagem foi verificada pelo sistema de antivírus e
> acredita-se estar livre de perigo.
>

-- 
Esta mensagem foi verificada pelo sistema de antiv�rus e
 acredita-se estar livre de perigo.



Re: [obm-l] Re: [obm-l] Problema de minimização

2018-03-11 Por tôpico Claudio Buffara
É isso aí!
Uma aplicação simples mas elegante da desigualdade triangular.
E o ponto O não parece ser tão difícil de conjecturar. Afinal, o ponto de 
intersecção das diagonais talvez seja o “ponto notável” 
mais óbvio de um quadrilátero (certamente é o mais fácil de construir - duas 
aplicações da régua e nenhuma do compasso).

E quando, digamos, o vértice D tende ao vértice C do quadrilátero, o ponto O de 
intersecção das diagonais não tende ao ponto de Fermat do triângulo ABC pois 
este é o que tem a menor soma das distâncias aos vértices enquanto que O, no 
limite, minimiza a soma PA + PB + 2PC.

Enviado do meu iPhone

Em 11 de mar de 2018, à(s) 17:20, Douglas Oliveira de Lima 
 escreveu:

> Seja o quadrilátero ABCD cujas diagonais são AC e BD, e O o ponto de 
> intersecção das diagonais. 
> Seja também um ponto P em seu interior e as distâncias PA, PB, PC, PD, 
> temos por desigualdade triângular 
> que PA+PC>=AC e PB+PD>=BD. Claramente vemos que o ponto P coincide com o 
> ponto O quando a soma das diagonais 
> coincide com a igualdade. Desta forma o ponto procurado é o encontro das 
> diagonais.
> 
> 
> Forte abraço.
> Douglas Oliveira. 
> 
> Em 10 de março de 2018 21:07, Claudio Buffara  
> escreveu:
>> Aqui vai um bonitinho que eu nunca tinha visto:
>> 
>> Dado um quadrilátero convexo, determine o ponto cuja soma das distâncias 
>> aos vértices do quadrilátero é mínima.
>> 
>> Interessante que quando a distância entre dois vértices adjacentes dados 
>> tende a zero (e o quadrilátero “tende” a um triângulo), o ponto de 
>> mínimo não parece tender ao ponto de Fermat do triângulo (exceto quando o 
>> triângulo tem um ângulo >= 120 graus.
>> 
>> Abs,
>> Claudio.
>> 
>> Enviado do meu iPhone
>> --
>> Esta mensagem foi verificada pelo sistema de antivírus e
>> Â acredita-se estar livre de perigo.
>> 
>> 
>> =
>> Instru�ões para entrar na lista, sair da lista e usar a lista em
>> http://www.mat.puc-rio.br/~obmlistas/obm-l.html
>> =
> 
> 
> -- 
> Esta mensagem foi verificada pelo sistema de antivírus e 
> acredita-se estar livre de perigo.

-- 
Esta mensagem foi verificada pelo sistema de antiv�rus e
 acredita-se estar livre de perigo.



[obm-l] Re: [obm-l] Problema de minimização

2018-03-11 Por tôpico Douglas Oliveira de Lima
Seja o quadrilátero ABCD cujas diagonais são AC e BD, e O o ponto de
intersecção das diagonais.
Seja também um ponto P em seu interior e as distâncias PA, PB, PC, PD,
temos por desigualdade triângular
que PA+PC>=AC e PB+PD>=BD. Claramente vemos que o ponto P coincide com o
ponto O quando a soma das diagonais
coincide com a igualdade. Desta forma o ponto procurado é o encontro das
diagonais.


Forte abraço.
Douglas Oliveira.

Em 10 de março de 2018 21:07, Claudio Buffara 
escreveu:

> Aqui vai um bonitinho que eu nunca tinha visto:
>
> Dado um quadrilátero convexo, determine o ponto cuja soma das distâncias
> aos vértices do quadrilátero é mínima.
>
> Interessante que quando a distância entre dois vértices adjacentes dados
> tende a zero (e o quadrilátero “tende” a um triângulo), o ponto de mínimo
> não parece tender ao ponto de Fermat do triângulo (exceto quando o
> triângulo tem um ângulo >= 120 graus.
>
> Abs,
> Claudio.
>
> Enviado do meu iPhone
> --
> Esta mensagem foi verificada pelo sistema de antivírus e
>  acredita-se estar livre de perigo.
>
>
> =
> Instru�ões para entrar na lista, sair da lista e usar a lista em
> http://www.mat.puc-rio.br/~obmlistas/obm-l.html
> =
>

-- 
Esta mensagem foi verificada pelo sistema de antiv�rus e
 acredita-se estar livre de perigo.



Re: [obm-l] Problema 2 da OBM U

2017-11-21 Por tôpico Anderson Torres
OPA! Tem um problema no meu problema!

Em 18 de novembro de 2017 16:48, Anderson Torres
 escreveu:
> Em 15 de novembro de 2017 15:01, Otávio Araújo
>  escreveu:
>> Alguém poderia me ajudar no problema 2 da segunda fase da obm u desse ano? O
>> enunciado é o seguinte:
>> "Fixados os inteiros positivos a e b, mostre que o conjunto dos divisores
>> primos dos termos da sequencia
>> an = a · 2017^n + b · 2016^n
>> é infinito."
>
> Não é difícil difícil, apenas precisa ter a ideia certa. Aqui eu fui
> um pouco mais formalista do que o necessário, mas a ideia é que se
> construirmos uma sequência dessas com poucos primos, eles explodem.
>
> Caso I: o conjunto {p_1,p_2,...} de todos os primos que aparecem em
> todas as fatorações de a_n é infinito. Neste caso, o problema acaba
> antes mesmo de começar.
>
> Caso II: o conjunto {p_1,p_2,...,p_k} de todos os primos que aparecem
> em todas as fatorações de a_n é finito. Vamos trabalhar apenas com
> esses primos, a não ser que seja dito algo contrário.
>
> Assim sendo, podemos fatorar cada a_n da seguinte forma:
>
> a_n = p_1^(e(1,n)) * p_2^(e(2,n)) * p_3^(e(3,n)) * ... * p_k^(e(k,n))
>
> A cada a_n, vamos associar a maior dessas potências. Por exemplo, 72 =
> 2^3 * 3^2 estaria associada a 3^2, pois 3^2 = 9 > 8 = 2^3.
>
> LEMA: Seja M_n = MDC(a*2017^n, b*2016^n). Então M_n é limitada.
>
> De fato, se tentarmos fatorar, os fatores de 2017 não aparecem em 2016
> e vice-versa, dado serem primos entre si. Logo os fatores de 2017 só
> podem aparecer em b, e os fatores de 2016 só podem aparecer em a.
> Portanto, M_n = MDC(a,2016^n) * MDC(b,2017^n) | ab, logo, limitado.
>
> Vamos dividir nossa sequência em fitas de tamanho (K+1):
>
> a_(0*k+0), a_(0*k+1),...,a_(0*k+k)
> a_(1*k+0), a_(1*k+1),...,a_(1*k+k)
> a_(2*k+0), a_(2*k+1),...,a_(2*k+k)
> a_(3*k+0), a_(3*k+1),...,a_(3*k+k)
> a_(4*k+0), a_(4*k+1),...,a_(4*k+k)
> a_(5*k+0), a_(5*k+1),...,a_(5*k+k)
> ... ... ...
>
> Se aplicarmos o princípio das gavetas em cada fita separadamente,
> podemos dizer que para cada fita existe um primo tal que potências
> desse primo aparecem (pelo menos) duas vezes entre os associados.
> Vamos então anotar este primo para cada faixa:
>
> a_(0*k+0), a_(0*k+1),...,a_(0*k+k)
> a_(1*k+0), a_(1*k+1),...,a_(1*k+k)
> a_(2*k+0), a_(2*k+1),...,a_(2*k+k)
> a_(3*k+0), a_(3*k+1),...,a_(3*k+k)
> a_(4*k+0), a_(4*k+1),...,a_(4*k+k)
> a_(5*k+0), a_(5*k+1),...,a_(5*k+k)
>
>
> Como o número de faixas é infinito e o número de possíveis primos
> associados é finito, então existe um primo tal que infinitas faixas
> tenham potências desse mesmo primo associadas.
>

Esta parte é falsa, pois eu não demonstrei que as fitas com o primo P
são consecutivas, aliás é capaz que elas não sejam.

Mas isso pouco abala a demonstração, pois em cada fita existirão dois
caras com o fator P, e a distância entre eles é limitada.

> Com esse raciocínio, nós demonstramos que existe uma sub-sequência de
> a, chamemos ela de A, tal que todas as potências associadas a A são
> potências de um mesmo primo p. Além disso, a distância entre os A
> consecutivos na sequência original a é no máximo 2(k+1), afinal o pior
> que pode acontecer é que pegamos dois índices que ficavam em extremos
> opostos de faixas distintas.
>
> Pois bem, peguemos dos A_t consecutivos, a saber, a_t, e a_(t+v).
> Podemos dizer que existe um u máximo tal que p^u | a_t e p^u |
> a_(t+v). Logo,
>
> p^u | a_(t+v) - 2017^v * a_t = b * 2016^t * (2017^v-2016^v) | M_t *
> (2017^v-2016^v)
>
> Mas isto implica p^u <= M_t * (2017^v-2016^v) <= ab *
> (2017^(2k+2)-2016^(2k+2)), ou seja, p^u é limitado.
>
> Porém, a_t = p_1^(e(1,t)) * p_2^(e(2,t)) * p_3^(e(3,t)) * ... *
> p_k^(e(k,t)) <= p^u * p^u * p^u * ... * p^u = (p^u)^k, o que implica
> que (p^u) é ilimitado (pois a_t é ilimitado).
>
> E isto é um absurdo! Logo, o nosso caso II não se sustenta - e ficamos
> com o caso I apenas!
>
>>
>> --
>> Esta mensagem foi verificada pelo sistema de antivírus e
>> acredita-se estar livre de perigo.
>
> Já que você diz...

-- 
Esta mensagem foi verificada pelo sistema de antiv�rus e
 acredita-se estar livre de perigo.


=
Instru��es para entrar na lista, sair da lista e usar a lista em
http://www.mat.puc-rio.br/~obmlistas/obm-l.html
=


Re: [obm-l] Problema 2 da OBM U

2017-11-18 Por tôpico Anderson Torres
Em 15 de novembro de 2017 15:01, Otávio Araújo
 escreveu:
> Alguém poderia me ajudar no problema 2 da segunda fase da obm u desse ano? O
> enunciado é o seguinte:
> "Fixados os inteiros positivos a e b, mostre que o conjunto dos divisores
> primos dos termos da sequencia
> an = a · 2017^n + b · 2016^n
> é infinito."

Não é difícil difícil, apenas precisa ter a ideia certa. Aqui eu fui
um pouco mais formalista do que o necessário, mas a ideia é que se
construirmos uma sequência dessas com poucos primos, eles explodem.

Caso I: o conjunto {p_1,p_2,...} de todos os primos que aparecem em
todas as fatorações de a_n é infinito. Neste caso, o problema acaba
antes mesmo de começar.

Caso II: o conjunto {p_1,p_2,...,p_k} de todos os primos que aparecem
em todas as fatorações de a_n é finito. Vamos trabalhar apenas com
esses primos, a não ser que seja dito algo contrário.

Assim sendo, podemos fatorar cada a_n da seguinte forma:

a_n = p_1^(e(1,n)) * p_2^(e(2,n)) * p_3^(e(3,n)) * ... * p_k^(e(k,n))

A cada a_n, vamos associar a maior dessas potências. Por exemplo, 72 =
2^3 * 3^2 estaria associada a 3^2, pois 3^2 = 9 > 8 = 2^3.

LEMA: Seja M_n = MDC(a*2017^n, b*2016^n). Então M_n é limitada.

De fato, se tentarmos fatorar, os fatores de 2017 não aparecem em 2016
e vice-versa, dado serem primos entre si. Logo os fatores de 2017 só
podem aparecer em b, e os fatores de 2016 só podem aparecer em a.
Portanto, M_n = MDC(a,2016^n) * MDC(b,2017^n) | ab, logo, limitado.

Vamos dividir nossa sequência em fitas de tamanho (K+1):

a_(0*k+0), a_(0*k+1),...,a_(0*k+k)
a_(1*k+0), a_(1*k+1),...,a_(1*k+k)
a_(2*k+0), a_(2*k+1),...,a_(2*k+k)
a_(3*k+0), a_(3*k+1),...,a_(3*k+k)
a_(4*k+0), a_(4*k+1),...,a_(4*k+k)
a_(5*k+0), a_(5*k+1),...,a_(5*k+k)
... ... ...

Se aplicarmos o princípio das gavetas em cada fita separadamente,
podemos dizer que para cada fita existe um primo tal que potências
desse primo aparecem (pelo menos) duas vezes entre os associados.
Vamos então anotar este primo para cada faixa:

a_(0*k+0), a_(0*k+1),...,a_(0*k+k)
a_(1*k+0), a_(1*k+1),...,a_(1*k+k)
a_(2*k+0), a_(2*k+1),...,a_(2*k+k)
a_(3*k+0), a_(3*k+1),...,a_(3*k+k)
a_(4*k+0), a_(4*k+1),...,a_(4*k+k)
a_(5*k+0), a_(5*k+1),...,a_(5*k+k)


Como o número de faixas é infinito e o número de possíveis primos
associados é finito, então existe um primo tal que infinitas faixas
tenham potências desse mesmo primo associadas.

Com esse raciocínio, nós demonstramos que existe uma sub-sequência de
a, chamemos ela de A, tal que todas as potências associadas a A são
potências de um mesmo primo p. Além disso, a distância entre os A
consecutivos na sequência original a é no máximo 2(k+1), afinal o pior
que pode acontecer é que pegamos dois índices que ficavam em extremos
opostos de faixas distintas.

Pois bem, peguemos dos A_t consecutivos, a saber, a_t, e a_(t+v).
Podemos dizer que existe um u máximo tal que p^u | a_t e p^u |
a_(t+v). Logo,

p^u | a_(t+v) - 2017^v * a_t = b * 2016^t * (2017^v-2016^v) | M_t *
(2017^v-2016^v)

Mas isto implica p^u <= M_t * (2017^v-2016^v) <= ab *
(2017^(2k+2)-2016^(2k+2)), ou seja, p^u é limitado.

Porém, a_t = p_1^(e(1,t)) * p_2^(e(2,t)) * p_3^(e(3,t)) * ... *
p_k^(e(k,t)) <= p^u * p^u * p^u * ... * p^u = (p^u)^k, o que implica
que (p^u) é ilimitado (pois a_t é ilimitado).

E isto é um absurdo! Logo, o nosso caso II não se sustenta - e ficamos
com o caso I apenas!

>
> --
> Esta mensagem foi verificada pelo sistema de antivírus e
> acredita-se estar livre de perigo.

Já que você diz...

-- 
Esta mensagem foi verificada pelo sistema de antiv�rus e
 acredita-se estar livre de perigo.


=
Instru��es para entrar na lista, sair da lista e usar a lista em
http://www.mat.puc-rio.br/~obmlistas/obm-l.html
=


[obm-l] Re: [obm-l] Problema de álgebra

2017-09-15 Por tôpico Carlos Nehab
Oi, Leonardo (e Ralph)

Resolvi postar meu "rabisco de tentativa de solução" pois acho (e com
certeza Ralph tb) que isso enriquece o aprendizado da gurizada (sorry pelo
gurizada, mas me formei em 1969...).

Fiz o seguinte:
(Supondo numa primeira abordagem que x, y e z fossem >= -1, prá ver onde
isso poderia parar).

Calculei MG e MA entre (1+x), (1+y) e (1+z).

A razão desse aparente coelho da cartola, Leonardo, é que o tal do x+y+z =
1, o xy+xz+yz e o xyz do seu enunciado me deram uma coceira num "produtinho
não tão notável, mas útil, que já matou inúmeros problemas, qual seja:

(1+x)(1+y)(1+z) = 1 + (x+y+z) + (xy+yz+xz) + xyz, que, nesse seu caso, vale
2 + a + xyz

Então, prosseguindo:

MA >= MG, acarreta 4/3 >= (2 + a + xyz)^1/3, ou seja, xyz <= (10/27 - a),
que aliás, é positivo, pois  a < 1/3).

Mas esse rabisco não funciona pois não conseguimos garantir que o xyz
atinge o valor (10/27) - a, até porque isso só ocorreria se x = y = z o que
não é o caso, conforme a criativa solução do Ralph.

Abs de um colega mais velho que a lista (rsrsrs)...
Nehab

Em 15 de setembro de 2017 15:13, Leonardo Joau 
escreveu:

> Dados os reais x, y,z, tais que:
>
> x+y+z = 1
>
> xy+xz+yz = a  0
> Calcule o max{xyz} em função de a.
>
>
> Att,
> Leonardo Joau
>
> --
> Esta mensagem foi verificada pelo sistema de antivírus e
> acredita-se estar livre de perigo.

-- 
Esta mensagem foi verificada pelo sistema de antiv�rus e
 acredita-se estar livre de perigo.



[obm-l] Re: [obm-l] Re: [obm-l] Problema de álgebra

2017-09-15 Por tôpico Leonardo Joau
On Fri, 15 Sep 2017 at 18:42 Ralph Teixeira  wrote:

> Bom, suponho que queremos alguma solucao que nao use tecnicas de Calculo?
>
> Que tal assim: x, y e z sao raizes do polinomio:
>
> t^3-t^2+at-P=0
>
> onde P eh o que voce quer maximizar.
>
> O polinomio f(t)=t^3-t^2+at-P sempre tem pelo menos uma raiz real (grau
> 3). Quando voce muda P, voce translada o grafico de f(t) para cima (ou para
> baixo). Assim, a ideia eh levar o grafico para cima o maximo possivel,
> mantendo sempre 3 raizes reais -- isto eh, o P maximo acontece quando temos
> uma raiz dupla!
>
> Assim, podemos supor spdg x=y=r no P maximo, e portanto z=1-2r. Jogue isso
> em xy+xz+yz=a para descobrir esse r otimo em termos de a (confira que esse
> r eh real, garantindo a existencia das 3 raizes de fato), e calcule
> P=r.r.(1-2r) para descobrir o tal produto maximo.
>
> Abraco, Ralph.
>
>
> 2017-09-15 15:13 GMT-03:00 Leonardo Joau :
>
>> Dados os reais x, y,z, tais que:
>>
>> x+y+z = 1
>>
>> xy+xz+yz = a  0>
>> Calcule o max{xyz} em função de a.
>>
>>
>> Att,
>> Leonardo Joau
>>
>> --
>> Esta mensagem foi verificada pelo sistema de antivírus e
>> acredita-se estar livre de perigo.
>
>
> --
> Esta mensagem foi verificada pelo sistema de antivírus e
> acredita-se estar livre de perigo.



Eu criei essa questão pensando exatamente nesse polinômio e na translação
do gráfico.
Acredito que fica bem dificil usando somente as desigualdades classicas.

Att,
Leonardo

>

-- 
Esta mensagem foi verificada pelo sistema de antiv�rus e
 acredita-se estar livre de perigo.



[obm-l] Re: [obm-l] Problema de álgebra

2017-09-15 Por tôpico Ralph Teixeira
Bom, suponho que queremos alguma solucao que nao use tecnicas de Calculo?

Que tal assim: x, y e z sao raizes do polinomio:

t^3-t^2+at-P=0

onde P eh o que voce quer maximizar.

O polinomio f(t)=t^3-t^2+at-P sempre tem pelo menos uma raiz real (grau 3).
Quando voce muda P, voce translada o grafico de f(t) para cima (ou para
baixo). Assim, a ideia eh levar o grafico para cima o maximo possivel,
mantendo sempre 3 raizes reais -- isto eh, o P maximo acontece quando temos
uma raiz dupla!

Assim, podemos supor spdg x=y=r no P maximo, e portanto z=1-2r. Jogue isso
em xy+xz+yz=a para descobrir esse r otimo em termos de a (confira que esse
r eh real, garantindo a existencia das 3 raizes de fato), e calcule
P=r.r.(1-2r) para descobrir o tal produto maximo.

Abraco, Ralph.


2017-09-15 15:13 GMT-03:00 Leonardo Joau :

> Dados os reais x, y,z, tais que:
>
> x+y+z = 1
>
> xy+xz+yz = a  0
> Calcule o max{xyz} em função de a.
>
>
> Att,
> Leonardo Joau
>
> --
> Esta mensagem foi verificada pelo sistema de antivírus e
> acredita-se estar livre de perigo.

-- 
Esta mensagem foi verificada pelo sistema de antiv�rus e
 acredita-se estar livre de perigo.



[obm-l] Re: [obm-l] Re: [obm-l] Re: [obm-l] Problema difícil.

2017-09-13 Por tôpico Ralph Teixeira
Oi, Douglas.

Acho que o que você fez é um bom começo.

Vamos adaptar: pense ao invés nos números de 1009 a 2017 (conjunto A).

i) Eles podem todos parear com os números de 1 a 1008?
ii) Então pelo menos um produto usando os elementos de A vai dar NO MÍNIMO
NO MÍNIMO...
iii) Esse número do item anterior, pode ser o máximo de todos eles? Como?

Abraço, Ralph.



2017-09-13 7:11 GMT-03:00 Douglas Oliveira de Lima <
profdouglaso.del...@gmail.com>:

> Então Bernardo, eu pensei numa parada mas não tenho certeza , pensei que
> os números 997,998,999,...,1994 Não poderiam ocupar as posições de 1 a
> 1997, logo pelo menos um deles ocuparia uma posição não inferior a 998, aí
> pensei no 997.998=995006.
>
> Em 12 de set de 2017 18:39, "Bernardo Freitas Paulo da Costa" <
> bernardo...@gmail.com> escreveu:
>
>> 2017-09-12 17:51 GMT-03:00 Douglas Oliveira de Lima
>> :
>> > Considere a sequência de números 1,2,3,4,5,...,2017.
>> > E uma certa ordenação deles a1, a2, a3, ..., a2017.
>> > Agora multiplique respectivamente os números das duas sequencias
>> > determinando assim uma nova sequência 1.a1
>> , 2.a2, 3.a3, ...,
>> 2017.a2017.
>> >
>> > Qual o menor valor que o maior produto da última sequência pode assumir?
>>
>> Esse problema não é tão difícil quanto parece.  O que você tentou fazer?
>>
>> Abraços,
>> --
>> Bernardo Freitas Paulo da Costa
>>
>> --
>> Esta mensagem foi verificada pelo sistema de antivírus e
>>  acredita-se estar livre de perigo.
>>
>>
>> =
>> Instru�ões para entrar na lista, sair da lista e usar a lista em
>> http://www.mat.puc-rio.br/~obmlistas/obm-l.html
>> =
>>
>
> --
> Esta mensagem foi verificada pelo sistema de antivírus e
> acredita-se estar livre de perigo.

-- 
Esta mensagem foi verificada pelo sistema de antiv�rus e
 acredita-se estar livre de perigo.



[obm-l] Re: [obm-l] Re: [obm-l] Problema difícil.

2017-09-13 Por tôpico Douglas Oliveira de Lima
Então Bernardo, eu pensei numa parada mas não tenho certeza , pensei que os
números 997,998,999,...,1994 Não poderiam ocupar as posições de 1 a 1997,
logo pelo menos um deles ocuparia uma posição não inferior a 998, aí pensei
no 997.998=995006.

Em 12 de set de 2017 18:39, "Bernardo Freitas Paulo da Costa" <
bernardo...@gmail.com> escreveu:

> 2017-09-12 17:51 GMT-03:00 Douglas Oliveira de Lima
> :
> > Considere a sequência de números 1,2,3,4,5,...,2017.
> > E uma certa ordenação deles a1, a2, a3, ..., a2017.
> > Agora multiplique respectivamente os números das duas sequencias
> > determinando assim uma nova sequência 1.a1, 2.a2, 3.a3, ..., 2017.a2017.
> >
> > Qual o menor valor que o maior produto da última sequência pode assumir?
>
> Esse problema não é tão difícil quanto parece.  O que você tentou fazer?
>
> Abraços,
> --
> Bernardo Freitas Paulo da Costa
>
> --
> Esta mensagem foi verificada pelo sistema de antivírus e
>  acredita-se estar livre de perigo.
>
>
> =
> Instru�ões para entrar na lista, sair da lista e usar a lista em
> http://www.mat.puc-rio.br/~obmlistas/obm-l.html
> =
>

-- 
Esta mensagem foi verificada pelo sistema de antiv�rus e
 acredita-se estar livre de perigo.



[obm-l] Re: [obm-l] Re: [obm-l] Re: [obm-l] Problema difícil.

2017-09-12 Por tôpico Israel Meireles Chrisostomo
Exatamente, aplique a desigualdade do rearranjo

Em 12 de setembro de 2017 19:08, Leonardo Joau 
escreveu:

>
> On Tue, 12 Sep 2017 at 18:39 Bernardo Freitas Paulo da Costa <
> bernardo...@gmail.com> wrote:
>
>> 2017-09-12 17:51 GMT-03:00 Douglas Oliveira de Lima
>> :
>> > Considere a sequência de números 1,2,3,4,5,...,2017.
>> > E uma certa ordenação deles a1, a2, a3, ..., a2017.
>> > Agora multiplique respectivamente os números das duas sequencias
>> > determinando assim uma nova sequência 1.a1
>> , 2.a2, 3.a3, ...,
>> 2017.a2017.
>> >
>> > Qual o menor valor que o maior produto da última sequência pode assumir?
>>
>> Esse problema não é tão difícil quanto parece.  O que você tentou fazer?
>>
>> Abraços,
>> --
>> Bernardo Freitas Paulo da Costa
>>
>> --
>> Esta mensagem foi verificada pelo sistema de antivírus e
>>  acredita-se estar livre de perigo.
>>
>>
>> =
>> Instru�ões para entrar na lista, sair da lista e usar a lista em
>> http://www.mat.puc-rio.br/~obmlistas/obm-l.html
>> =
>
>
> Parece uma aplicação da desigualdade do rearranjo.
>
> Link:
> https://pt.m.wikipedia.org/wiki/Desigualdade_do_rearranjo
>
>
>>
> --
> Esta mensagem foi verificada pelo sistema de antivírus e
> acredita-se estar livre de perigo.
>



-- 
Israel Meireles Chrisostomo

-- 
Esta mensagem foi verificada pelo sistema de antiv�rus e
 acredita-se estar livre de perigo.



[obm-l] Re: [obm-l] Re: [obm-l] Problema difícil.

2017-09-12 Por tôpico Leonardo Joau
On Tue, 12 Sep 2017 at 18:39 Bernardo Freitas Paulo da Costa <
bernardo...@gmail.com> wrote:

> 2017-09-12 17:51 GMT-03:00 Douglas Oliveira de Lima
> :
> > Considere a sequência de números 1,2,3,4,5,...,2017.
> > E uma certa ordenação deles a1, a2, a3, ..., a2017.
> > Agora multiplique respectivamente os números das duas sequencias
> > determinando assim uma nova sequência 1.a1, 2.a2, 3.a3, ..., 2017.a2017.
> >
> > Qual o menor valor que o maior produto da última sequência pode assumir?
>
> Esse problema não é tão difícil quanto parece.  O que você tentou fazer?
>
> Abraços,
> --
> Bernardo Freitas Paulo da Costa
>
> --
> Esta mensagem foi verificada pelo sistema de antivírus e
>  acredita-se estar livre de perigo.
>
>
> =
> Instru�ões para entrar na lista, sair da lista e usar a lista em
> http://www.mat.puc-rio.br/~obmlistas/obm-l.html
> =


Parece uma aplicação da desigualdade do rearranjo.

Link:
https://pt.m.wikipedia.org/wiki/Desigualdade_do_rearranjo


>

-- 
Esta mensagem foi verificada pelo sistema de antiv�rus e
 acredita-se estar livre de perigo.



[obm-l] Re: [obm-l] Problema difícil.

2017-09-12 Por tôpico Bernardo Freitas Paulo da Costa
2017-09-12 17:51 GMT-03:00 Douglas Oliveira de Lima
:
> Considere a sequência de números 1,2,3,4,5,...,2017.
> E uma certa ordenação deles a1, a2, a3, ..., a2017.
> Agora multiplique respectivamente os números das duas sequencias
> determinando assim uma nova sequência 1.a1, 2.a2, 3.a3, ..., 2017.a2017.
>
> Qual o menor valor que o maior produto da última sequência pode assumir?

Esse problema não é tão difícil quanto parece.  O que você tentou fazer?

Abraços,
-- 
Bernardo Freitas Paulo da Costa

-- 
Esta mensagem foi verificada pelo sistema de antiv�rus e
 acredita-se estar livre de perigo.


=
Instru��es para entrar na lista, sair da lista e usar a lista em
http://www.mat.puc-rio.br/~obmlistas/obm-l.html
=


Re: [obm-l] Problema estranho

2017-09-07 Por tôpico Anderson Torres
Bora lá...


Pelo que a galera já demonstrou, o resultado vale se todos os números
da sequência forem racionais. Agora, falta cobrir os irracionais.

Considere

- real eps>0
- inteiro m>0
- inteiros p_1, p_2, ... p_(2n+1)

tais que, para todo i, vale |p_i-mx_i| < eps.

A ideia é que se eps for bem pequenininho, os p_i e os x_i terão a
mesma propriedade (se tirar um, dá para rachar ao meio).

De fato, fixando i:

soma(j <> i)(a_ij * m* x_j) = 0 , para alguma combinação de a_ij em
{-1,+1} (tente imaginar uma balança: se o número x_j está no prato
direito, usamos -1; caso contrário, +1).

Ou também

soma(j <> i)(a_ij * (m* x_j-p_j)) =  - soma(j <> i)(a_ij * p_j)

Passa o módulo:

| soma(j <> i)(a_ij * p_j) | = | soma(j <> i)(a_ij * (m* x_j-p_j)) | <= 2n * eps

Mas olha só, o | soma(j <> i)(a_ij * p_j) | é um inteiro positivo
arbitrariamente pequeno! Isso na minha terra tem um nome: ZERO!

LOGO, como os ilustres colegas da lista mostraram, todos esse p_i
devem ser iguais.

LOGO, para todo K grandão existem inteiros n_K e p_K tais que |p_K  -
n_K * x_i| <= 1/K

Como pelo menos um dos caras é irracional, é fácil ver que n_K pode
ser arbitrariamente grande. Mas 2/N > |n_K| * max |x_i-x_j|, e isso
implica max |x_i-x_j| = 0.

That's it!





Em 15 de julho de 2017 20:21, Anderson Torres
 escreveu:
> Uma ideia pode ser tentar aproximar os reais para racionais e usar o
> argumento das potências, não?
>
> Em 11 de julho de 2017 18:21, Matheus Secco  escreveu:
>> Oi Ralph, tava sem tempo de escrever, mas vou aproveitar a deixa porque você
>> já fez quase tudo. Acho que dá pra fazer o caso geral usando que os reais
>> admitem uma base considerando como um espaço vetorial sobre os racionais.
>>
>> Em ter, 11 de jul de 2017 às 18:18, Ralph Teixeira 
>> escreveu:
>>>
>>> Bom, eu sei resolver se todos os números forem racionais. Deve ter um
>>> jeito de usar isso para o caso geral...
>>>
>>> A propriedade desse conjunto não se altera se todos os elementos do
>>> conjunto forem multiplicados por um mesmo número, nem se a gente somar uma
>>> certa constante a todos eles.
>>>
>>> Assim, *SE* eles forem todos racionais, a gente pode multiplicar todos
>>> eles por um m.m.c imenso e supor que são todos inteiros, spdg.
>>>
>>> Mas então todos teriam que ter a mesma paridade -- afinal a soma de todos
>>> eles, menos qualquer um deles, é um número par.
>>>
>>> Então, enquanto todos forem pares, divida-os por 2; em algum momento,
>>> **todos** ficarão ímpares. Quando isso acontecer, some 1, e ficam todos
>>> pares. Então divida por 2 de novo, e de novo, até ficarem ímpares, então
>>> some 1 de novo, repita e enxágue.
>>>
>>> Esse processo vai parar? Oras, esses inteiros vão diminuir em módulo
>>> até até até cada um deles virar 0, ou 1! De fato, |x|/2<|x| quando
>>> x<>0, e |x+1|/2 < |x| para x<>0,1. Então a cada um ou dois passos o valor
>>> absoluto de todos eles diminui -- a menos que eles sejam 0 ou 1. Ou seja, em
>>> tempo finito, todos eles vão virar 0 ou 1.
>>>
>>> Agora é fácil -- lembra que todos sempre têm a mesma paridade?? Então são
>>> todos 0, ou todos 1.
>>>
>>> ---///---
>>>
>>> Para o caso geral, tenho uma ideia, mas não estou com tempo de
>>> desenvolvê-la -- será que dá para começar com os reais, e multiplicar todos
>>> eles por algum número real imenso de forma que eles sejam quase inteiros
>>> (tipo, todos eles a menos de 1/(4n) de algum inteiro)? Talvez dê para
>>> mostrar então pela propriedade que eles têm que ser inteiros, ou pelo menos
>>> "comensuráveis" e daí matar o problema.
>>>
>>> Abraço, Ralph.
>>>
>>>
>>>
>>> 2017-07-11 15:06 GMT-03:00 Nowras Ali :

 Uma prova por indução me parece o melhor caminho.
 O Bernardo já provou para o caso base, basta agora tentar
 provar para n+1, assumindo verdadeiro para n. Tentarei resolver
 o problema assim que puder.

 Abraços, Nowras.

 Em 9 de julho de 2017 18:54, Otávio Araújo 
 escreveu:
>
>
> Já tentei isso, porém não parece ajudar em muita coisa  mas de
> qualquer forma obrigado
>
> > Em 9 de jul de 2017, às 18:00, Bernardo Freitas Paulo da Costa
> >  escreveu:
> >
> > Não pensei muito, mas acho que você deveria tentar provar os casos
> > n=1
> > e n=2 "no braço" para ter a intuição.  E, na verdade, o enunciado
> > deveria ser: dados a_1, a_2, ... a_{2n+1} números reais, não
> > necessariamente distintos, tais que, para cada escolha de 2n dentre
> > eles, é possível separar em dois grupos de n cada, com a mesma soma.
> > (evitando falar de conjuntos, você pode ter à vontade os elementos
> > repetidos).
> >
> > Assim, o caso n=1 fica: temos a_1, a_2, a_3.  Tomando os elementos
> > a_1, a_2, é possĩvel separar em dois grupos de um elemento, com a
> > soma
> > igual.  Logo 

Re: [obm-l] Problema de grafos

2017-09-03 Por tôpico Daniel da Silva
Obrigado pela ajuda Esdras e Matheus.

Daniel Rocha da Silva

> Em 2 de set de 2017, às 13:23, Esdras Muniz  
> escreveu:
> 
> Cada vértice pode ter como grau um número de 0 a n-1, porém o 0 e o n-1 
> não podem ambos ser graus de vértices, pois se um tem grau n-1 então ele 
> está ligado a todos os outros vértices. Então há apenas n-1 
> possibilidades para o grau de cada vértice. Pelo pcp há dois vértices com 
> o mesmo grau.
> 
> Em 2 de set de 2017 12:34 PM, "Daniel Rocha"  
> escreveu:
>> Bom dia,
>> 
>> Seja G um grafo com n vértices, n maior que 1. Suponha que G não possua
>> loops nem mais de uma aresta unindo pares de vértices. Prove que G possui
>> dois vértices de graus iguais.
>> 
>> Obrigado,
>> Daniel
>> --
>> Esta mensagem foi verificada pelo sistema de antivírus e
>> Â acredita-se estar livre de perigo.
>> 
>> 
>> =
>> Instruções para entrar na lista, sair da lista e usar a lista em
>> http://www.mat.puc-rio.br/~obmlistas/obm-l.html
>> =
> 
> -- 
> Esta mensagem foi verificada pelo sistema de antivírus e 
> acredita-se estar livre de perigo.

-- 
Esta mensagem foi verificada pelo sistema de antiv�rus e
 acredita-se estar livre de perigo.



Re: [obm-l] Problema de grafos

2017-09-02 Por tôpico Esdras Muniz
Cada vértice pode ter como grau um número de 0 a n-1, porém o 0 e o n-1 não
podem ambos ser graus de vértices, pois se um tem grau n-1 então ele está
ligado a todos os outros vértices. Então há apenas n-1 possibilidades para
o grau de cada vértice. Pelo pcp há dois vértices com o mesmo grau.

Em 2 de set de 2017 12:34 PM, "Daniel Rocha" 
escreveu:

> Bom dia,
>
> Seja G um grafo com n vértices, n maior que 1. Suponha que G não possua
> loops nem mais de uma aresta unindo pares de vértices. Prove que G possui
> dois vértices de graus iguais.
>
> Obrigado,
> Daniel
> --
> Esta mensagem foi verificada pelo sistema de antivírus e
>  acredita-se estar livre de perigo.
>
>
> =
> Instruções para entrar na lista, sair da lista e usar a lista em
> http://www.mat.puc-rio.br/~obmlistas/obm-l.html
> =
>

-- 
Esta mensagem foi verificada pelo sistema de antiv�rus e
 acredita-se estar livre de perigo.



Re: [obm-l] Problema de grafos

2017-09-02 Por tôpico Matheus Secco
Olá Daniel, veja que os graus podem variar de 0 até n - 1. Entretanto, não
é possível ter um vértice com grau 0 e outro com grau n - 1. Desta forma,
em vez de n possibilidades para o grau de cada vértice, há n - 1
possibilidades para o grau de cada vértice. Como há n vértices, pelo
Princípio da Casa dos Pombos, há dois com o mesmo grau.
Abraços,
Matheus Secco

2017-09-02 11:26 GMT-03:00 Daniel Rocha :

> Bom dia,
>
> Seja G um grafo com n vértices, n maior que 1. Suponha que G não possua
> loops nem mais de uma aresta unindo pares de vértices. Prove que G possui
> dois vértices de graus iguais.
>
> Obrigado,
> Daniel
> --
> Esta mensagem foi verificada pelo sistema de antivírus e
>  acredita-se estar livre de perigo.
>
>
> =
> Instruções para entrar na lista, sair da lista e usar a lista em
> http://www.mat.puc-rio.br/~obmlistas/obm-l.html
> =
>

-- 
Esta mensagem foi verificada pelo sistema de antiv�rus e
 acredita-se estar livre de perigo.



Re: [obm-l] Problema de Probabilidade

2017-08-08 Por tôpico Ralph Teixeira
Ah, se voce preferir, pode dividir a tabela por jogador mesmo, assim:

/// A B CD E FG  Total
JV   60   60   60   60   45   45   25  355
JP   40   40   40   40   55   55   75  345
Tot 100 100 100 100 100 100 100  700

a) Pr(JV)=355/700
b) Pr(E|JV)=45/355

Abraco, Ralph.

2017-08-08 11:26 GMT-03:00 Ralph Teixeira :

> Este problema sai formalmente usando a Regra de Bayes Mas eu sempre
> achei que, quando o problema eh pequeno, fica muito mais facil de entender
> o que estah havendo e resolver varios itens usando usando uma tabela.
>
> (Obs.: antes que alguem critique: minha tabela NAO reflete o que VAI
> acontecer quando jogarmos x jogos; eh apenas uma tabela CUJAS PROPORCOES
> sao identicas aas probabilidades, e que portanto pode ser usada para
> calcular qualquer probabilidade condicional.)
>
> Para economizar bits, vou denotar alguns eventos assim:
>
> P1: evento "o adversario veio do grupo 1 {A,B,C,D}"
> P2: evento "o adversario veio do grupo 2 {E,F}"
> P3: evento "o adversario foi G"
> JV: J vence seu jogo
> JP: J perde seu jogo
>
> Entao, vou supor 700 jogos no total e usar que 4/7 deste vao para P1, 2/7
> para P2 e 1/7 para P3 (suponho que "selecionado aleatoriamente" signifique
> "uniformemente"):
>
> ///  P1   P2   P3   Tot
> JV
> JP
> Tot 400 200 100 700
>
> (Obs.2: 700 eh um numero arbitrario para as contas ficarem redondas; use
> qualquer outra coisa se desejar, nao importa, pois vamos fazer apenas
> proporcoes mesmo.)
>
> Agora vamos usar as condicionais dadas: Pr(JV|P1)=0,6, por exemplo. Isto
> significa que, daqueles 400 jogos em que o adversario vem de P1, J vence
> 0,6*400=240 deles. Analogamente, Pr(JV|P2)=0,45 e Pr(JV|P3)=0,25. Assim,
> completo a tabela:
>
> ///  P1P2 P3   Tot
> JV 240   90 25  355
> JP 160  11075  345
> Tot 400  200  100 700
>
> Agora eh muito facil responder QUALQUER coisa. Vejamos:
>
> a) Queremos Pr(JV). Temos da tabela Pr(JV)=355/700
> b) Queremos Pr(P2|JV), ou quase isso. Bom, SABENDO que J venceu, estamos
> na linha 1, estamos nos restringindo a algum daqueles 355 jogos. Neste
> caso, a probabilidade do jogador ter vindo do grupo 2 seria:
> Pr(P2 | JV) = 90/355
> Entao a resposta eh 45/355 (pois ha 2 jogadores no grupo 2, igualmente
> provaveis)
>
> Abraco, Ralph.
>
> 2017-08-08 10:21 GMT-03:00 Luiz Antonio Rodrigues :
>
>> Olá, pessoal!
>> Bom dia!
>> Será que alguém pode me ajudar com o problema abaixo? Estou quebrando a
>> cabeça e não consigo resolvê-lo.
>> Muito obrigado e um abraço!
>> Luiz
>>
>> Um jogador J entra em um torneio de tênis com jogos eliminatórios. Seu
>> primeiro adversário será selecionado aleatoriamente a partir de um conjunto
>> de 7 jogadores: {A,B,C,D,E,F,G}. Contra 4 adversários (A,B,C,D) desse
>> conjunto, a probabilidade de vitória de J é 0,6; contra dois adversários
>> desse conjunto (E,F), a probabilidade de vitória de J é 0,45 e contra o
>> adversário restante (G), a probabilidade de vitória de J é 0,25.
>> a) Qual a probabilidade de vitória de J na primeira partida do torneio?
>>
>>
>> b) Suponha que a primeira partida já tenha sido realizada. Você fica
>> sabendo que J venceu esse jogo. Qual a probabilidade de que J tenha jogado
>> contra E?
>>
>>
>>
>> --
>> Esta mensagem foi verificada pelo sistema de antivírus e
>> acredita-se estar livre de perigo.
>
>
>

-- 
Esta mensagem foi verificada pelo sistema de antiv�rus e
 acredita-se estar livre de perigo.



[obm-l] Re: [obm-l] Problema de função elementar

2017-07-17 Por tôpico Alexandre Antunes
Boa noite,

Encontrei um resultado aproximado
F (21/2017)=F(0,01)=0,054719

Não sei se fiz "besteiras", mas usando a expressão em b

F (x/3) = F(x)/2

x=1 =》F(1/3)=F(1)/2=1/2
x=1/3 =》F(1/9)=F(1/3)/2=1/4
Generalizando
x=1/3^n =》F(1/3^n)=1/2^n

Por outro lado
Para x=21/2017=0,01=1/3^n, portanto (se não errei), n=2/log 3

Então,  F(1/3^(2/log 3) =
  = F(0,01) =1/2^(2/log 3) =   = 0,054719...

Tem sentido?




Em 13/07/2017 09:51, "Douglas Oliveira de Lima" <
profdouglaso.del...@gmail.com> escreveu:

Seja F uma função crescente definida para todo número real x, 0<=x<=1, tal
que

a)  F(0)=0

b)  F(x/3)=F(x)/2

c)  F(1-x)=1-F(x)

Encontrar F(21/2017).


Douglas Oliveira

-- 
Esta mensagem foi verificada pelo sistema de antivírus e
acredita-se estar livre de perigo.

-- 
Esta mensagem foi verificada pelo sistema de antiv�rus e
 acredita-se estar livre de perigo.



[obm-l] Re: [obm-l] Re: [obm-l] Re: [obm-l] Problema de função elementar

2017-07-17 Por tôpico Guilherme Oliveira
Aproveitando o problema, quanto seria f (0,1)?


Tenham uma boa noite,
Guilherme

Em 17/07/2017 12:45, "Pedro José"  escreveu:

Bom dia!

Seguindo a linha proposta pelo Anderson.

7/3^6 < 21/2017 < 8/3^6 ==> F(21/2017)= F(7/3^6)=F(8/3^6)

F(7/9) = 3/4.  F(7/3^6) = F(7/9/3^4)= F(7/9)/2^4= 3/2^6= 3/64.

Sds,
PJMS


Em 17 de julho de 2017 10:48, Pedro José  escreveu:

> Bom dia!
>
> Há uma restrição para a função ser crescente. Portanto F(1) é máximo e
> F(1) = 1, logo não pode ser 87. tem que ser um valor menor ou igual a 1 e
> maior ou igual a zero.
>
> Sds,
> PJMS
>
> Em 15 de julho de 2017 20:54, Matheus Herculano <
> matheusherculan...@gmail.com> escreveu:
>
>> O resultado é 87
>>
>> Em 13 de jul de 2017 09:51, "Douglas Oliveira de Lima" <
>> profdouglaso.del...@gmail.com> escreveu:
>>
>>> Seja F uma função crescente definida para todo número real x, 0<=x<=1,
>>> tal que
>>>
>>> a)  F(0)=0
>>>
>>> b)  F(x/3)=F(x)/2
>>>
>>> c)  F(1-x)=1-F(x)
>>>
>>> Encontrar F(21/2017).
>>>
>>>
>>> Douglas Oliveira
>>>
>>> --
>>> Esta mensagem foi verificada pelo sistema de antivírus e
>>> acredita-se estar livre de perigo.
>>
>>
>> --
>> Esta mensagem foi verificada pelo sistema de antivírus e
>> acredita-se estar livre de perigo.
>>
>
>

-- 
Esta mensagem foi verificada pelo sistema de antivírus e
acredita-se estar livre de perigo.

-- 
Esta mensagem foi verificada pelo sistema de antiv�rus e
 acredita-se estar livre de perigo.



[obm-l] Re: [obm-l] Re: [obm-l] Problema de função elementar

2017-07-17 Por tôpico Pedro José
Bom dia!

Seguindo a linha proposta pelo Anderson.

7/3^6 < 21/2017 < 8/3^6 ==> F(21/2017)= F(7/3^6)=F(8/3^6)

F(7/9) = 3/4.  F(7/3^6) = F(7/9/3^4)= F(7/9)/2^4= 3/2^6= 3/64.

Sds,
PJMS


Em 17 de julho de 2017 10:48, Pedro José  escreveu:

> Bom dia!
>
> Há uma restrição para a função ser crescente. Portanto F(1) é máximo e
> F(1) = 1, logo não pode ser 87. tem que ser um valor menor ou igual a 1 e
> maior ou igual a zero.
>
> Sds,
> PJMS
>
> Em 15 de julho de 2017 20:54, Matheus Herculano <
> matheusherculan...@gmail.com> escreveu:
>
>> O resultado é 87
>>
>> Em 13 de jul de 2017 09:51, "Douglas Oliveira de Lima" <
>> profdouglaso.del...@gmail.com> escreveu:
>>
>>> Seja F uma função crescente definida para todo número real x, 0<=x<=1,
>>> tal que
>>>
>>> a)  F(0)=0
>>>
>>> b)  F(x/3)=F(x)/2
>>>
>>> c)  F(1-x)=1-F(x)
>>>
>>> Encontrar F(21/2017).
>>>
>>>
>>> Douglas Oliveira
>>>
>>> --
>>> Esta mensagem foi verificada pelo sistema de antivírus e
>>> acredita-se estar livre de perigo.
>>
>>
>> --
>> Esta mensagem foi verificada pelo sistema de antivírus e
>> acredita-se estar livre de perigo.
>>
>
>

-- 
Esta mensagem foi verificada pelo sistema de antiv�rus e
 acredita-se estar livre de perigo.



[obm-l] Re: [obm-l] Re: [obm-l] Problema de função elementar

2017-07-17 Por tôpico Pedro José
Bom dia!

Há uma restrição para a função ser crescente. Portanto F(1) é máximo e F(1)
= 1, logo não pode ser 87. tem que ser um valor menor ou igual a 1 e maior
ou igual a zero.

Sds,
PJMS

Em 15 de julho de 2017 20:54, Matheus Herculano <
matheusherculan...@gmail.com> escreveu:

> O resultado é 87
>
> Em 13 de jul de 2017 09:51, "Douglas Oliveira de Lima" <
> profdouglaso.del...@gmail.com> escreveu:
>
>> Seja F uma função crescente definida para todo número real x, 0<=x<=1,
>> tal que
>>
>> a)  F(0)=0
>>
>> b)  F(x/3)=F(x)/2
>>
>> c)  F(1-x)=1-F(x)
>>
>> Encontrar F(21/2017).
>>
>>
>> Douglas Oliveira
>>
>> --
>> Esta mensagem foi verificada pelo sistema de antivírus e
>> acredita-se estar livre de perigo.
>
>
> --
> Esta mensagem foi verificada pelo sistema de antivírus e
> acredita-se estar livre de perigo.
>

-- 
Esta mensagem foi verificada pelo sistema de antiv�rus e
 acredita-se estar livre de perigo.



[obm-l] Re: [obm-l] Problema de função elementar

2017-07-15 Por tôpico Matheus Herculano
O resultado é 87

Em 13 de jul de 2017 09:51, "Douglas Oliveira de Lima" <
profdouglaso.del...@gmail.com> escreveu:

> Seja F uma função crescente definida para todo número real x, 0<=x<=1, tal
> que
>
> a)  F(0)=0
>
> b)  F(x/3)=F(x)/2
>
> c)  F(1-x)=1-F(x)
>
> Encontrar F(21/2017).
>
>
> Douglas Oliveira
>
> --
> Esta mensagem foi verificada pelo sistema de antivírus e
> acredita-se estar livre de perigo.

-- 
Esta mensagem foi verificada pelo sistema de antiv�rus e
 acredita-se estar livre de perigo.



[obm-l] Re: [obm-l] Problema de função elementar

2017-07-15 Por tôpico Anderson Torres
F(1) = 1
F(1/3)=1/2, F(2/3)=1/2 - logo, F(x) = 1/2 se x está em [1/3,2/3]

F(1/9)=1/4, F(2/9)=1/4, F(7/9)=3/4, F(8/9)=3/4 logo, F(x) = 1/4 se x
está em [1/9,2/9] e F(x) = 3/4 se x está em [7/9,8/9]

Acho que a ideia é por aí: ver em que terço-médio cairá o valor 21/2017.

Em 13 de julho de 2017 09:31, Douglas Oliveira de Lima
 escreveu:
> Seja F uma função crescente definida para todo número real x, 0<=x<=1, tal
> que
>
> a)  F(0)=0
>
> b)  F(x/3)=F(x)/2
>
> c)  F(1-x)=1-F(x)
>
> Encontrar F(21/2017).
>
>
> Douglas Oliveira
>
>
> --
> Esta mensagem foi verificada pelo sistema de antivírus e
> acredita-se estar livre de perigo.

-- 
Esta mensagem foi verificada pelo sistema de antiv�rus e
 acredita-se estar livre de perigo.


=
Instru��es para entrar na lista, sair da lista e usar a lista em
http://www.mat.puc-rio.br/~obmlistas/obm-l.html
=


Re: [obm-l] Problema estranho

2017-07-15 Por tôpico Anderson Torres
Uma ideia pode ser tentar aproximar os reais para racionais e usar o
argumento das potências, não?

Em 11 de julho de 2017 18:21, Matheus Secco  escreveu:
> Oi Ralph, tava sem tempo de escrever, mas vou aproveitar a deixa porque você
> já fez quase tudo. Acho que dá pra fazer o caso geral usando que os reais
> admitem uma base considerando como um espaço vetorial sobre os racionais.
>
> Em ter, 11 de jul de 2017 às 18:18, Ralph Teixeira 
> escreveu:
>>
>> Bom, eu sei resolver se todos os números forem racionais. Deve ter um
>> jeito de usar isso para o caso geral...
>>
>> A propriedade desse conjunto não se altera se todos os elementos do
>> conjunto forem multiplicados por um mesmo número, nem se a gente somar uma
>> certa constante a todos eles.
>>
>> Assim, *SE* eles forem todos racionais, a gente pode multiplicar todos
>> eles por um m.m.c imenso e supor que são todos inteiros, spdg.
>>
>> Mas então todos teriam que ter a mesma paridade -- afinal a soma de todos
>> eles, menos qualquer um deles, é um número par.
>>
>> Então, enquanto todos forem pares, divida-os por 2; em algum momento,
>> **todos** ficarão ímpares. Quando isso acontecer, some 1, e ficam todos
>> pares. Então divida por 2 de novo, e de novo, até ficarem ímpares, então
>> some 1 de novo, repita e enxágue.
>>
>> Esse processo vai parar? Oras, esses inteiros vão diminuir em módulo
>> até até até cada um deles virar 0, ou 1! De fato, |x|/2<|x| quando
>> x<>0, e |x+1|/2 < |x| para x<>0,1. Então a cada um ou dois passos o valor
>> absoluto de todos eles diminui -- a menos que eles sejam 0 ou 1. Ou seja, em
>> tempo finito, todos eles vão virar 0 ou 1.
>>
>> Agora é fácil -- lembra que todos sempre têm a mesma paridade?? Então são
>> todos 0, ou todos 1.
>>
>> ---///---
>>
>> Para o caso geral, tenho uma ideia, mas não estou com tempo de
>> desenvolvê-la -- será que dá para começar com os reais, e multiplicar todos
>> eles por algum número real imenso de forma que eles sejam quase inteiros
>> (tipo, todos eles a menos de 1/(4n) de algum inteiro)? Talvez dê para
>> mostrar então pela propriedade que eles têm que ser inteiros, ou pelo menos
>> "comensuráveis" e daí matar o problema.
>>
>> Abraço, Ralph.
>>
>>
>>
>> 2017-07-11 15:06 GMT-03:00 Nowras Ali :
>>>
>>> Uma prova por indução me parece o melhor caminho.
>>> O Bernardo já provou para o caso base, basta agora tentar
>>> provar para n+1, assumindo verdadeiro para n. Tentarei resolver
>>> o problema assim que puder.
>>>
>>> Abraços, Nowras.
>>>
>>> Em 9 de julho de 2017 18:54, Otávio Araújo 
>>> escreveu:


 Já tentei isso, porém não parece ajudar em muita coisa  mas de
 qualquer forma obrigado

 > Em 9 de jul de 2017, às 18:00, Bernardo Freitas Paulo da Costa
 >  escreveu:
 >
 > Não pensei muito, mas acho que você deveria tentar provar os casos
 > n=1
 > e n=2 "no braço" para ter a intuição.  E, na verdade, o enunciado
 > deveria ser: dados a_1, a_2, ... a_{2n+1} números reais, não
 > necessariamente distintos, tais que, para cada escolha de 2n dentre
 > eles, é possível separar em dois grupos de n cada, com a mesma soma.
 > (evitando falar de conjuntos, você pode ter à vontade os elementos
 > repetidos).
 >
 > Assim, o caso n=1 fica: temos a_1, a_2, a_3.  Tomando os elementos
 > a_1, a_2, é possĩvel separar em dois grupos de um elemento, com a
 > soma
 > igual.  Logo a_1 = a_2.  Por simetria, a_1 = a_3, e acabou.  Para n=2,
 > dá mais trabalho.
 >
 > 2017-07-08 23:20 GMT+03:00 Otávio Araújo
 > :
 >> Galera, queria que alguém pudesse resolver essa questão pra mim
 >> ( passei muito tempo nela já kkk):
 >> " Seja n um natural positivo e A um conjunto de 2n+1 números
 >> reais, não necessariamente distintos, com a seguinte propriedade:
 >> - Todo subconjunto de A com 2n elementos pode ser particionado em
 >> dois conjuntos de n elementos tais que a soma dos elementos de cada um
 >> desses dois conjuntos de n elementos são iguais.
 >>   Prove que todos os elementos de A são iguais."
 >>
 >>
 >>
 >>
 >>
 >>
 >>
 >> --
 >> Esta mensagem foi verificada pelo sistema de antivírus e
 >> acredita-se estar livre de perigo.
 >>
 >>
 >>
 >> =
 >> Instruções para entrar na lista, sair da lista e usar a lista em
 >> http://www.mat.puc-rio.br/~obmlistas/obm-l.html
 >>
 >> =
 >
 >
 >
 > --
 > Bernardo Freitas Paulo da Costa
 >
 > --
 > Esta mensagem foi verificada pelo sistema de antivírus e
 > acredita-se estar livre de perigo.
 >

Re: [obm-l] Problema estranho

2017-07-11 Por tôpico Matheus Secco
Oi Ralph, tava sem tempo de escrever, mas vou aproveitar a deixa porque
você já fez quase tudo. Acho que dá pra fazer o caso geral usando que os
reais admitem uma base considerando como um espaço vetorial sobre os
racionais.
Em ter, 11 de jul de 2017 às 18:18, Ralph Teixeira 
escreveu:

> Bom, eu sei resolver se todos os números forem racionais. Deve ter um
> jeito de usar isso para o caso geral...
>
> A propriedade desse conjunto não se altera se todos os elementos do
> conjunto forem multiplicados por um mesmo número, nem se a gente somar uma
> certa constante a todos eles.
>
> Assim, *SE* eles forem todos racionais, a gente pode multiplicar todos
> eles por um m.m.c imenso e supor que são todos inteiros, spdg.
>
> Mas então todos teriam que ter a mesma paridade -- afinal a soma de todos
> eles, menos qualquer um deles, é um número par.
>
> Então, enquanto todos forem pares, divida-os por 2; em algum momento,
> **todos** ficarão ímpares. Quando isso acontecer, some 1, e ficam todos
> pares. Então divida por 2 de novo, e de novo, até ficarem ímpares, então
> some 1 de novo, repita e enxágue.
>
> Esse processo vai parar? Oras, esses inteiros vão diminuir em módulo
> até até até cada um deles virar 0, ou 1! De fato, |x|/2<|x| quando
> x<>0, e |x+1|/2 < |x| para x<>0,1. Então a cada um ou dois passos o valor
> absoluto de todos eles diminui -- a menos que eles sejam 0 ou 1. Ou seja,
> em tempo finito, todos eles vão virar 0 ou 1.
>
> Agora é fácil -- lembra que todos sempre têm a mesma paridade?? Então são
> todos 0, ou todos 1.
>
> ---///---
>
> Para o caso geral, tenho uma ideia, mas não estou com tempo de
> desenvolvê-la -- será que dá para começar com os reais, e multiplicar todos
> eles por algum número real imenso de forma que eles sejam quase inteiros
> (tipo, todos eles a menos de 1/(4n) de algum inteiro)? Talvez dê para
> mostrar então pela propriedade que eles têm que ser inteiros, ou pelo menos
> "comensuráveis" e daí matar o problema.
>
> Abraço, Ralph.
>
>
>
> 2017-07-11 15:06 GMT-03:00 Nowras Ali :
>
>> Uma prova por indução me parece o melhor caminho.
>> O Bernardo já provou para o caso base, basta agora tentar
>> provar para n+1, assumindo verdadeiro para n. Tentarei resolver
>> o problema assim que puder.
>>
>> Abraços, Nowras.
>>
>> Em 9 de julho de 2017 18:54, Otávio Araújo 
>> escreveu:
>>
>>>
>>> Já tentei isso, porém não parece ajudar em muita coisa  mas de
>>> qualquer forma obrigado
>>>
>>> > Em 9 de jul de 2017, às 18:00, Bernardo Freitas Paulo da Costa <
>>> bernardo...@gmail.com> escreveu:
>>> >
>>> > Não pensei muito, mas acho que você deveria tentar provar os casos
>>> n=1
>>> > e n=2 "no braço" para ter a intuição.  E, na verdade, o enunciado
>>> > deveria ser: dados a_1, a_2, ... a_{2n+1} números reais, não
>>> > necessariamente distintos, tais que, para cada escolha de 2n dentre
>>> > eles, é possível separar em dois grupos de n cada, com a mesma soma.
>>> > (evitando falar de conjuntos, você pode ter à vontade os elementos
>>> > repetidos).
>>> >
>>> > Assim, o caso n=1 fica: temos a_1, a_2, a_3.  Tomando os elementos
>>> > a_1, a_2, é possĩvel separar em dois grupos de um elemento, com a
>>> soma
>>> > igual.  Logo a_1 = a_2.  Por simetria, a_1 = a_3, e acabou.  Para n=2,
>>> > dá mais trabalho.
>>> >
>>> > 2017-07-08 23:20 GMT+03:00 Otávio Araújo >> >:
>>> >> Galera, queria que alguém pudesse resolver essa questão pra mim
>>> ( passei muito tempo nela já kkk):
>>> >> " Seja n um natural positivo e A um conjunto de 2n+1 números
>>> reais, não necessariamente distintos, com a seguinte propriedade:
>>> >> - Todo subconjunto de A com 2n elementos pode ser particionado em
>>> dois conjuntos de n elementos tais que a soma dos elementos de cada um
>>> desses dois conjuntos de n elementos são iguais.
>>> >>   Prove que todos os elementos de A são iguais."
>>> >>
>>> >>
>>> >>
>>> >>
>>> >>
>>> >>
>>> >>
>>> >> --
>>> >> Esta mensagem foi verificada pelo sistema de antivírus e
>>> >> acredita-se estar livre de perigo.
>>> >>
>>> >>
>>> >>
>>> =
>>> >> Instruções para entrar na lista, sair da lista e usar a lista em
>>> >> http://www.mat.puc-rio.br/~obmlistas/obm-l.html
>>> >>
>>> =
>>> >
>>> >
>>> >
>>> > --
>>> > Bernardo Freitas Paulo da Costa
>>> >
>>> > --
>>> > Esta mensagem foi verificada pelo sistema de antivírus e
>>> > acredita-se estar livre de perigo.
>>> >
>>> >
>>> >
>>> =
>>> > Instruções para entrar na lista, sair da lista e usar a lista em
>>> > http://www.mat.puc-rio.br/~obmlistas/obm-l.html
>>> >
>>> =
>>>
>>> --
>>> Esta mensagem foi 

Re: [obm-l] Problema estranho

2017-07-11 Por tôpico Ralph Teixeira
Ah, melhor ainda: depois que seus números forem inteiros, some uma certa
constante a todos eles de forma que um deles seja 0. Agora divida por 2,
quantas vezes você quiser (eles vão ser sempre todos pares pelo argumento
de paridade anterior!). Então são todos inteiros divisíveis por poências
arbitrariamente grandes de 2 Pode isso, Arnaldo? Bom, pode, mas só tem
um jeito -- são todos 0.

2017-07-11 18:01 GMT-03:00 Ralph Teixeira :

> Bom, eu sei resolver se todos os números forem racionais. Deve ter um
> jeito de usar isso para o caso geral...
>
> A propriedade desse conjunto não se altera se todos os elementos do
> conjunto forem multiplicados por um mesmo número, nem se a gente somar uma
> certa constante a todos eles.
>
> Assim, *SE* eles forem todos racionais, a gente pode multiplicar todos
> eles por um m.m.c imenso e supor que são todos inteiros, spdg.
>
> Mas então todos teriam que ter a mesma paridade -- afinal a soma de todos
> eles, menos qualquer um deles, é um número par.
>
> Então, enquanto todos forem pares, divida-os por 2; em algum momento,
> **todos** ficarão ímpares. Quando isso acontecer, some 1, e ficam todos
> pares. Então divida por 2 de novo, e de novo, até ficarem ímpares, então
> some 1 de novo, repita e enxágue.
>
> Esse processo vai parar? Oras, esses inteiros vão diminuir em módulo
> até até até cada um deles virar 0, ou 1! De fato, |x|/2<|x| quando
> x<>0, e |x+1|/2 < |x| para x<>0,1. Então a cada um ou dois passos o valor
> absoluto de todos eles diminui -- a menos que eles sejam 0 ou 1. Ou seja,
> em tempo finito, todos eles vão virar 0 ou 1.
>
> Agora é fácil -- lembra que todos sempre têm a mesma paridade?? Então são
> todos 0, ou todos 1.
>
> ---///---
>
> Para o caso geral, tenho uma ideia, mas não estou com tempo de
> desenvolvê-la -- será que dá para começar com os reais, e multiplicar todos
> eles por algum número real imenso de forma que eles sejam quase inteiros
> (tipo, todos eles a menos de 1/(4n) de algum inteiro)? Talvez dê para
> mostrar então pela propriedade que eles têm que ser inteiros, ou pelo menos
> "comensuráveis" e daí matar o problema.
>
> Abraço, Ralph.
>
>
>
> 2017-07-11 15:06 GMT-03:00 Nowras Ali :
>
>> Uma prova por indução me parece o melhor caminho.
>> O Bernardo já provou para o caso base, basta agora tentar
>> provar para n+1, assumindo verdadeiro para n. Tentarei resolver
>> o problema assim que puder.
>>
>> Abraços, Nowras.
>>
>> Em 9 de julho de 2017 18:54, Otávio Araújo 
>> escreveu:
>>
>>>
>>> Já tentei isso, porém não parece ajudar em muita coisa  mas de
>>> qualquer forma obrigado
>>>
>>> > Em 9 de jul de 2017, às 18:00, Bernardo Freitas Paulo da Costa <
>>> bernardo...@gmail.com> escreveu:
>>> >
>>> > Não pensei muito, mas acho que você deveria tentar provar os casos
>>> n=1
>>> > e n=2 "no braço" para ter a intuição.  E, na verdade, o enunciado
>>> > deveria ser: dados a_1, a_2, ... a_{2n+1} números reais, não
>>> > necessariamente distintos, tais que, para cada escolha de 2n dentre
>>> > eles, é possível separar em dois grupos de n cada, com a mesma soma.
>>> > (evitando falar de conjuntos, você pode ter à vontade os elementos
>>> > repetidos).
>>> >
>>> > Assim, o caso n=1 fica: temos a_1, a_2, a_3.  Tomando os elementos
>>> > a_1, a_2, é possĩvel separar em dois grupos de um elemento, com a
>>> soma
>>> > igual.  Logo a_1 = a_2.  Por simetria, a_1 = a_3, e acabou.  Para n=2,
>>> > dá mais trabalho.
>>> >
>>> > 2017-07-08 23:20 GMT+03:00 Otávio Araújo >> >:
>>> >> Galera, queria que alguém pudesse resolver essa questão pra mim
>>> ( passei muito tempo nela já kkk):
>>> >> " Seja n um natural positivo e A um conjunto de 2n+1 números
>>> reais, não necessariamente distintos, com a seguinte propriedade:
>>> >> - Todo subconjunto de A com 2n elementos pode ser particionado em
>>> dois conjuntos de n elementos tais que a soma dos elementos de cada um
>>> desses dois conjuntos de n elementos são iguais.
>>> >>   Prove que todos os elementos de A são iguais."
>>> >>
>>> >>
>>> >>
>>> >>
>>> >>
>>> >>
>>> >>
>>> >> --
>>> >> Esta mensagem foi verificada pelo sistema de antivírus e
>>> >> acredita-se estar livre de perigo.
>>> >>
>>> >>
>>> >> 
>>> =
>>> >> Instruções para entrar na lista, sair da lista e usar a lista em
>>> >> http://www.mat.puc-rio.br/~obmlistas/obm-l.html
>>> >> 
>>> =
>>> >
>>> >
>>> >
>>> > --
>>> > Bernardo Freitas Paulo da Costa
>>> >
>>> > --
>>> > Esta mensagem foi verificada pelo sistema de antivírus e
>>> > acredita-se estar livre de perigo.
>>> >
>>> >
>>> > 
>>> =
>>> > Instruções para entrar na lista, sair da lista e usar a lista em
>>> > 

Re: [obm-l] Problema estranho

2017-07-11 Por tôpico Ralph Teixeira
Bom, eu sei resolver se todos os números forem racionais. Deve ter um jeito
de usar isso para o caso geral...

A propriedade desse conjunto não se altera se todos os elementos do
conjunto forem multiplicados por um mesmo número, nem se a gente somar uma
certa constante a todos eles.

Assim, *SE* eles forem todos racionais, a gente pode multiplicar todos eles
por um m.m.c imenso e supor que são todos inteiros, spdg.

Mas então todos teriam que ter a mesma paridade -- afinal a soma de todos
eles, menos qualquer um deles, é um número par.

Então, enquanto todos forem pares, divida-os por 2; em algum momento,
**todos** ficarão ímpares. Quando isso acontecer, some 1, e ficam todos
pares. Então divida por 2 de novo, e de novo, até ficarem ímpares, então
some 1 de novo, repita e enxágue.

Esse processo vai parar? Oras, esses inteiros vão diminuir em módulo
até até até cada um deles virar 0, ou 1! De fato, |x|/2<|x| quando
x<>0, e |x+1|/2 < |x| para x<>0,1. Então a cada um ou dois passos o valor
absoluto de todos eles diminui -- a menos que eles sejam 0 ou 1. Ou seja,
em tempo finito, todos eles vão virar 0 ou 1.

Agora é fácil -- lembra que todos sempre têm a mesma paridade?? Então são
todos 0, ou todos 1.

---///---

Para o caso geral, tenho uma ideia, mas não estou com tempo de
desenvolvê-la -- será que dá para começar com os reais, e multiplicar todos
eles por algum número real imenso de forma que eles sejam quase inteiros
(tipo, todos eles a menos de 1/(4n) de algum inteiro)? Talvez dê para
mostrar então pela propriedade que eles têm que ser inteiros, ou pelo menos
"comensuráveis" e daí matar o problema.

Abraço, Ralph.



2017-07-11 15:06 GMT-03:00 Nowras Ali :

> Uma prova por indução me parece o melhor caminho.
> O Bernardo já provou para o caso base, basta agora tentar
> provar para n+1, assumindo verdadeiro para n. Tentarei resolver
> o problema assim que puder.
>
> Abraços, Nowras.
>
> Em 9 de julho de 2017 18:54, Otávio Araújo 
> escreveu:
>
>>
>> Já tentei isso, porém não parece ajudar em muita coisa  mas de
>> qualquer forma obrigado
>>
>> > Em 9 de jul de 2017, às 18:00, Bernardo Freitas Paulo da Costa <
>> bernardo...@gmail.com> escreveu:
>> >
>> > Não pensei muito, mas acho que você deveria tentar provar os casos n=1
>> > e n=2 "no braço" para ter a intuição.  E, na verdade, o enunciado
>> > deveria ser: dados a_1, a_2, ... a_{2n+1} números reais, não
>> > necessariamente distintos, tais que, para cada escolha de 2n dentre
>> > eles, é possível separar em dois grupos de n cada, com a mesma soma.
>> > (evitando falar de conjuntos, você pode ter à vontade os elementos
>> > repetidos).
>> >
>> > Assim, o caso n=1 fica: temos a_1, a_2, a_3.  Tomando os elementos
>> > a_1, a_2, é possĩvel separar em dois grupos de um elemento, com a soma
>> > igual.  Logo a_1 = a_2.  Por simetria, a_1 = a_3, e acabou.  Para n=2,
>> > dá mais trabalho.
>> >
>> > 2017-07-08 23:20 GMT+03:00 Otávio Araújo :
>> >> Galera, queria que alguém pudesse resolver essa questão pra mim
>> ( passei muito tempo nela já kkk):
>> >> " Seja n um natural positivo e A um conjunto de 2n+1 números reais,
>> não necessariamente distintos, com a seguinte propriedade:
>> >> - Todo subconjunto de A com 2n elementos pode ser particionado em dois
>> conjuntos de n elementos tais que a soma dos elementos de cada um desses
>> dois conjuntos de n elementos são iguais.
>> >>   Prove que todos os elementos de A são iguais."
>> >>
>> >>
>> >>
>> >>
>> >>
>> >>
>> >>
>> >> --
>> >> Esta mensagem foi verificada pelo sistema de antivírus e
>> >> acredita-se estar livre de perigo.
>> >>
>> >>
>> >> 
>> =
>> >> Instruções para entrar na lista, sair da lista e usar a lista em
>> >> http://www.mat.puc-rio.br/~obmlistas/obm-l.html
>> >> 
>> =
>> >
>> >
>> >
>> > --
>> > Bernardo Freitas Paulo da Costa
>> >
>> > --
>> > Esta mensagem foi verificada pelo sistema de antivírus e
>> > acredita-se estar livre de perigo.
>> >
>> >
>> > 
>> =
>> > Instruções para entrar na lista, sair da lista e usar a lista em
>> > http://www.mat.puc-rio.br/~obmlistas/obm-l.html
>> > 
>> =
>>
>> --
>> Esta mensagem foi verificada pelo sistema de antivírus e
>>  acredita-se estar livre de perigo.
>>
>>
>> =
>> Instru�ões para entrar na lista, sair da lista e usar a lista em
>> http://www.mat.puc-rio.br/~obmlistas/obm-l.html
>> =
>>
>
>
> --
> Esta mensagem foi verificada pelo sistema de antivírus e
> acredita-se estar livre de perigo.


Re: [obm-l] Problema estranho

2017-07-11 Por tôpico Nowras Ali
Uma prova por indução me parece o melhor caminho.
O Bernardo já provou para o caso base, basta agora tentar
provar para n+1, assumindo verdadeiro para n. Tentarei resolver
o problema assim que puder.

Abraços, Nowras.

Em 9 de julho de 2017 18:54, Otávio Araújo 
escreveu:

>
> Já tentei isso, porém não parece ajudar em muita coisa  mas de
> qualquer forma obrigado
>
> > Em 9 de jul de 2017, às 18:00, Bernardo Freitas Paulo da Costa <
> bernardo...@gmail.com> escreveu:
> >
> > Não pensei muito, mas acho que você deveria tentar provar os casos n=1
> > e n=2 "no braço" para ter a intuição.  E, na verdade, o enunciado
> > deveria ser: dados a_1, a_2, ... a_{2n+1} números reais, não
> > necessariamente distintos, tais que, para cada escolha de 2n dentre
> > eles, é possível separar em dois grupos de n cada, com a mesma soma.
> > (evitando falar de conjuntos, você pode ter à vontade os elementos
> > repetidos).
> >
> > Assim, o caso n=1 fica: temos a_1, a_2, a_3.  Tomando os elementos
> > a_1, a_2, é possĩvel separar em dois grupos de um elemento, com a soma
> > igual.  Logo a_1 = a_2.  Por simetria, a_1 = a_3, e acabou.  Para n=2,
> > dá mais trabalho.
> >
> > 2017-07-08 23:20 GMT+03:00 Otávio Araújo :
> >> Galera, queria que alguém pudesse resolver essa questão pra mim (
> passei muito tempo nela já kkk):
> >> " Seja n um natural positivo e A um conjunto de 2n+1 números reais,
> não necessariamente distintos, com a seguinte propriedade:
> >> - Todo subconjunto de A com 2n elementos pode ser particionado em dois
> conjuntos de n elementos tais que a soma dos elementos de cada um desses
> dois conjuntos de n elementos são iguais.
> >>   Prove que todos os elementos de A são iguais."
> >>
> >>
> >>
> >>
> >>
> >>
> >>
> >> --
> >> Esta mensagem foi verificada pelo sistema de antivírus e
> >> acredita-se estar livre de perigo.
> >>
> >>
> >> 
> =
> >> Instruções para entrar na lista, sair da lista e usar a lista em
> >> http://www.mat.puc-rio.br/~obmlistas/obm-l.html
> >> 
> =
> >
> >
> >
> > --
> > Bernardo Freitas Paulo da Costa
> >
> > --
> > Esta mensagem foi verificada pelo sistema de antivírus e
> > acredita-se estar livre de perigo.
> >
> >
> > 
> =
> > Instruções para entrar na lista, sair da lista e usar a lista em
> > http://www.mat.puc-rio.br/~obmlistas/obm-l.html
> > 
> =
>
> --
> Esta mensagem foi verificada pelo sistema de antivírus e
>  acredita-se estar livre de perigo.
>
>
> =
> Instru�ões para entrar na lista, sair da lista e usar a lista em
> http://www.mat.puc-rio.br/~obmlistas/obm-l.html
> =
>

-- 
Esta mensagem foi verificada pelo sistema de antiv�rus e
 acredita-se estar livre de perigo.



Re: [obm-l] Problema estranho

2017-07-09 Por tôpico Otávio Araújo

Já tentei isso, porém não parece ajudar em muita coisa  mas de qualquer 
forma obrigado

> Em 9 de jul de 2017, às 18:00, Bernardo Freitas Paulo da Costa 
>  escreveu:
> 
> Não pensei muito, mas acho que você deveria tentar provar os casos n=1
> e n=2 "no braço" para ter a intuição.  E, na verdade, o enunciado
> deveria ser: dados a_1, a_2, ... a_{2n+1} números reais, não
> necessariamente distintos, tais que, para cada escolha de 2n dentre
> eles, é possível separar em dois grupos de n cada, com a mesma soma.
> (evitando falar de conjuntos, você pode ter à vontade os elementos
> repetidos).
> 
> Assim, o caso n=1 fica: temos a_1, a_2, a_3.  Tomando os elementos
> a_1, a_2, é possĩvel separar em dois grupos de um elemento, com a soma
> igual.  Logo a_1 = a_2.  Por simetria, a_1 = a_3, e acabou.  Para n=2,
> dá mais trabalho.
> 
> 2017-07-08 23:20 GMT+03:00 Otávio Araújo :
>> Galera, queria que alguém pudesse resolver essa questão pra mim ( 
>> passei muito tempo nela já kkk):
>> " Seja n um natural positivo e A um conjunto de 2n+1 números reais, 
>> não necessariamente distintos, com a seguinte propriedade:
>> - Todo subconjunto de A com 2n elementos pode ser particionado em dois 
>> conjuntos de n elementos tais que a soma dos elementos de cada um desses 
>> dois conjuntos de n elementos são iguais.
>>   Prove que todos os elementos de A são iguais."
>> 
>> 
>> 
>> 
>> 
>> 
>> 
>> --
>> Esta mensagem foi verificada pelo sistema de antivírus e
>> acredita-se estar livre de perigo.
>> 
>> 
>> =
>> Instruções para entrar na lista, sair da lista e usar a lista em
>> http://www.mat.puc-rio.br/~obmlistas/obm-l.html
>> =
> 
> 
> 
> -- 
> Bernardo Freitas Paulo da Costa
> 
> -- 
> Esta mensagem foi verificada pelo sistema de antivírus e
> acredita-se estar livre de perigo.
> 
> 
> =
> Instruções para entrar na lista, sair da lista e usar a lista em
> http://www.mat.puc-rio.br/~obmlistas/obm-l.html
> =

-- 
Esta mensagem foi verificada pelo sistema de antiv�rus e
 acredita-se estar livre de perigo.


=
Instru��es para entrar na lista, sair da lista e usar a lista em
http://www.mat.puc-rio.br/~obmlistas/obm-l.html
=


Re: [obm-l] Problema estranho

2017-07-09 Por tôpico Bernardo Freitas Paulo da Costa
Não pensei muito, mas acho que você deveria tentar provar os casos n=1
e n=2 "no braço" para ter a intuição.  E, na verdade, o enunciado
deveria ser: dados a_1, a_2, ... a_{2n+1} números reais, não
necessariamente distintos, tais que, para cada escolha de 2n dentre
eles, é possível separar em dois grupos de n cada, com a mesma soma.
(evitando falar de conjuntos, você pode ter à vontade os elementos
repetidos).

Assim, o caso n=1 fica: temos a_1, a_2, a_3.  Tomando os elementos
a_1, a_2, é possĩvel separar em dois grupos de um elemento, com a soma
igual.  Logo a_1 = a_2.  Por simetria, a_1 = a_3, e acabou.  Para n=2,
dá mais trabalho.

2017-07-08 23:20 GMT+03:00 Otávio Araújo :
> Galera, queria que alguém pudesse resolver essa questão pra mim ( passei 
> muito tempo nela já kkk):
> " Seja n um natural positivo e A um conjunto de 2n+1 números reais, não 
> necessariamente distintos, com a seguinte propriedade:
> - Todo subconjunto de A com 2n elementos pode ser particionado em dois 
> conjuntos de n elementos tais que a soma dos elementos de cada um desses dois 
> conjuntos de n elementos são iguais.
>Prove que todos os elementos de A são iguais."
>
>
>
>
>
>
>
> --
> Esta mensagem foi verificada pelo sistema de antivírus e
>  acredita-se estar livre de perigo.
>
>
> =
> Instruções para entrar na lista, sair da lista e usar a lista em
> http://www.mat.puc-rio.br/~obmlistas/obm-l.html
> =



-- 
Bernardo Freitas Paulo da Costa

-- 
Esta mensagem foi verificada pelo sistema de antiv�rus e
 acredita-se estar livre de perigo.


=
Instru��es para entrar na lista, sair da lista e usar a lista em
http://www.mat.puc-rio.br/~obmlistas/obm-l.html
=


Re: [obm-l] Problema estranho

2017-07-09 Por tôpico Luiz Antonio Rodrigues
Olá, Francisco!
Eu também pensei nisso, mas vou consultar o site que o Bruno indicou...
Muito obrigado e um abraço!
Luiz

On Jul 8, 2017 9:13 PM, "Francisco Barreto" 
wrote:

>
> On Sat, 8 Jul 2017 at 20:21 Otávio Araújo 
> wrote:
>
>>
>> O enunciado original eu não vi, quem me falou desse problema foi um amigo
>> meu. assim me perdoe pelo erro grosseiro. Mas considerando esse A um
>> multiconjunto, essa questão é verdadeira ou se tem um contra-exemplo?
>>
>> Em 8 de jul de 2017, às 19:47, Bruno Visnadi 
>> escreveu:
>>
>> Tecnicamente não dá para chamar de conjunto, quando há números
>> repetidos. O correto seria Multiconjunto:Â https://pt.wikipedia.org/wiki/
>> Multiconjunto
>>
>> Em 8 de julho de 2017 19:27, Luiz Antonio Rodrigues <
>> rodrigue...@gmail.com> escreveu:
>>
>> Luiz, arrisco dizer que pode, mas é equivalente a {1,2,3}. Alguem me
> corrija se eu estiver errado, por favor.
>
>> Olá, Otávio!
>>> Desculpe a intromissão. Eu não sei como resolver seu problema, mas
>>> quero aproveitá-lo para colocar uma questão que me atormenta desde a
>>> faculdade: pode existir um conjunto {1,1,1,2,3}? O número 1 não é único?
>>> Um abraço!
>>>
>>
> Luiz
>>>
>>> On Jul 8, 2017 5:35 PM, "Otávio Araújo" 
>>> wrote:
>>>
>>> Galera, queria que alguém pudesse resolver essa questão pra mim (
>>> passei muito tempo nela já kkk):
>>> " Seja n um natural positivo e A um conjunto de 2n+1 números reais,
>>> não necessariamente distintos, com a seguinte propriedade:
>>> - Todo subconjunto de A com 2n elementos pode ser particionado em dois
>>> conjuntos de n elementos tais que a soma dos elementos de cada um desses
>>> dois conjuntos de n elementos são iguais.
>>>    Prove que todos os elementos de A são iguais."
>>>
>>>
>>>
>>>
>>>
>>>
>>>
>>> --
>>> Esta mensagem foi verificada pelo sistema de antivírus e
>>> Â acredita-se estar livre de perigo.
>>>
>>>
>>> 
>>> =
>>> Instruções para entrar na lista, sair da lista e usar a lista em
>>> http://www.mat.puc-rio.br/~obmlistas/obm-l.html
>>> 
>>> =
>>>
>>>
>>>
>>> --
>>> Esta mensagem foi verificada pelo sistema de antivírus e
>>>
>> acredita-se estar livre de perigo.
>>>
>>
>> --
>> Esta mensagem foi verificada pelo sistema de antivírus e
>> acredita-se estar livre de perigo.
>>
>> --
>> Esta mensagem foi verificada pelo sistema de antivírus e
>> acredita-se estar livre de perigo.
>>
>
> --
> Esta mensagem foi verificada pelo sistema de antivírus e
> acredita-se estar livre de perigo.

-- 
Esta mensagem foi verificada pelo sistema de antiv�rus e
 acredita-se estar livre de perigo.



Re: [obm-l] Problema estranho

2017-07-09 Por tôpico Luiz Antonio Rodrigues
Olá, Bruno!
Muito obrigado pelo esclarecimento!
Um abraço!
Luiz

On Jul 8, 2017 8:01 PM, "Bruno Visnadi"  wrote:

> Tecnicamente não dá para chamar de conjunto, quando há números repetidos.
> O correto seria Multiconjunto: https://pt.wikipedia.org/wiki/Multiconjunto
>
> Em 8 de julho de 2017 19:27, Luiz Antonio Rodrigues  > escreveu:
>
>> Olá, Otávio!
>> Desculpe a intromissão. Eu não sei como resolver seu problema, mas quero
>> aproveitá-lo para colocar uma questão que me atormenta desde a faculdade:
>> pode existir um conjunto {1,1,1,2,3}? O número 1 não é único?
>> Um abraço!
>> Luiz
>>
>> On Jul 8, 2017 5:35 PM, "Otávio Araújo" 
>> wrote:
>>
>> Galera, queria que alguém pudesse resolver essa questão pra mim (
>> passei muito tempo nela já kkk):
>> " Seja n um natural positivo e A um conjunto de 2n+1 números reais, não
>> necessariamente distintos, com a seguinte propriedade:
>> - Todo subconjunto de A com 2n elementos pode ser particionado em dois
>> conjuntos de n elementos tais que a soma dos elementos de cada um desses
>> dois conjuntos de n elementos são iguais.
>>Prove que todos os elementos de A são iguais."
>>
>>
>>
>>
>>
>>
>>
>> --
>> Esta mensagem foi verificada pelo sistema de antivírus e
>>  acredita-se estar livre de perigo.
>>
>>
>> =
>> Instruções para entrar na lista, sair da lista e usar a lista em
>> http://www.mat.puc-rio.br/~obmlistas/obm-l.html
>> =
>>
>>
>>
>> --
>> Esta mensagem foi verificada pelo sistema de antivírus e
>> acredita-se estar livre de perigo.
>>
>
>
> --
> Esta mensagem foi verificada pelo sistema de antivírus e
> acredita-se estar livre de perigo.

-- 
Esta mensagem foi verificada pelo sistema de antiv�rus e
 acredita-se estar livre de perigo.



Re: [obm-l] Problema estranho

2017-07-08 Por tôpico Francisco Barreto
On Sat, 8 Jul 2017 at 20:21 Otávio Araújo  wrote:

>
> O enunciado original eu não vi, quem me falou desse problema foi um amigo
> meu. assim me perdoe pelo erro grosseiro. Mas considerando esse A um
> multiconjunto, essa questão é verdadeira ou se tem um contra-exemplo?
>
> Em 8 de jul de 2017, às 19:47, Bruno Visnadi 
> escreveu:
>
> Tecnicamente não dá para chamar de conjunto, quando há números
> repetidos. O correto seria Multiconjunto:Â
> https://pt.wikipedia.org/wiki/Multiconjunto
>
> Em 8 de julho de 2017 19:27, Luiz Antonio Rodrigues  > escreveu:
>
> Luiz, arrisco dizer que pode, mas é equivalente a {1,2,3}. Alguem me
corrija se eu estiver errado, por favor.

> Olá, Otávio!
>> Desculpe a intromissão. Eu não sei como resolver seu problema, mas
>> quero aproveitá-lo para colocar uma questão que me atormenta desde a
>> faculdade: pode existir um conjunto {1,1,1,2,3}? O número 1 não é único?
>> Um abraço!
>>
>
Luiz
>>
>> On Jul 8, 2017 5:35 PM, "Otávio Araújo" 
>> wrote:
>>
>> Galera, queria que alguém pudesse resolver essa questão pra mim (
>> passei muito tempo nela já kkk):
>> " Seja n um natural positivo e A um conjunto de 2n+1 números reais,
>> não necessariamente distintos, com a seguinte propriedade:
>> - Todo subconjunto de A com 2n elementos pode ser particionado em dois
>> conjuntos de n elementos tais que a soma dos elementos de cada um desses
>> dois conjuntos de n elementos são iguais.
>>    Prove que todos os elementos de A são iguais."
>>
>>
>>
>>
>>
>>
>>
>> --
>> Esta mensagem foi verificada pelo sistema de antivírus e
>> Â acredita-se estar livre de perigo.
>>
>>
>> =
>> Instruções para entrar na lista, sair da lista e usar a lista em
>> http://www.mat.puc-rio.br/~obmlistas/obm-l.html
>> =
>>
>>
>>
>> --
>> Esta mensagem foi verificada pelo sistema de antivírus e
>>
> acredita-se estar livre de perigo.
>>
>
> --
> Esta mensagem foi verificada pelo sistema de antivírus e
> acredita-se estar livre de perigo.
>
> --
> Esta mensagem foi verificada pelo sistema de antivírus e
> acredita-se estar livre de perigo.
>

-- 
Esta mensagem foi verificada pelo sistema de antiv�rus e
 acredita-se estar livre de perigo.



Re: [obm-l] Problema estranho

2017-07-08 Por tôpico Francisco Barreto
On Sat, 8 Jul 2017 at 17:35 Otávio Araújo  wrote:

> Galera, queria que alguém pudesse resolver essa questão pra mim ( passei
> muito tempo nela já kkk):
> " Seja n um natural positivo e A um conjunto de 2n+1 números reais, não
> necessariamente distintos, com a seguinte propriedade:
> - Todo subconjunto de A com 2n elementos pode ser particionado em dois
> conjuntos de n elementos tais que a soma dos elementos de cada um desses
> dois conjuntos de n elementos são iguais.
>Prove que todos os elementos de A são iguais."
>
>
>
> Pegue o conjunto de 2n elementos, ordene de tal forma que você pega pares,
> o menor e o maior, o segundo menor e o segundo maior.

Vai colocando um elemento de cada par em uma urna distinta. Minha primeira
fungada no problema seria lembrar da soma dos n primeiros termos de uma
sequencia. Espero que faça sentido. Fui

>
>
>
>
> --
> Esta mensagem foi verificada pelo sistema de antivírus e
>  acredita-se estar livre de perigo.
>
>
> =
> Instruções para entrar na lista, sair da lista e usar a lista em
> http://www.mat.puc-rio.br/~obmlistas/obm-l.html
> =
>

-- 
Esta mensagem foi verificada pelo sistema de antiv�rus e
 acredita-se estar livre de perigo.



Re: [obm-l] Problema estranho

2017-07-08 Por tôpico Otávio Araújo

O enunciado original eu não vi, quem me falou desse problema foi um amigo meu. 
assim me perdoe pelo erro grosseiro. Mas considerando esse A um multiconjunto, 
essa questão é verdadeira ou se tem um contra-exemplo?

> Em 8 de jul de 2017, às 19:47, Bruno Visnadi  
> escreveu:
> 
> Tecnicamente não dá para chamar de conjunto, quando há números repetidos. 
> O correto seria Multiconjunto:Â https://pt.wikipedia.org/wiki/Multiconjunto
> 
> Em 8 de julho de 2017 19:27, Luiz Antonio Rodrigues  
> escreveu:
>> Olá, Otávio!
>> Desculpe a intromissão. Eu não sei como resolver seu problema, mas quero 
>> aproveitá-lo para colocar uma questão que me atormenta desde a faculdade: 
>> pode existir um conjunto {1,1,1,2,3}? O número 1 não é único?
>> Um abraço!
>> Luiz
>> 
>> On Jul 8, 2017 5:35 PM, "Otávio Araújo"  wrote:
>> Galera, queria que alguém pudesse resolver essa questão pra mim ( 
>> passei muito tempo nela já kkk):
>> " Seja n um natural positivo e A um conjunto de 2n+1 números reais, 
>> não necessariamente distintos, com a seguinte propriedade:
>> - Todo subconjunto de A com 2n elementos pode ser particionado em dois 
>> conjuntos de n elementos tais que a soma dos elementos de cada um desses 
>> dois conjuntos de n elementos são iguais.
>>    Prove que todos os elementos de A são iguais."
>> 
>> 
>> 
>> 
>> 
>> 
>> 
>> --
>> Esta mensagem foi verificada pelo sistema de antivírus e
>> Â acredita-se estar livre de perigo.
>> 
>> 
>> =
>> Instruções para entrar na lista, sair da lista e usar a lista em
>> http://www.mat.puc-rio.br/~obmlistas/obm-l.html
>> =
>> 
>> 
>> -- 
>> Esta mensagem foi verificada pelo sistema de antivírus e 
>> acredita-se estar livre de perigo.
> 
> 
> -- 
> Esta mensagem foi verificada pelo sistema de antivírus e 
> acredita-se estar livre de perigo.

-- 
Esta mensagem foi verificada pelo sistema de antiv�rus e
 acredita-se estar livre de perigo.



Re: [obm-l] Problema estranho

2017-07-08 Por tôpico Bruno Visnadi
Tecnicamente não dá para chamar de conjunto, quando há números repetidos. O
correto seria Multiconjunto: https://pt.wikipedia.org/wiki/Multiconjunto

Em 8 de julho de 2017 19:27, Luiz Antonio Rodrigues 
escreveu:

> Olá, Otávio!
> Desculpe a intromissão. Eu não sei como resolver seu problema, mas quero
> aproveitá-lo para colocar uma questão que me atormenta desde a faculdade:
> pode existir um conjunto {1,1,1,2,3}? O número 1 não é único?
> Um abraço!
> Luiz
>
> On Jul 8, 2017 5:35 PM, "Otávio Araújo"  wrote:
>
> Galera, queria que alguém pudesse resolver essa questão pra mim ( passei
> muito tempo nela já kkk):
> " Seja n um natural positivo e A um conjunto de 2n+1 números reais, não
> necessariamente distintos, com a seguinte propriedade:
> - Todo subconjunto de A com 2n elementos pode ser particionado em dois
> conjuntos de n elementos tais que a soma dos elementos de cada um desses
> dois conjuntos de n elementos são iguais.
>Prove que todos os elementos de A são iguais."
>
>
>
>
>
>
>
> --
> Esta mensagem foi verificada pelo sistema de antivírus e
>  acredita-se estar livre de perigo.
>
>
> =
> Instruções para entrar na lista, sair da lista e usar a lista em
> http://www.mat.puc-rio.br/~obmlistas/obm-l.html
> =
>
>
>
> --
> Esta mensagem foi verificada pelo sistema de antivírus e
> acredita-se estar livre de perigo.
>

-- 
Esta mensagem foi verificada pelo sistema de antiv�rus e
 acredita-se estar livre de perigo.



Re: [obm-l] Problema estranho

2017-07-08 Por tôpico Luiz Antonio Rodrigues
Olá, Otávio!
Desculpe a intromissão. Eu não sei como resolver seu problema, mas quero
aproveitá-lo para colocar uma questão que me atormenta desde a faculdade:
pode existir um conjunto {1,1,1,2,3}? O número 1 não é único?
Um abraço!
Luiz

On Jul 8, 2017 5:35 PM, "Otávio Araújo"  wrote:

Galera, queria que alguém pudesse resolver essa questão pra mim ( passei
muito tempo nela já kkk):
" Seja n um natural positivo e A um conjunto de 2n+1 números reais, não
necessariamente distintos, com a seguinte propriedade:
- Todo subconjunto de A com 2n elementos pode ser particionado em dois
conjuntos de n elementos tais que a soma dos elementos de cada um desses
dois conjuntos de n elementos são iguais.
   Prove que todos os elementos de A são iguais."







--
Esta mensagem foi verificada pelo sistema de antivírus e
 acredita-se estar livre de perigo.


=
Instruções para entrar na lista, sair da lista e usar a lista em
http://www.mat.puc-rio.br/~obmlistas/obm-l.html
=

-- 
Esta mensagem foi verificada pelo sistema de antiv�rus e
 acredita-se estar livre de perigo.



  1   2   3   4   5   6   7   8   9   10   >